202
លឹម សុវណ វិចិរ ករងលំត់គណិ តវិា កមិរវ ិាល័យ - នវនត ាយលើកទី៣ សី ២០១០

Problem Book in Mathematical- Vol I Arithmetic

Embed Size (px)

DESCRIPTION

Problem Book in Mathematical- Vol I Arithmetic

Citation preview

Page 1: Problem Book in Mathematical- Vol I Arithmetic

លម សវណណ វចតរ

កមរងលហាតគណតវទយា

កតមរវទយាលយ

ភាគ ១- នពវនត

ផសាយល ើកទ៣ សហា ២០១០

Page 2: Problem Book in Mathematical- Vol I Arithmetic

blank

Page 3: Problem Book in Mathematical- Vol I Arithmetic

i

លម សវណណ វចតរ

កមរងលហាតគណតវទយា

កតមរវទយាលយ

ភាគ ១- នពវនត

ផសាយល ើកទ៣ សហា ២០១០

Page 4: Problem Book in Mathematical- Vol I Arithmetic

ii

ភាគ១ - នពវនត

ផសាយល ើកទ១ : គណតវទាអឡាពច -នពវនត ២០០៨

ផសាយល ើកទ២ : កមរង ាហាតគណតវទា-នពវនត ២០០៩

ភាគ២ - ពជគណត វភាគ

ដោយ លម សវណណ វចតរ

ទយនាកទយនង

- វវបសាយ http://www.dahlina.com/

- អវមល [email protected]

Page 5: Problem Book in Mathematical- Vol I Arithmetic

iii

<<ប ើផលប ើតបេញព ណយរ សខញ មានមែន ជាតបរោយសែឱយខញ ឆា ែ ប ើតបៅ ែព ញជា

ែដងប ៀត>>

(តពះបាទយឧទយយាទយរយទយពរ, សវ. ទយ១១)

Page 6: Problem Book in Mathematical- Vol I Arithmetic

iv

Page 7: Problem Book in Mathematical- Vol I Arithmetic

v

មាតកា វផែកទយ១ តទយសតបទយ

១. នយមនយ ទរសដបរ ................................................................................... 1 ១.១ នយមនយ ................................................................................... 1 ១.២ គោលការណទរងទាប ................................................................ 3 ១.៣ លកខខណឌ ចាបាចនងទរបទោន ....................................................... 4 ២. ភាពចចកដាច ............................................................................................. 7 ២.១ ភាពចចកដាច .............................................................................. 7 ២.២ នយមនយ................................................................................... 7 ២.៣ ចាននបឋម .................................................................................. 7 ២.៤ ទរសដបរ ..................................................................................... 9 ២.៥ ទរសដបរ ..................................................................................... 9 ២.៦ វធចចកចបបអរលដ ........................................................................ 12 ២.៧ ភាពសមមល ............................................................................... 14 ២.៨ ទរសដបរ ..................................................................................... 14 ២.៩ ភាពចចកដាចនង ៩ ...................................................................... 16 ៣. ការបាចបកជាកតតា ផលរណ .......................................................................... 17 ៣.១ តចចករមធាបាផត PGCD .............................................................. 17 ៣.២ ពហរណរមតចបាផត PPCM ....................................................... 18 ៣.៣ ទរសដបរ Bachet-Bézout ........................................................... 19 ៣.៤ ករ ចលរអរលដ ............................................................................... 20 ៣.៥ ទរសដបរ ..................................................................................... 20

Page 8: Problem Book in Mathematical- Vol I Arithmetic

vi

៣.៦ ទរសដបរ ..................................................................................... 22 ៣.៧ ទរសដបរអរលដ ............................................................................. 23 ៣.៨ ទរសដបរ ..................................................................................... 23 ៣.៩ ទរសដបរ ..................................................................................... 24 ៣.១០ ទរសដបរ .................................................................................. 24 ៣.១១ ទរសដបរ ................................................................................. 25 ៣.១២ ទរសដបរ ................................................................................. 25 ៣.១៣ ទរសដបរ ................................................................................. 26 ៣.១៤ ទបមាណវធចចកចបបអរលដ ........................................................ 28 ៣.១៥ ទរសដបរ .................................................................................. 28 ៣.១៦ ទរសដបរ .................................................................................. 28 ៣.១៧ ទរសដបរ ................................................................................. 30 ៤. ភាពសមមលកនងសាណា ចាននរត .................................................................... 31 ៤.១ សមការសមមលលគនចអែរ ............................................................ 31 ៤.២ ទរសដបរ ..................................................................................... 31 ៤.៣ ទរសដបរ ..................................................................................... 33 ៤.៤ ករ ចលរ ........................................................................................ 33 ៤.៥ ទរសដបរទបបន ............................................................................ 34 ៤.៦ ទរសដបរ ...................................................................................... 36 ៤.៧ ទរសដបរសាណលចចកចន .............................................................. 36 ៤.៨ ទរសដបរវលសន .......................................................................... 37 ៤.៩ កនទរសដបរចភមា ......................................................................... 39

Page 9: Problem Book in Mathematical- Vol I Arithmetic

vii

៤.១០ វបាក ........................................................................................ 40 ៥. ចផនករត ...................................................................................................... 41 ៥.១ ចផនករត ចផនករសភារ .................................................................. 41 ៥.២ ទរសដបរ ..................................................................................... 41 ៥.៣ ទរសដបរ ..................................................................................... 43 ៥.4 ទរសដបរឌប លញ ក ...................................................................... 44 ៦. អនរមននពវនា ............................................................................................. 47 ៦.១ អនរមននពវនា .............................................................................. 47 ៦.២ ទរសដបរ ..................................................................................... 48 ៦.៣ ទរសដបរអចល ............................................................................. 52 ៦.៤ ទរសដបរ ...................................................................................... 53 ៦.៥ ទរសដបរ ...................................................................................... 54 ៦.៦ ទរសដបរអចល ............................................................................. 55 ៦.៧ ទរសដបរឡសង ........................................................................... 57 ៦.៨ ទរសដបរខាមរ ............................................................................. 58 ៦.៩ ទរសដបរ ...................................................................................... 59

វផែកទយ២ លហារ

ឡសច ........................................................................................................... 61 ទបពនធរបាប ចាននរត ភាពចចកដាច .................................................................. 67 ទបពនធរបាប ......................................................................................... 67 ចាននរត .............................................................................................. 68 ភាពចចកដាច ....................................................................................... 69

Page 10: Problem Book in Mathematical- Vol I Arithmetic

viii

សាណល ភាពសមមល .......................................................................... 72 ចាននកាគរ ............................................................................................ 73 ចាននចដលមានរាងណាមយ ................................................................... 74 ពហរណរមតចបាផត តចចករមធាបាផត .............................................................. 77 ចាននបឋម ........................................................................................... 77 ចាននពហរណ ..................................................................................... 77 ពហរណរមតចបាផត តចចករមធាបាផត .................................................. 78 បាចបកជាកតតា បឋម .............................................................................. 79 ចផនករត ........................................................................................................... 81 សមការដយផង .................................................................................................. 83

វផែកទយ៣ ដដ ះតសាយ

ឡសច ........................................................................................................... 89 ទបពនធរបាប ចាននរត ភាពចចកដាច .................................................................. 107 ទបពនធរបាប ......................................................................................... 107 ចាននរត .............................................................................................. 112 ភាពចចកដាច ....................................................................................... 114 សាណល ភាពសមមល .......................................................................... 130 ចាននកាគរ ............................................................................................ 131 ចាននចដលមានរាងណាមយ ................................................................... 138 ពហរណរមតចបាផត តចចករមធាបាផត .............................................................. 143 ចាននបឋម ........................................................................................... 143 ចាននពហរណ ..................................................................................... 145

Page 11: Problem Book in Mathematical- Vol I Arithmetic

ix

ពហរណរមតចបាផត តចចករមធាបាផត .................................................. 146 បាចបកជាកតតា បឋម .............................................................................. 152 ចផនករត ........................................................................................................... 155 សមការដយផង .................................................................................................. 163

Page 12: Problem Book in Mathematical- Vol I Arithmetic

x

Page 13: Problem Book in Mathematical- Vol I Arithmetic

លម សវណណ វចតរ | 1.1. នយមនយ 1

១ - នយមនយ.ទរសតបរ

នយមនយ

ចននគរធមមជារ ជាបណតា ចនន ។

ចននគរ :

ចននគរវជជមាន

ចននគរអវជជមាន

ចននគរមនអវជជមាន

ចននអសនទាន ជាចននដែលមនអាចសរសសរជារាង បាន ដែល ជាចននគរ។

ចននគរគ រចននគ ជាចននដែលមានរាង ដែល ជាចននគរ។ ឧទាហរណ ជាចនន

គ។

ចននគរសសស រចននសសស ជាចននដែលមានរាង ដែល ជាចននគរ។ ឧទាហរណ

ឧទាហរណ: ប ើ ជា២ចននគត ដែល គ ប ោះ រតវដតមានលកខណៈគបេេែចគនា ។

ចសមលើយ

យ ើងនងបងហា ញថា យបើ ររវតរមានលកខណៈគយេេផទ គនា យ ោះ យេេ។ ដយចាោះ យ ើងេនមរថា ររវតរមានលកខណៈគយេេផទ គនា ។ យ ើងេនមរថា គ យ ើ យេេ។ ដយចាោះមានចននគរ នង តដល នង ។ ដយចាោះ

ជាចននយេេ។

Page 14: Problem Book in Mathematical- Vol I Arithmetic

2 ១ - នយមនយ.តរសាបរ | លម សវណណ វចតរ

ឧទាហរណ: ចរ ងហា ញថា ជាចននអេនទាន។

ចសមលើយ

យ ើងេនមរថា េយណើ ខាងយលើមនពរ។ មានន ថា ជាចននេនទាន ដយចាោះ មានចននគរ តដល ។ កាងចននគរតដលអាចមានទាងយ ោះ េនមរថា ,a b រចជាងយគ។ ររវតរជាចននគ។ តដល ជាចននគរមនេនយ ររវតរជាចននគ។ តដល ជាចននគរមនេនយ។ ដយចាោះ

តដល នង មានន ថា មនតមនជាគរលល

រចបផរយេ ផទ ពេមមរកមម។ មានន ថា ជាចននអេនទាន។

ឧទាហរណ: ចរ ងហា ញថា

ជាចននគតគ បហើយថា

ចប ោះចននគតវជជមាន b នង

ចប ោះចននគត ។

ចសមលើយ

ករណ

គរគ ពរ

ពរ។

េនមរថាពរ រ រដល ដយចាោះ

តដល នង ជាចននគរវជ ជមាន។ យ ើងមាន

Page 15: Problem Book in Mathematical- Vol I Arithmetic

លម សវណណ វចតរ | 1.2. សោលការណតរងតាប 3

ជាចននគរគ។ យ ើ ដចគនា

ដយចាោះេយន ើខាងយលើ ពរ ចយ ោះ ចយ ោះចននគរវជ ជមាន នង ចយ ោះរគបចននគរ ។

សោលការណតរងតាប

របរេងរ ប

សបើមានតាបចនន សហើយ តរងមានរនធចនន ស ោះវាតរវដរមាន

តាបយា ងរច២ ដែលឋរសៅកនងរនធដរមយ។

សោលការណសនោះ សមើលសៅហាកែចជា ងាយណតស នរណតកែងដែរ ដរវាសខានខាល ងណតស។

ឧទាហរណខាងសតកាមនងបងាា ញពតបសយជន របសវា។

Page 16: Problem Book in Mathematical- Vol I Arithmetic

4 ១ - នយមនយ.តរសាបរ | លម សវណណ វចតរ

ឧទាហរណ: តាង ជាេណននចននគត បរជើេបរ ើេបចញព េវវតពជគណត ។ ចរ ងហា ញថា រតវដតមានចននគត បសេងគនា ដែលមានសល កបេមើ ។

ចសមលើយ

យ ើងតចក ធារទាង របេេវរយនោះ យៅជា រកម ។ យោ យ ើងររវយរជើេយរ ើេ កចននគរចនន យចញព េណចនន ខាងយលើ យ ោះតាមយគនលការណរេងរ ប វាររវតរមានចននគរ តដលឋរយៅកាងេណតរម លនបណតា េណខាងយលើ យ ើ តដលមានផលបក ។

លកខខណឌ ចបាចនងតគបតោន

បតើ កយថា លកខខណឌ ចាបាចនងគរបគាន មាននយថាប េច?

លកខខណឌ ចាបាចជាលកខខណឌ ដែលចបាចបែើ បឱយេបណើ យពត។ ដតភាពចបាចបនោះ នរបាកែ

ថារគ រគននបែើ បឱយេបណើ យប ោះពតប ើយ។

ឧទា រណ េយណើ ររយកាណ ប នររយកាណ ។ យបើររយកាណទាងពរប នគនា យ ោះជាចបាចររវតរ មទាងបរបេវាប នគនា ។ តរររយកាណតដលមានមបប នគនា មនរគបរគនននងឱយយ ើងេនាោា នបានថាវាប នគនា បានយេ។ តរយបើមទាងបរបេវាមនប នគនា យ ោះររយកាណពរយនោះកមនប នគនា តដរ ដយចាោះយ ើ បានជាយគនយា ថា វាជាលកខខណឌ ចបាច។ លកខខណឌ ចបាចមនមានតរម យេ។ ករណររយកាណប នគនា យ ើងមានលកខខណឌ ចបាចយរចើន៖

- រជងម ប នគនា - រកលាលផទប នគនា - មម ប នគនា - …។ល។

Page 17: Problem Book in Mathematical- Vol I Arithmetic

លម សវណណ វចតរ | 1.3. លកខខណឌ ចបាចនងតគបតោន 5

ដយចាោះយ ើងយឃើញថា យបើររយកាណពរប នគនា យ ោះ ឱយបណតា លកខខណឌ « រជងម ប នគនា , រកលាលផទប នគនា , មម ប នគនា ,.. » ពរទាងអេ។

លកខខណឌ ចបាច លកខខណឌ រគបរគនន

ឧទា រណ េយណើ ។ យ ើងទាញបានជាចបាច ររវតរធជាងេនយយរ ោះយបើ យ ោះ មនអាចយេមើ បានយេ។ ដយចាោះ ជាលកខខណឌ ចបាចម ។ តរលកខខណឌ មនរគបរគនន ឱយ បានយេ។ យដើមបឱយ ទាលតរ ។ ដយចាោះ ជាលកខខណឌ រគបរគននយដើមបឱយ ។ កាងករណយនោះ យ ើងមានលកខខណឌចបាចយរចើនរាបមនអេ ដចជា ។

កយគនលោះបសេងប ៀត ដែលបយើងបរចើនជ រ ោះមានែចជា “ …បបើ នងមានតែបបើ …” នង

« ..នាឱយ … នង គចសមកវញ »

ពរយបើនងមានតរយបើ ពរ។ ឃលល យនោះមានន ថា មានតរឬទាលតរ ពរ តរប យណតណ ោះ យេើប អាចពរបាន។

ឧទា រណដចជា ពរ យបើនងមានតរយបើ : គនម ន ណតយផេងយេៀរ តដលយផទៀងផទទ រលកខខណឌ យនោះបានយេ។ ឧទា រណម យេៀរ យ ើងមនអាចនយា ថា យបើនងមានតរយបើ បានយេ យរ ោះ កយផទៀងផទទ រ បានតដរ។ តរយ ើងអាចនយា ថា យបើ បាន។

«… យបើ នងមានតរយបើ … » = « …if and only if … » = «…si et seulement si …»

ឧទា រណ យ ើ អងគទាងពរយេមើគនា យបើនងមានតរយបើ ។ ឃលល យនោះមានន ថា គនម នលកខខណឌណតយផេងយេៀរ យរៅព ឧទា រណ តដល ឱយ បានយេ។

Page 18: Problem Book in Mathematical- Vol I Arithmetic

6 ១ - នយមនយ.តរសាបរ | លម សវណណ វចតរ

ចមងល៖ យរើអាចជនេ កយ « យបើនងមានតរយបើ » យោ កយ « ទាលតរ-លោះរតាតរ » បានតដរឬយេ?

ពត ឱយ ពត នងរចេ កវញ។ ឃលល បនោះមាននយថា ប ើ ពត ប ោះ ឱយ ពតដែរ បហើយរចេ កវញ ប ើ ពត ឱយ ពត វញ។ ែបចាោះវាមាននយែចគនា នង ដែរ។

ឧទា រណ ឱយ នងរចេមកវញ។ យរ ោះ

។ រចេមកវញ យបើ យ ោះ ពរ។

ឧទា រណ យ ើងមនអាចនយា ថា » ឱយ នងរចេមកវញ » បានយេ យរ ោះ រចេមកវញ ឱយ តមន តរ កយផទៀងផទទ រលកខខណឌ តដរ។ ដយចាោះឃលល យនោះខេររង « ឱយ » ។ របយយាគតដលររវ ររវនយា ថា « ឱយ នងរចេមកវញ» ។

ឧទា រណ យ ើងមនអាចនយា ថា « ឱយ នងរចេមកវញ » បានយេ យរ ោះ ឱយ តមន តររចេមកវញ កយផទៀងផទទ រលកខខណឌ តដរ។ ដយចាោះឃលល យនោះខេររង «រចេមកវញ ឱយ » ។ របយយាគតដលររវ ររវនយា ថា « ឱយ នងរចេមកវញ» ។

Page 19: Problem Book in Mathematical- Vol I Arithmetic

លម សវណណ វចតរ | 2.1. ភាពចចកដាច 7

២ - ភាពចែកដាែ

ភាពចចកដាច

បបើ ជាចននគរ ប ើងនយា ថា ចចកដាច បបើសនជា មានចននគរ ម

ចែល ។ ប ើងសរបសរថា ។

ជា រចចក របស ។

ជា ពហគណ នន ។

ឧទាហរណ : ចែក ដាែ។ យ ើងមាន ។ នង ជាតចែករបស ។ តចែករបស មាន នង ខលនវា។

ន មន

ចននកាបរ ជាចននចែលមានរាង ចែល ជាចននគរ។

ចននគប ជាចននចែលមានរាង ចែល ជាចននគរ។

ឧទាហរណ : ជាែននកាយរ យររោះ ។ ជាែននកាយរយររោះ ។

ចននបឋម

ចននបឋម ជាចននគរវជជមាន ធជាង ១ ចែលរចចករបសវាមានចរបលខ នង ។

បបើចននគរ មនចមនជាចននបឋម ប ើងបៅវាថា ចននពហគណ ។ ប ើងអាចសរ

បសរ ជា ចែល ។

Page 20: Problem Book in Mathematical- Vol I Arithmetic

8 ២ - ភាពចចកដាច | លម សវណណ វចតរ

ឧទាហរណ ែននបឋមៈ ែននពហគណ មនមមនជាចននបឋម ហ ើយកមនមមនជា ចននព គណមែរ។ ហយើងហ ើញថា ជាចននបឋមគមែមយគែ ហ ើយ នង ជាចននគែែហរៀងគនា មែមយគែ មែលបឋមទង២។

ឧទាហរណ: ចរកណរចននគរវជជមាន ចែល នង ជាចននបឋម។

ែប ោះតា

ែននទាងបបកែលគនន យសមើនង ជាយលខគ។ ដយែនោះកនងែយោមយលខទាងប រតវមានយាងយោែោសម ចដលជាយលខគ។ យ ើងដងថា មានយលខ ចតម បាយោណ ោះ ចដលែននបឋមគ។ យ ើងមាន ជាយលខយសស ដយែនោះែននចដលគរចតយសមើ មានចត នង ។ យបើ ។ យបើ យ ើងទាញបាន ែននទាងបយសមើនង យផទៀងផទទ ត។ យបើ ។ យបើ យ ើងទាញបាន ែននទាងបយសមើនង មនយផទៀងផទទត។ ដយែនោះ ។៚ ឧទាហរណ: [AHSME 1976] បបើ នង ជាចននបឋម នង មានឬសគរពរ

បផេងគនា ចរគណនា នង ។

ែប ោះតា

តាង នង ចដល ជាឬសគតវជ ជមានពរយផេងគនន ។ ដយែនោះ

នង ។ យដា ជាែននបឋម យ ោះ ។ ដយែនោះ នង ។ យដា នង ជាែននបឋមទាងពរ យ ោះ នង ។ ៚

Page 21: Problem Book in Mathematical- Vol I Arithmetic

លម សវណណ វចតរ | 2.4. តរសតបរ 9

តរសតបរ

1° បបើ ជាចននគរចែល បនាោះ ចប ោះតគបចននគរ ។

2° បបើ ជាចននគរចែល បនាោះ ។

3° បបើ នង បនាោះ

4° បបើ នង បនាោះ

តរសតបរ

តគបចននគរវជជមានទាងអស មានចននរចចកវជជមានជាចននគ បលើកចលងចរវាជាចននកាបរ។

សតមា បញជជ ក

តាង ជាែននគតវជ ជមានម ។ យ ើងដងថា យបើ ចែក ដាែ យ ោះ កចែក ដាែចដរ យររោះ

។ ដយែនោះ យបើ ជាតចែករបស យ ោះយ ើងមាន កជាតចែករបស ចដរ។ ករណ

មនចមនជាែននកាយរ យ ើងមាន

ែយរោះរគប ។ ករណ ជាែននកាយរ យ ើងអាែរកបាន

ចដល នង យររោះ ជាែននកាយរ។

ដយែនោះតាមលកខណៈយនោះ យបើ ជាតចែករបសែននគត យ ោះ យគអាែរកបានតចែកយផេងយ ៀតរបស តាងយដា ចដល ។

ករណ មនចមនជាែននកាយរ យ ើងមាន យររោះយបើមាន យ ោះ

ជាែននកាយរ។ ដយែនោះ ែននតចែករបស យសមើ ជាែននគ។ តចែករបស មាន ។

ករណ ជាែននកាយរ យ ើងអាែរកបាន ម ចដល យររោះយបើមាន យ ោះ

ជាែននកាយរ។ ដយែនោះតចែករបស មាន

Page 22: Problem Book in Mathematical- Vol I Arithmetic

10 ២ - ភាពចចកដាច | លម សវណណ វចតរ

។ យ ើងទាញបាន មានតចែកែនន ជាែននយសស។

ឧទាហរណ ែយរោះ ជាែននកាយរ យរៅពយលខ ម យែញ យបើ ជាតចែករបស យ ោះ យ ើងអាែរកបាន ចដលជាតចែករបស ចដរ បានជានែច។ ដែជា ជាតចែកម របស នង

កជាតចែករបស ចដរ។ ជាតចែកម របស នង

កជាតចែក

របស ចដរ។ល។ យដា ផគនន ជាគៗ តចែករបស មានទាងអស

ដយែនោះែននតចែករបសរបស យសមើ ចដលជាែននយសស។

ឧទាហរណ: សសេបគចបរៀនចនននមៃនាក បានបែើរឆលងការរដាកសមាៃ រៈម ជរចែលបរទាា រជរទាង

អស បហើ បងបលខពបលខ១ ែលបលខ២០។ សសេរម បបើកទាា រទាងអសបឡើង។ បនាា បមកសសេ

រពរបរទាា ររបសរទាងអសចែលមានបលខ ២, ៤, ៦, ៨, ១០, ១២, ១៤, ១៦, ១៨, ២០។ សសេរបបបើក

បររទាងអសចែលមានបលខ ៣ ៦ ៩ ១២ ១៥ ១៨ : បបើរ បរ សសេបនាោះបបើក, បបើរ បបើក

សសេបនាោះបរវញ។ សសេបផេងបរៀរបធាើចបបបនោះបនតបនាា បមកបរៀរ បដា សសេរ បបើកបររ

ចែលមានបលខជាពហគណនន : បបើរ បរ សសេបនាោះបបើក, បបើរ បបើក សសេបនាោះបរវញ។

បរើរបលខប នាា នចែលនងចហទាា រ បតកា បពលសសេទាងនមៃនាកបនាោះបធាើការបបើកបរមដងមាា ករចរាល

បហើ ?

ែប ោះតា

យ ើងដងថា នងែហ យបើវារតវយបើកប ែននយសសដង នងថា រតវយបើកប យដា សសេ ទាលចត ជាតចែករបស ។ ដយែនោះ ែននយបើកប យៅ យសមើនងែននតចែករបសយលខ ។ ឧទាហរណ យលខ៤ រតវសសេ ១យបើក សសេ ២ប សសេ ៤យបើក ដយែនោះែននយបើកប មាន៣ដង។ យលខ៦ រតវសសេ ១យបើក សសេ ២ប សសេ ៣យបើក សសេ ៦ប ដយែនោះែននយបើកប មាន៤ដង។

Page 23: Problem Book in Mathematical- Vol I Arithmetic

លម សវណណ វចតរ | 2.5. តរសតបរ 11

យ ើងដងយ ៀតថា ែននគតវជ ជមានចតមានែននតចែកជាែននយសសគមានចតែននកាយរចតបាយោណ ោះ។ ែននកាយរព ដល មាន ។ ដយែនោះ យលខ ១ យលខ៤ យលខ៩ នងយលខ១៦ រតវយបើកប ែននយសសដង ដយែនោះជាែងយរកា រតវយៅែហ។ ដយែនោះ ចដលែហមានែននបន៕៚

ឧទាហរណ: តាង ជាចននគរធជាង ។ ចរបងហា ញថា (a) ជាផលបកននចននគរបសសពររ

ជាបគនា ; (b) ជាផលបកននចននគរបរគនា ។

ែប ោះតា

(a) យ ើងមាន

ដយែនោះជាផលបកននែននយសស នង ។ ែននយសសពរយនោះជាែននយសសតជាបគនន ។

(b) យ ើងមាន

ដយែនោះជាផលបកននែននគតបតជាបគនន ៕៚

ឧទាហរណ: តាង ជាចននគ ។ បរើបគអាចសរបសរបលខ ជាផលបកននចតមាសរបសចនន

បសសចនន បផេងគនា បានចែរឬបរ។

ែប ោះតា

យ ើងសនមតថា យគអាែសរយសរបាន

ចដល ជាែននយសស។ កយនោមខាងយលើសមមលនង

ចដល ។ យ ើងមាន ជាែននយសស នង ស ធចតជាែននយសស។ យ ើងដងថា ផលបកននែននយសសែននគដង(យររោះ ជាែននគ) ឧទាហរណ 2ដង 4ដង 6ដង។ល។ ជាែននគ។ ដយែនោះ

Page 24: Problem Book in Mathematical- Vol I Arithmetic

12 ២ - ភាពចចកដាច | លម សវណណ វចតរ

អងគខាងយវវងននសមភាពជាែននគ យហើ អងគខាងសជាែននយសស ដយែនោះអងគទាងពរមនអាែយសមើគននបានយ ៕៚ ឧទាហរណ: ប បតជើសបរើសបលខចននតបាបចញពសណ ។ បបើប តបាបបៅនារ

អពផលគណននចននទាងតបា បនាោះនារមនអាចកណរបានថាផលបកននចននទាងតបាបនោះគឬបសសបាន

បរ។ បរើផលគណននចននទាងតបាចែលប បានបតជើសបរើសបសាើប នាា ន?

ែប ោះតា

យ ើងមាន ។ ដយែនោះ របាបពផលគណននែននគតរបាចដលយ ាបានយរជើសយរ ើស គដែជារបាបពផលគណននែននពរចដលយ ាមនបានយរជើសយរ ើសចដរ។ យបើសគ លផលគណននែននពរចដលយៅសល ធមមតា រអាែទាញបានែននទាងពរយ ោះជាអវ រែទាញបានែននទាងរបាចដលយ ាបានយរជើសយរ ើស ឱយ រអាែកណតបានថាផលបកននែននទាងរបាថាគឬយសសបាន។ ចតមានផលបកពរ ចដល រមនអាែកណតបាន យ ោះគ ចដលអាែជា ឬ នង ចដលអាែជា នង ។ ចតមនចមនជា យ យររោះយបើជា យ ោះ យ ើងមាន យសស នង យសស យហើ យដា គ យ ោះ រនងកណតបានថាផលបកននែននទាងរបាចដលយ ាបានយរជើសយរ ើសជាែននយសស។ ដយែនោះផលគណននែននពរចដលយ ាមនបានយរជើសយរ ើសរតវយសមើ ។ ដយែនោះផលគណននែននទាងរបាចដលយ ាបានយរជើសយរ ើសរតវចតយសមើនង

2.6. តរសតបរ- វធចចកចបបអគលែ

បបើ ជាចននគរវជជមាន បនាោះ បគមានចននគរ ម គរ ចែល

ឧទាហរណ៖ ែយរោះែននគត នង យ ើងមាន ចដល ជាសណល យហើ ។

Page 25: Problem Book in Mathematical- Vol I Arithmetic

លម សវណណ វចតរ | 2.5. តរសតបរ 13

សតមា បញជជ ក

ដបងយ ើងបងហា ញថា មាន ។ សយណើ ពត យដា យ ើង ក

នង

ចដល តាងឱយចផនកគតនន ; ឧទាហរណ ។ យ ើងមាន ។ យ ើងរតវបងហា ញថា ។ យ ើងដងថា 0 r b ។ ប ទ បមកយ ៀត យ ើងបងហា ញថា យគមាន ចតម គត។ យ ើងឧបមាថា យគអាែរកយ ើញ ចដល

ដយែនោះ ចែក ដាែ ចត ដយែនោះ មានចត ឱយ រែយហើ ។

គរកែសមគា លថា ចហ ោះចននគែ មយ តាមវធមចកមបបអគលែ ហគអាចមចកសណចននគែហៅតាមសណលរបសវធមចកនង ។ ឧទ រណ ហធៀបនង , កាងចហោមចននគែទងអស ហយើងអាចមចកជាចននមែលមគនរាង មែល ។ ឧទ រណ ។ ឧទ រណ ហធៀបនង , កាងចហោមចននគែទងអស ហយើងអាចមចកជាចននមែលមគនរាង មែល ។ ឧទ រណ ។ ឧទ រណ ហធៀបនង , កាងចហោមចននគែទងអស ហយើងអាចមចកជាចននមែលមគនរាង មែល ។ ឧទ រណ

ឧទាហរណ: តាង ជាចននគរវជជមាន។ ចរបងហា ញថា ចចកដាចនង ចរចចកមនដាចនង

ែប ោះតា

Page 26: Problem Book in Mathematical- Vol I Arithmetic

14 ២ - ភាពចចកដាច | លម សវណណ វចតរ

យ ើងមាន ជាែននយសស ដយែនោះ ជាែននគ ដយែនោះ ចែកដាែនង ។

យ ើងមាន យ ើងមាន

។ តាមរបមនតយ វធាញតន

ក នង នង យ ើងទាញបាន

តនម ៗកនងផលបកខាងយលើ យលើកចលងចតតខាងែង ស ធចតជាពហគណនន ដយែនោះតទាងយនោះចែកដាែនង ។ យ ើងទាញបាន ចែកនង សលសណល ។ ដយែនោះ

ចដល ជាែនន‹គត។ ដយែនោះ

ចែកនង សលសណល ៕៚

. ភាពសមមល

សរបសរថា អានថា សមមល តាម( ) ។

មានន ថា ចចកដាច រ ចែល ជាចននគរ។

ឬមានន មយ ងបរៀរថា នង មានសណលែចគនា បពលចចកជាម ។

ឧទាហរណ ឬ ។ ។ ដយែនោះ

. តរសតបរ

តាង ចែល នង បនាោះ

Page 27: Problem Book in Mathematical- Vol I Arithmetic

លម សវណណ វចតរ | 2.8. តរសតបរ 15

5° បបើ ជាពហធាមានបមគណជាចននគរ បនាោះ

សតមា បញជជ ក

យដា នង យ ោះ យគអាែរកបាន ចដល នង ។ ដយែនោះ

ដយែនោះយ ើងទាញបានលកខណៈយលខ 1 ដលយលខ 3 ។ ក ជនសែល កនងយលខ 3 យ ើងទាញបាន ។ យដើមបរស បញជជ កលកខណៈយលខ តាង

តាមលកខណៈយលខ 4 យ ើងទាញបាន

យដា ក តាមលកខណៈយលខ 3 យ ើងទាញបាន

យហើ តាមលកខណៈយលខ 1 យ ើងទាញបាន

ឧទាហរណ: ចរគណនា រនមលសមមលរបសចននកាបរ តាម 13 ។

ចបមលើ

សនរគែងបាន ។ យ ើងមាន ។

Page 28: Problem Book in Mathematical- Vol I Arithmetic

16 ២ - ភាពចចកដាច | លម សវណណ វចតរ

ដយែនោះ ែននកាយរ សមមលនង ។

. តរសតបរ-ភាពចចកដាចនង ៩

ចននគរធមាជារ ម ចចកដាចនង បបើផលបកននបលខតគបខាងទាងអសបញចលគនា

ចចកដាចនង ។

សតមា បញជជ ក

តាង ។ យដា

ដយែនោះយបើផលបកតយលខរបស (យសមើនង )ចែកដាែនង យ ោះ ចែកដាែនង ចដរ។

Page 29: Problem Book in Mathematical- Vol I Arithmetic

លម សវណណ វចររ | 3.1. រចចករមធបផរ 17

៣ - ការបបបកជាកតតា ផលគណ

3.1. រចចករមធបផរ

ឧទាហរណ គណនារចចករមនៃ ៃង ។

យ ើងមាន ដយចនេះតចចករបសវាមាន នង ដយចនេះតចចករបសវាមាន ។ ដយចនេះ នង មានតចចករម ។ កនងចយោមតចចករមទងយនេះ តចចករមចដលធជាងយគគ ។ យ ើងនយា ថា តចចករមធបផតរបស នង យសមើ ។

ៃយមៃយ

បបើ មៃសៃយទាង២រពមគនា បនាោះ ចៃៃគរធបផរ ចែលចចក ដាចទាងពរ បៅ

ថា រចចករមធបផរ របស ៃង ។

បគតាងបដាយ របដាយ ។

លកខណៈ

បបើ ៃង បនាោះ

ឧទាហរណ គណនា PGCD នៃ ៃង ។

យ ើងមាន

ឧទាហរណ គណនា PGCD នៃ ៃង ។

Page 30: Problem Book in Mathematical- Vol I Arithmetic

18 ៣ - ការបចបកជាកតាា ផលគណ | លម សវណណ វចររ

យ ើងមាន

យ ើងយឃើញថា

លកខណៈ

បបើ បនាោះបគៃយាយថា ៃង បឋមៃងគនា ។

ែបចាោះ បបើ ៃង បឋមៃងគនា បនាោះចៃៃ២បៃោះមៃអាចមាៃកតាា រមធជាង ១បេ។

3.2. ពហគណរមរចបផរ

ឧទាហរណ គណនាពហគណរមនៃ ៃង ។

យ ើងមាន ចននចដលជាពហគណនន ផងនងនន មាន

។ ចននទងយនេះយៅថា ចននពហគណរមនន នង ។ ចននចបបយនេះមានយរចើនរាបមនអស។ ចតចននចដលតចជាងយគ គ ។ ដយចនេះ ជាពហគណរមតចបផតរវាង នង ។

ៃយមៃយ

បបើ មៃសៃយទាង២រពមគនា បនាោះ ចៃៃគរវជជមាៃរចបផរចែលជាពហគណនៃ

ផងៃង ផង បៅថា ពហគណរមរចបផរ នៃ ៃង ។

បគតាងបដាយ រ ។

ឧទាហរណ គណនាពហគណរមរចបផរនៃ ៃង

យ ើងមាន

Page 31: Problem Book in Mathematical- Vol I Arithmetic

លម សវណណ វចររ | 3.3. រេសាបេ Bachet-Bézout 19

លកខណៈ

បយើងបឃើញថា បបើ ៃង បនាោះ ។

3.3. រេសាបេ Bachet-Bézout

រចចករមធបផររបសរគបចៃៃគរពរ តាងបដាយ អាចសរបសរជាបៃសលបៃច ែរនៃ

ៃង បាៃជាៃចច មាៃៃយថាមាៃចៃៃគរពរតាងបដាយ ចែល

សរមាយបញជជ ក

ឧទហរណ ។ យ ើងមាន ។ ឧទហរណ ។ យ ើងមាន ។ ឧទហរណ ។ យ ើងមាន ។

តាង ។ យ ើងដងថា មានម កនងចយោម ជាធាតរបស យររេះ មនអាចសនយទងពរ។ មានធាតតចបផត តាងយោ ។ ដយចនេះ យគមាន ចដល ។ យ ើងនងបងហា ញថា ។ ដយចនេះ យ ើងរតវបងហា ញថា ( ជាតចចករមរបស )នងថា យបើ នង យ េះ ( ជាតចចករមធបផតរបស )។

ជាដបងយ ើងនងបងហា ញថា ។ តាមរបមាណវធចចកចបបអគលដ យ ើងអាចរកបានចននគត ចដល យហើ ចដល

យបើ យ េះ ។ យោ យ េះ វាមានតនលតចជាងធាតតចជាងយគរបស ចដលតាងយោ ដយចនេះវាផទ ពការសនមត។ ដយចនេះ ។

Page 32: Problem Book in Mathematical- Vol I Arithmetic

20 ៣ - ការបចបកជាកតាា ផលគណ | លម សវណណ វចររ

ដចគនន យ ើងអាចបងហា ញថា ។

សនមតថា ចយរេះចននគត ។ ដយចនេះ

មានន ថា ។ ដយចនេះ ររសតបររតវបានរា បញជា ក។

សមាា ល

បយើងអាចចចងរេសដបេបៃោះតាមមយចបបបេៀរថា « រគបបៃសលបៃច ែរនៃ ចចកដាចៃង

។ »

3.4. ករ ចលរ គលែ

បបើ ចចកដាច ៃង បបើ បនាោះ ចចកដាច ។

សរមាយបញជជ ក

យោ យ េះ តាមររសតបរ Bachet-Bézout យគមានចននគត ចដល ។ យោ យ េះ យគមានចននគត ម ចដល ។ យ េះ

ដយចនេះ យ ើងទញបាន ។

3.5 រេសាបេ

1° បបើ បនាោះ ។

2° បបើ ជាចៃៃគរវជជមាៃ បនាោះ ។

3° ចប ោះចៃៃគរមៃសៃយ បគមាៃ ។

4° ចប ោះចៃៃគរមៃសៃយ បគមាៃ

Page 33: Problem Book in Mathematical- Vol I Arithmetic

លម សវណណ វចររ | 3.5 រេសាបេ 21

សរមាយបញជជ ក

1° តាមររសតបរ Bachet-Bézout យគមានចននគត ចដល ។ ដយចនេះ

។ យ ើងមាន នង ជាចននគត។ យ ើងដងថា ចចកោច

រគបបនសលយនចអែរនន នង ។ យោ មានបនសលយនចអែរម គ

ចដលយសមើ យ េះ

ចចក ោច។ យ ើងទញបាន ។

2° តាង នង ។ យ ើងបងហា ញថា នង ។ យ ើងមាន ដយចនេះតាមររសតបរ Bachet-Bézout យគអាចរកបានចននគត ចដល ។ តាមររសតបរ Bachet-Bézout ដចដល យគមានរគបបនសលយនចអែររវាង នង ចចកោចនង ។ ដយចនេះ ។ តាមររសតបរ Bachet-Bézout យគអាចរកបានចននគត ចដល

។ ចត ជាបនសលយនចអែរនន យហើ ដយចនេះ វាចចកោចនង ។ យគមានចននគត ចដល ។ ដយចនេះ មានន ថា ។

ចណ៖

ដចគនន យ ើងទញបាន ចយរេះរគបចននគតមនសនយ ។

3° យោ ចចកោច យ េះ វាចចកោច ។ ដយចនេះ ចចកោច នង យហើ ដយចនេះ

មយាងវញយរៀត ចចកោច នង យ េះ វាចចកោច នង ។ ដយចនេះ ចចកោច ។ ដយចនេះ ចចកោច នង យហើ ដយចនេះ វាចចកោច ។

4° សនមតថា ។ តាមររសតបរ 3° យ ើងទញបាន

Page 34: Problem Book in Mathematical- Vol I Arithmetic

22 ៣ - ការបចបកជាកតាា ផលគណ | លម សវណណ វចររ

យោ យ េះ

តាមររសតបរ3° ដចដល យ ើងទញបាន

ដយចនេះ ។ តាមររសតបរ 1°

ដយចនេះ

តាមររសតបរ 2° យោ ក យ ើងទញបាន

3.6 រេសាបេ

បបើ បនាោះ ចចកដាចយា ងបោចណស ៃងចៃៃបឋមមយ។

សរមាយបញជជ ក

យោ យ េះវាមានយាាងយោចោសតចចកម ធជាងម ។ ដយចនេះ វាមានតចចកម ចដលតចជាងយគ យហើ ធជាងម តាងយោ ។ យ ើងថា ជាចននបឋម។ យបើ មនចមនជាចនន

Page 35: Problem Book in Mathematical- Vol I Arithmetic

លម សវណណ វចររ | 3.7 រេសាបេ គលែ 23

បឋមយរ យ េះ យ ើងអាចសរយសរ ជា ។ ចតយបើដយចនេះ ជាតចចករបស ចដលធជាងម នងតចជាង ចដលផទ នងការសនមត ចដលថា តចជាងយគ។

3.7 រេសាបេ គលែ

សណចៃៃបឋម ជាសណ ៃៃា។

សរមាយបញជជ ក

សនមតថា សណចននបឋមជាសណកណត ចដលមានចននបឋមចត ។ យ ើងដងថា រគបចននគតទងអសចដលធជាងម សរធចតចចកោចនងចននបឋមយាាងយោចោសម (ររសតបរ 3.6)។ ដយចនេះ រគបចននគតទងអស រតវចតចចកោចនងយាាងយោចោសចននបឋមម កនងចយោម ។ យ ើងពនតយចននគតធជាងម ។ រតវចតមានចននបឋមម កនងចយោម ចដលចចក ោច។ ចតយបើមាន ម ចចក ោច យ េះ រតវចតចចក ោច យររេះ ចចកោចនង ។ ដយចនេះ ចត ជាចននបឋម ដយចនេះ ។ ដយចនេះមនអាច។ យ ើងទញបាន សណចននបឋមមនអាចជាសណកណតបានយរ៕៚

3.8 រេសាបេ

ផលគណនៃចៃៃគរពរចែលមាៃរាង កជាចៃៃចែលមាៃរាង ចែរ។

សរមាយបញជជ ក

យ ើងមាន

ដយចនេះសយណើ ពត៕៚

Page 36: Problem Book in Mathematical- Vol I Arithmetic

24 ៣ - ការបចបកជាកតាា ផលគណ | លម សវណណ វចររ

3.9 រេសាបេ

ចៃៃបឋមចែលមាៃរាង មាៃបរចើៃរាបមៃ ស។

សរមាយបញជជ ក

យ ើងនងរា បញជា កតាមវធផទ យោ សនមតថាចននបឋមមានរាង មានចននកណត តាងយោ ។ យ ើងពនតយចនន ។ តាង ជាតចចកបឋមម របស ។ អាចមានរាង ។ មនអាចមានរាង យរ យររេះមនចមនជាចននបឋម។ យបើ : យ េះ ជាចននបឋមម ចដលមានរាង ជាចននបឋមម កនងចយោម (យររេះសណចននបឋមចដលមានរាង មានចត ចតបាយោណ េះ)។ ចតកនងករណយនេះ យបើ យ េះ យររេះ សរធចតជាចននបឋម ចដលធជាង ។ ចតយបើ យ េះ ។ ដយចនេះ មនអាចជាតចចករបស បានយរ ផទ ពការសនមត។ ដយចនេះមានចត ។ ដយចនេះតចចករបស សរធចតមានរាង ។ ដយចនេះ យសមើនងផលគណននបោត យលខចដលមានរាង , ដយចនេះ រតវចតមានរាង ចដរ។ យ ើងទញបាន ចតមនអាចយររេះ ។ ដយចនេះ យ ើងទញបានថា ចននបឋមចដលមានរាង រតវចតមានចននយរចើនមនកណត៕៚

3.10 រេសាបេ

បបើចៃៃគរវជជមាៃ ជាចៃៃពហគណ បនាោះ វាមាៃកតាា បឋម មយ ចែល ។

សរមាយបញជជ ក

Page 37: Problem Book in Mathematical- Vol I Arithmetic

លម សវណណ វចររ | 3.11 រេសាបេ 25

សនមតថា ។ យបើ នង សរធចតធជាង ទង២ យ េះ

មនពត។ ដយចនេះ រតវមានកតាត ម ខសព នង ។ ដយចនេះវារតវមានកតាត បឋមម ចដល ៕៚

3.11 រេសាបេ

បបើ ជាចៃៃបឋម បនាោះ ចចកដាចៃង ចប ោះរគប ។

សរមាយបញជជ ក

យ ើងមាន

ដយចនេះ ឱយ ។ យោ យ េះ ចចកមនោច ។ តាមករ ាចលរអគលដ យ េះ មានចត

៕៚

3.12 រេសាបេ រគបចៃៃគរទាង ស សេធចរអាចបចបកជាផលគណ ចែលមាៃកតាា ទាង សជាចៃៃបឋម។

សរមាយបញជជ ក

ឧទហរណ ជាផលគណនន ចដលសរធចតជាចននបឋម ឬកតាាផលគណរបស សរធចតជាចននបឋម។ យ ើងនងបងហា ញថា រគបចននគតទងអសសរធចតអាចបចបកជាផលគណដចខាងយលើបានទងអស។ យ ើងពនតយចននគត ម ។ កករណ ជាចននបឋម យ េះ សយណើ ពត។

Page 38: Problem Book in Mathematical- Vol I Arithmetic

26 ៣ - ការបចបកជាកតាា ផលគណ | លម សវណណ វចររ

ខករណ មនចមនជាចននបឋម ដយចនេះយាាងយោចោស មានតចចកបឋម ម ចដល ចដល យ ើងមាន២ករណយរៀត ១ករណ ជាចននបឋម យ េះ ជាផលគណននចននបឋម២។ ២ករណ មនចមនជាចននបឋម។ មានតចចកបឋម ម យាាងតចចដល នង ចដល ។ តាមវចារដចគនន យ ើងទញបាន កតាត បឋមរបស ចដល ។ យោ ចននតចចករបស មានចននកណត យ េះ មាន ម ចដល ជាចននបឋម យហើ យសមើ ។ ដយចនេះ អាចសរយសរជា ចដល សរធចតជាចននបឋម (មានខលេះខសគនន ខលេះយសមើគនន )៕៚

យ ើងអាចយ ៀបកតតា បឋមទាងយ ោះជារាង

ដែល ជាបណតា ចាននបឋមខសៗគនា ។ យ ើងយៅកា បាដបកជាផលគណកតតា បឋមនន ខាងយលើ ថា

កតតា ផលគណរាងកាណនចនន ។ ឧទហ ណ ជា កតតា ផលគណរាងកាណនចនន

3.13 រេសាបេរគោះនៃៃពវៃា

រគបចៃៃគរធជាងមយ អាចបចបកជាកតាា ផលគណកាណៃចបាៃចរមយចបបគរ ។ ឬ

ៃយាយម ាងបេៀរថា ចប ោះរគបចៃៃគរ បគមាៃចៃៃបឋម ចែលខសគនា ពរៗ

ចរមយចបបគរ ៃងចៃៃគរវជជមាៃ ចរមយចបបគរ ចែល

Page 39: Problem Book in Mathematical- Vol I Arithmetic

លម សវណណ វចររ | 3.13 រេសាបេរគោះនៃៃពវៃា 27

សរមាយបញជជ ក

ឧបមាថាវាអាចបចបកបាន២យាាង

នង

ចដល ជាចននបឋមខសគនន ពរៗ នង ជាចននបឋមខសគនន ពរៗ។ យ ើង

មាន

យោ

។ ចត

សរធចតជាចននបឋម យ េះរតវមានម កនងចយោម

ចដលយសមើនង ។ សនមតថា ។ តាមរយបៀបដចគនន យ ើងទញបាន នង

។ ដយចនេះ អាចោកជាផលគណកតាត បឋមបានចតម ចបបគត៕៚

ររសាបរខាងយលើ ាឱយយ ើងទញបានរា ករានងសរាបគណ នង ននចាននគត២យបើសនជាយ ើងសគា លផលគណកតតា បឋម បសវា។ តតង

ដែលកាងយ ោះ បណតា ខសគនា ២ៗ ដតបណតា នង អាចានខលោះយសមើសនយ។ យ ើងាន

គ កតសាា លថា ចាយ ោះចាននពត នង យ ើងាន

ាឱយយ ើងទញបាន

Page 40: Problem Book in Mathematical- Vol I Arithmetic

28 ៣ - ការបចបកជាកតាា ផលគណ | លម សវណណ វចររ

3.14 របមាណវធចចកចបប គលែ

តាង ជាចៃៃគរវជជមាៃ។ បនាា បបធវើរបមាណវធចចកបៃាបនាា បគនា បយើងទាញបាៃបសែរនៃ

វសមភាពែចរបៅ៖

បសែរនៃសណលៃងបៅែល ចែលៃងបសមើសៃយ។ បរ ោះថា ជាសវែរចោះនៃប

ណា ចៃៃគរ ែបចាោះ សវែរបៃោះ មៃអាចមាៃចៃៃរវជជមាៃបលើសព បេ។

3.15 រេសាបេ

បបើ ជាចៃៃគរវជជមាៃ បនាោះ

សរមាយបញជជ ក

តាង នង ។ យោ នង យ េះ ។ ដយចនេះ ជាតចចករមរបស នង ។ ដយចនេះ ។ មយាងវញយរៀត នង យ េះ ។ ដយចនេះ ជាតចចករមនន នង មានន ថា ។ ដយចនេះ នង ឱយ ។

3.16 រេសាបេ

បបើ ជាសណលចងបរកាយ មៃសៃយ បៅកាងរបមាណវធចចក គលែ បនាោះ

Page 41: Problem Book in Mathematical- Vol I Arithmetic

លម សវណណ វចររ | 3.16 រេសាបេ 29

សរមាយបញជជ ក

យ ើងមាន

តាង ។ សមការរម ឱយ ។ រពរ ឱយ ។ បនតប ទ បមកយរៀត ។

ចតយោ យចញពសមការចងយរកា យហើ បកយៅយលើវញ យ ើងទញបាន

ដយចនេះ ជាតចចករមរបស នង ដយចនេះ ឬក ។

ដយចនេះ ៕៚

ឧទាហរណ: គណនា

ចបមលើយ

យ ើងមាន

សណលមនសនយគ ដយចនេះ ។

Page 42: Problem Book in Mathematical- Vol I Arithmetic

30 ៣ - ការបចបកជាកតាា ផលគណ | លម សវណណ វចររ

3.17 រេសាបេ

សៃមរថា ជាចៃៃគរចែល ។ បបើបគសគា លគចបមលើយ នៃសម

ការែយផង* បនាោះចបមលើយបផសងបេៀរនៃសមការបៃោះ ៃងមាៃរាង

ចែល ៃង ។

*សមការែយផងជាសមការចែលមាៃឫសឋរកាងសណចៃៃគរ។

សរមាយបញជជ ក

យ ើងថា យបើ ជាឫសរបសសមការ យ េះ

កជា

ឫសចដរ។ ចតយ ើងនងបងហា ញថា រគបចយមលើ ននសមការ នងមានរាងចបបយនេះទងអស។ សនមតថា យផទៀងផទទ ត ។ ដចគនន ចដរ ។ ដយចនេះយ ើងមាន

ចចកអងគទង២នង យ ើងទញបាន

យោ

យ េះ

យោ យយាងតាមករ ាចលរអគលដ។ ដយចនេះយគមានចនន

គត ចដល

ឱយ

។ យចញពយនេះ យ ើងទញបាន

Page 43: Problem Book in Mathematical- Vol I Arithmetic

លម សវណណ វចររ | 4.1 សមការសមមលលនេអ ែរ 31

៤ - ភាពសមមលកនងសណ

ចននគត

4.1 សមការសមមលលនេអ ែរ

សមការសមមលអញញា ត មយកណតដោយ ។ កដសោម មាសសយថាមាស ដែល ។ ែដ នេះ សមការសមមលអញញា ត ដែល មាសដមល ើយ ល េះតរាដតសមការែយផង មាសដមល ើយ សងតរាសមកវញ។ ែដ នេះ ដយើងដ ើញថា សមការសមមល មាសដមល ើយទាលដត ។

4.2 ររសតបរ

នេឱយចនេេេរ ។ នបើសមការសមមល មាេចនមលើយមយ

ន ោះ វាមាេចនមលើយមេសមមលគនា តាម ចនេេ ។

សរមាយបញជា ក

តាងចម លើយយម ោះមោយ ។ តាររសតបរ 3.17 មយើងទាញបាន សការដយផងលមនអ ែរ មានចម លើយមានរាង

Page 44: Problem Book in Mathematical- Vol I Arithmetic

32 ៤ - ភាពសមមលកាងសនណន ចនេេេរ | លម សវណណ វចររ

មយើងឱយ មានតម ល មយើងទាញបាន ចម លើយនសលគនន តា ចនន ។

មបើ

ជាចម លើយយមផេងមរៀត អដល ម ោះមយើងសរមសរ ជា

។ មយើងទាញបាន

មាននយថាឫស

សលមៅនងចម លើយយអដលមានរចកម ើយកនងចមោ

ដមចនោះ មបើសការមានចម លើយយ ម ោះ មានចម លើយនសលគនន តា ចនន ៕៚

ឧទាហរណ: ន ោះរាយសមការ

ចនមលើយ

តាររសតបរ 4.2 សការមនោះមានចម លើយនសលគនន តា អតយគត មររោះ ។ ជាដបងមយើងមោោះរាយសការដយផង ។ មយើងមាន

មយើងទាញបាន ។

ដមចនោះ ឬ ៕៚

ឧទាហរណ: ន ោះរាយសមការ 3 6 mod12x

ចនមលើយ

មោយ ម ើយ ម ោះ សការមានចម លើយចនន អដលនសលគនន តា ។ មយើងពនតយមឃើញថា ជាចម លើយយ។ តាររសតបរ3.17 រគបចម លើយទាង សមានរាង

។ មោយឱយ ម ោះចម លើយនសលគនន ទាង របសស

ការសលតា គ ៕៚

Page 45: Problem Book in Mathematical- Vol I Arithmetic

លម សវណណ វចររ | 4.3 ររសតបរ 33

4.3 ររសតបរ

នេឱយចនេេេរ េង ចនេេេរមេសេយ ន ោះ

លោះរតាអរ

េងរាសមកវញ។

សរមាយបញជា ក

មបើ ម ោះ ចមរោះចននគត ោយ។ ដមចនោះ

តាររសតបរ 3.5 មយើងទាញបាន

។ តាររសតបរ 3.4 មយើង

ទាញបាន

អចកោច ។

ដមចនោះ

រាសកវញ មបើ

មោយ អចកោច ម ោះ

4.4 ករ អលរ

នបើ េង ន ោះ ។

ឧទាហរណ : ចរេណ អែល េង

ចនមលើយ

Page 46: Problem Book in Mathematical- Vol I Arithmetic

34 ៤ - ភាពសមមលកាងសនណន ចនេេេរ | លម សវណណ វចររ

មោយ ដមចនោះ ។ មោយ ដមចនោះ ។ ដមចនោះ ។ ដមចនោះ មផទៀងផទទ តសការសលអដលឱយ៕៚

ឧទាហរណ : ចរេណ ចនេេេរ មយ អែលនពលអចកេង សលសនណល , នពលអចកេង

សលសនណល នហើយ នពលអចកេង សលសនណល ។

ចនមលើយ

មយើងចងបាន អដល

មយើងមាន

ដមចនោះ ៕៚

4.5 ររសតបររបបេ

នេឱយចនេេេរវជាមាេ ។ នបើ ន ោះ ចនេេនេចនេេេរវជាមាេ

អែលមេអាចសរនសរជារាង ចនន ោះចនេេេរមេ វជាមាេ បាេ

នសមើេង ។

សរមាយបញជា ក

មយើងសនមតមៅថា ជាចននអាចបអបកបានតា មបើចមរោះចននគត មគមានចននគតន វជ ជមាន អដល ។ មយើងពនតយតារាងអដលចននធាត នកណតខាងមរកា

Page 47: Problem Book in Mathematical- Vol I Arithmetic

លម សវណណ វចររ | 4.5 ររសតបររបបេ 35

មលខកនងជរឈរន យៗមនតារាងមនោះ ជាសវតនព វន តអដលមានផលសងរ មសមើ ។ ចននមៅពមរកាចននគត ោយកនងតារាងមនោះ គ អដល ជាចននគតធ មជាត។ មយើងដងថាមបើ អាចបអបកជា បាន ម ោះ កអាចបអបកបានអដរ មររោះ

។ ដមចនោះមយើងទាញបានថា មបើចននគត យអាចបអបកបាន ម ោះរគបចននគតអដលមៅពមរកាវាសរធអតអាចបអបកបានទាង ស។

រគបចននអដលជាព គណមនb សរធអតអាចបអបកបានទាង ស មររោះ ។ មយើងនយាយថា គនម នព គណមនb ពរខសគនន តាងមោយ នង អដល អាចឋតមៅកនងជរឈរអតយមនតារាងខាងមលើបានមរ។ មររោះថា មបើន ញចងមរ ម ោះមយើងនងមាន ។ មោយ មាននយថា នង បឋគនន នងតាររសតបរ 3.4 មយើងទាញបាន ។ មោយ ម ោះមយើងទាញបាន ។ ដមចនោះនអាចមានចននព គណមន ពរខសគនន ឋតកនងជរឈរអតយបានមរ។

ឥលវមយើងនងបងហា ញថា រគបចននទាង សកនងជរឈរអតយមនតារាង អដលមៅពមលើព គណមន តាងមោយ សរធអតជាចនននអាចបអបកបាន។ រគបចននអដលមៅកនងជរឈរអតយមលើ មានរាង ចមរោះចននគត យ។ មបើ អាចបអបកបានចមរោះ ម ោះមានចននគតន វជ ជមាន អដលមផទៀងផទទ ត ។ ដមចនោះ ។ ដមចនោះ ។ យាងវញមរៀត ចននពរឋតមៅមលើជរឈរអតយ សលគនន តា ។ ដមចនោះ

។ មោយ ម ោះ (តាករ ាអលរ 4.4) ផទយព ។

ដមចនោះ ចននមនចនននអាចបអបកបានតា មសមើនងចននមនចននអដលមៅពមលើចននអដលមានរាង ។ មៅមលើជរឈររ មគមានចននចនន ពមលើ ។ ដមចនោះចននមនចននអដលនអាចបអបកបានតា មសមើនង

Page 48: Problem Book in Mathematical- Vol I Arithmetic

36 ៤ - ភាពសមមលកាងសនណន ចនេេេរ | លម សវណណ វចររ

ចននគតអដលនអាចបអបកបានធបផតឋតមៅចពមលើ ដមចនោះ ចននគតអដលនអាចបអបកបានធបផត មានតម លមសមើនង ៕៚

4.6 ររសតបរ

នេឱយ ជាចនេេេរបឋមរវាងគនា ។ សមការ គនម េចនមលើយជាចនេេេរ

មេ វជាមាេ ចនន ោះ ។ នបើ ន ោះ សមការ

នេោះ មាេចនមលើយជាចនេេេរមេ វជាមាេ។

4.7 ររសតបរសនណលអចកចេ

នេឱយ ជាចនេេវជាមាេបឋមរវាងគនា ពរៗ នហើយសរធអរធនជាងមយ។

តាង ជាចនេេេរណាមយ ន ោះរបពេធសមការសមមល

មាេចនមលើយ នហើយអរមយេរតាម ។

សរមាយបញជា ក

តាង ។ តាង អដល ។ មយើងដងថា មាន អដលមផទៀងផទទ តលកខខណឌ ខាងមលើ មររោះរគប បឋរវាងគនន ពរៗ ឱយ បឋនង ។ មយើងបមងកើតចនន

មយើងមឃើញថាចននខាងមលើមផទៀងផទទ តរបពន ធសការ។

Page 49: Problem Book in Mathematical- Vol I Arithmetic

លម សវណណ វចររ | 4.8 ររសតបរវលសេ 37

មដើ បបងហា ញថា ចមលើយមានអតយ មយើងសនមតថា មាន មផេងមរៀត អដល ចមរោះរគប ។ ដមចនោះ ចមរោះរគប ។ មយើងទាញបានថា ន យៗអចកោច ។ មោយ បឋរវាងគនន ពរៗ ម ោះមយើងទាញបាន

ឬនយាយយាងមរៀត ៕៚

ឧទាហរណ : នរើនេអាចរកបាេមយលាេចនេេេររនរៀងគនា អែលចនេេទានង សន ោះ សរធអរអចក

ចេងចនេេកានរឬនរ?។

ចនមលើយ

តាង ជាចននគតទាងយលានបឋគនន ពរៗ។ តាររសតបរសណលអចកចន របពន ធសការខាងមរកាមានចម លើយ

បោត ចនន ជាយលានចននគតតមរៀងគនន អដលន យៗអចកោចនងកាមរមនចននបឋ៕៚

4.8 ររសតបរវលសេ

ចនន ោះរេបចនេេបឋម នេមាេ ។

សរមាយបញជា ក

មយើងដងថា មបើ បឋនង ម ោះមគមាន អដល ។ មររោះមបើ ម ោះតាររសតបរ Bachet-Bézout មគមានចននគត អដល ។ តាង ។ ដមចនោះ ។ មោយ នង ម ោះ ។ មាននយថា ចមរោះ យអដលបឋនង មគមាន យ (ម ើយមានអតយគត) អដល ។ ដមចនោះ ចមរោះ មគមាន អដល

Page 50: Problem Book in Mathematical- Vol I Arithmetic

38 ៤ - ភាពសមមលកាងសនណន ចនេេេរ | លម សវណណ វចររ

នង ។ មបើ ម ោះ

មោយ ម ោះ ។ ដមចនោះចមរោះ មផេងគនន ម ោះមយើងមាន មផេងគនន អដរ។ មបើ ឬ ម ោះររសតខាងមលើពត។ សនមតថា ។ ពនតយ អដល ។ ចមរោះ ន យៗខាងមដើ មនោះ មយើងកណត យមរៀតអដល ។ មបើ ម ោះ នអាចមររោះ ។ មបើ ម ោះ

។ អត ។ ដមចនោះ ។ ចមរោះ មផេងគនន មយើងមាន មផេងគនន អដរ។ មបើ ម ោះ

។ មោយ ជាចននបឋម ោះ ឬន ញចងមរ ។ អត ឬ ។ អតនអាចទាងពរមររោះ នង ។ ដមចនោះ ។ មយើងមាន នង ចមរោះ យមយើងអាចរកបាន យគត អដល ម ើយមបើ មផេងគនន មយើងររលបាន មផេងគនន អដរ។ ដមចនោះមយើងអាចបអបកសណ ជាពរគ អដលមផទៀងផទទ ត នង នង ។ សណ នង មានចននធាតមសមើនង

តាង

អដល នង

។ ដមចនោះ

ឧទាហរណ : នបើ ជាចនេេបឋម អែល ចរបងហា ញថា

Page 51: Problem Book in Mathematical- Vol I Arithmetic

លម សវណណ វចររ | 4.9 កេររសតបរអែមា 39

ចនមលើយ

មយើងសមងកតមឃើញថា

មយើងមាន

មោយ

ម ោះ

។ ដមចនោះ

តាររសតបរវលសន ដមចនោះ

4.9 កេររសតបរអែមា

( Fermat’s Little Theorem)

រេបចនេេបឋម េងរេបចនេេេរ នេមាេ ។

សរមាយបញជា ក

មយើងនងរាយបញជជ កតាវារមោយកមណើ នតា ។ មពល មយើងទាញបានថា សមណើ ពត។ សនមតថា អចកោច ។ មយើងមាន

Page 52: Problem Book in Mathematical- Vol I Arithmetic

40 ៤ - ភាពសមមលកាងសនណន ចនេេេរ | លម សវណណ វចររ

ចមរោះ មយើងមាន

មាននយថា អតមោយ ដមចនោះ

។ ចមរោះ កមយើងមាន

អដរ។ ដមចនោះ

ដមចនោះ អចកោចនង ។ ដមចនោះសមណើ ពត៕៚

ាមរយៈតររសតបរដសេះដយើងទាញបាសថា ។ ែដ នេះ ដបើ ដក មសោដរ ដ េះមាសសយថា ដក ោ។ ែដ នេះ ។

4.10 វបាក

តាង ជាចនេេបឋមេងសេមរថា អចក មេ ច ន ោះ ។

ឧទាហរណ : តាង ។ ចរេណ សនណលនេ អចកេង ។

ចនមលើយ

តាររសតបរភែមា មយើងមាន ។ មយើងមាន ចមរោះ

រគបចននគតវជ ជមាន មាននយថា ចមរោះចននគត ោយ។ ដមចនោះ

អចកនង សល ៕៚

Page 53: Problem Book in Mathematical- Vol I Arithmetic

លម សវណណ វចររ | 5.1 ផផែកគរ នង ផផែកទសភាគ 41

៥ - ផនែកគត

5.1 ផផែកគរ នង ផផែកទសភាគ

ផផែកគរនន តាងដោយ ជាចននគរធបផរផែលរចជាងឬដសមើ ។

ឧទាហរណ

ដយើងមាន ឬក

ដគដរបើសញញា សមាា ល តាងឱយផផែកទសភាគ។

ែដចែេះ

5.2 រទសតបទ

ដគឱយ ដ េះ

1)

2)

3)

សរមាយបញញា ក

1) តាង ។ ដចនេះ មាននយថា ។

2)

Page 54: Problem Book in Mathematical- Vol I Arithmetic

42 ៥ - ផផែកគរ | លម សវណណ វចររ

ចោយ

ជាននគត ច េះតាម 1) ចយើងទាញបាន

ចោយ ដចនេះ

3) តាមវសមភាព ចយើងទាញបាន

ចោយ ជាននគតត ជាងឬចសមើ ច េះវារតវតតត ជាងឬចសមើតផនកគតរបស ។ មាននយថា ។

ឧទាហរណ : ចរបងហា ញថា ចដ េះរគប វសមភាពខាងដរោមពរ

ែដ េះរាយ

ចយើងមាន នង ។ ដចនេះ

ដចនេះចយើងរតវបងហា ញថា

Page 55: Problem Book in Mathematical- Vol I Arithmetic

លម សវណណ វចររ | 5.3 រទសតបទ 43

ចោយសារវសមភាពគមម នលកខណៈពចសសរវាង នង ច េះចយើងអាសនមតថា ។ ចយើងដងថា ដចនេះ

ពត៕៚

5.3 រទសតបទ

ដបើ ជាចននគរធមមជារបឋមរវាងគនែ ចរបងហា ញថា

សរមាយបញញា ក

ពនតយតចោណតកងមយ តដលមានកពលរតង ។ តចោណតកងចនេះ មាន ណតដលមានកអរចោចនជាននគត។ តចោណតកងចនេះតកជាពរតណកចសមើគមន ចោយប ា ត

ចយើងនយាយថា គមម នណណាមយតដលឋតចៅចលើប ា តចនេះ មានកអរចោចនជាននគតចេ ចលើកតលងតតណងទាងពរ។ ចររេះថាចបើសនជាមានណមានកអរចោចនគតឋតចៅចលើប ា តចនេះ រតង តដល ច េះ

។ ច េះ មាននយថារបភាគ

អាបរងម

ចៅជា

ផាយពសមមតកមមតដល បឋមរវាងគមន ។

Page 56: Problem Book in Mathematical- Vol I Arithmetic

44 ៥ - ផផែកគរ | លម សវណណ វចររ

ជាបណាា ណតដលឋតចៅចលើប ា តចនេះ។ ចសមើនង

ននណមានកអរចោចនគត តដលឋតចៅចលើប ា តឈរ ចញព ចៅ

។ដចនេះ

ជាននណមានកអរចោចនគត តដលឋតចៅកនងរកកណាា លតផនកខាងចរោមរបស

តចោណតកង។ ដ គមន តដរ ជាននណមានកអរចោចនគត តដលឋតចៅកនងរកក

ណាា លតផនកខាងចលើរបសតចោណតកង។ ចោយជាសរបមាន ណមានកអរចោចនគត ច ើយតកចសមើគមន តផនកខាងចលើនងខាងចរោមតចោណតកង សចណើ ខាងចលើពត៕៚

5.4 រទសតបទឌប លញ ក

(De Polignac)

តាង ជាចននបឋម។ ចននគរធបផរ ផែល ផចកោច កណរដោយ

សរមាយបញញា ក

កនងចណាមននគតព ដល , ននគតតដលតកោនង មាននន , ននគតតដលតកោនង មាននន ,.....។ ដចនេះ តកោនង សវយគណ

ឧទា រណ ។ ព ដល ននតកោនង មាននន គ ។ ននតកោនង មាននន គ ។ ដចនេះ តកោនង ៕៚

ឧទាហរណ : ដរើមានដលខសនយចននប ម នននខាងចង ែដ េះរាយ

ចលខសនយខាងង ចសមើនង ននតដលជាសវយគណនន ធបផតតដលតកោ មាននយថា ធបផតតដលតកោ ។ ចយើងមនអាយករបមនា ឌប លញ ក មកចរបើចោយផទា លបានចេ ចររេះ មនតមនជាននបឋម។ តត ។ ដចនេះ ធបផត តដល តកោ

Page 57: Problem Book in Mathematical- Vol I Arithmetic

លម សវណណ វចររ | 5.4 រទសតបទឌប លញ ក 45

ចសមើនង តនមលត ជាងចគនន កនងចណាម ( ធបផត តដល តកោ ) នង ( ធបផត តដល តកោ )។ តាមរបមនា ឌប លញ ក ធបផត តដល តកោ ចសមើនង ធបផត តដល តកោ ចសមើនង

ដចនេះ មានចលខសនយចៅខាងងនន ខាង៕៚

Page 58: Problem Book in Mathematical- Vol I Arithmetic

46 ៥ - ផផែកគរ | លម សវណណ វចររ

Page 59: Problem Book in Mathematical- Vol I Arithmetic

លម សវណណ វចររ | 6.1 អនគមននពវនត 47

៦ - អនគមននពវនត

6.1 អនគមននពវនត

អនគមននពវនតជាអនគមន ដែលមានដែនកណរជាសណរងននចននគរវជជមាន។ ខាងករោម

កនេះជាអនគមននពវនតសខានៗកនងនពវនត

ជាចននរដចកវជជមានរបស

ជាផលបកននរដចកវជជមានទងអសរបស

ជាផលបកសវយគណ ននរដចកវជជមានទងអសរបស

ជាផលគណរដចកវជជមានទងអសរបស

ជាចននននចននគរវជជមានដែលធមនកលើសព នងបឋមនង ។ កគកៅ ថា

អនគមនអដល

ជាចននរដចកបឋមខសៗគនន របស

ជាចននរដចកបឋមរបស n រាបមនបាចគរថាខសគនន ឬអរកេ(កសមើនងផលបកននសវ

យគណរបសកតតត បឋមរបស ឧទហរណ កយើងមាន ចននពរែង ចននមដង

ែកចនេះ )។

អនគមនខាងកលើអាចតតងជាសញញា កោយ

Page 60: Problem Book in Mathematical- Vol I Arithmetic

48 ៦ - អនគមននពវនត | លម សវណណ វចររ

(សញញា កៅកនង មាននយថា ដរ )

ឧទាហរណ ជាតចែករបស ។ យ ើងមាន

។ យោ ជាែននគតវជជមានធមនយ ើសព នងបឋមនង យ ោះ ។

មានន ថា ែយ ោះ ចរបរប ព ដ យបើមានែននគត ណាម ចែក ោែ យ ើងបយងក ើនផ បកម ឯកតា។ ឧទាហរណ យ ើងចរបរប

ដយែនោះ មានន ថា មានតចែកែននបនគ ។

6.2 រេសតបេ

កបើផលគណកតតត បឋមរបស មានរាង

ក េះ កយើងមាន

សរមាយបញញជ ក

ដោយ

ដ ោះ តចចករបស មានរាង

ចដល ។ ដោយយកតមលល ចរបរបលព ដល ដយើងផសបានតចចករបស មានចនន

Page 61: Problem Book in Mathematical- Vol I Arithmetic

លម សវណណ វចររ | 6.2 រេសតបេ 49

តចចករបស មានដចជា

ដយើងមាន

កនងរបមាណវធខាងដលើ ដបងដយើងទបតមលល ឱយដៅដថរសន ប ា បលកដយើងដធវ ើរបមាណវធបកតាល ដោយឱយ ចរបរបលព ដល ។ ដយើងដធវ ើដចគនន ចដ ោះសវយគណដផសងដទៀត។ តចចកវជ ជមានលយរបស មានរាង

ចដល ។ ផលគណរបសបណតា តចចកទងអសរបស មានរាង

។ ដយើងនងគណ ។ ដយើងយកចននគត លយចដល ។ តចចករបស ចដល មានទងអសចនន ។ ដពលដយើងគណតចចកទងអសដនោះ បញចលគនន ដយើងបាន

Page 62: Problem Book in Mathematical- Vol I Arithmetic

50 ៦ - អនគមននពវនត | លម សវណណ វចររ

តាលរដបៀបដចគនន ដយើងទញបាន ។ ដដចនោះ

ឧទហរណ : ចរបងហា ញថា ។

ចកមលើយ

រគបតចចកវជ ជមាន នលយៗមន ដយើងចតងចតអាចរកបានតចចកលយដទៀតរបស ចដលដសមើនង ។ ដោយ ដ ោះ រតវចតមានលយកនងចដណតល នង ចដល ដរ ោះដបើ នង ដ ោះ លនពត។ តាង ជាតចចកវជ ជមានរបស ចដលធលនដលើសព ។ តចចករបស ដផសងដទៀតគ

ដោយ ដ ោះ ។ ដយើងមាន ៕៚

ឧទហរណ : ចរគណ រគបចននគរ ដែល ។

ចកមលើយ

ដយើងមាន ។ ដបើ មានកតាា បឋលដផសងគនន ដរចើនដលើសពពរ ដ ោះ ។ ដដចនោះ nរតវចតមានកតាា បឋលខសគនន ចត២ប ដណតណ ោះ គ នង ។ដដចនោះ នង

ដដចនោះ រតវចតមានរាង ឬក ចដល ជាចននបឋលខសគនន ៕៚

ឧទហរណ : ចរបងហា ញថា

Page 63: Problem Book in Mathematical- Vol I Arithmetic

លម សវណណ វចររ | 6.2 រេសតបេ 51

ចកមលើយ

ដយើងមាន

ចដល ដបើ នង ដបើ ចចក លនោច។

ដសមើនងចននចននគតចដលមានរាង ។ ដយើងមាន

ដរ ោះ ជាផលចចកគត ចដលបានលកដោយចចកចននគត នង នង

ចននចននគតព ដល ចដលមានរាង ដចជា ។ ដបើដយើងពនតយដលើរដបៀបចចកសនលកដបៀចនន សនលកដលលនសស ក មាននយថា ចចកគនន លដងលយសនលកៗរហតដលរគបគនន រចចចកលដងលយសនលកដទៀត ចចករហតដលសលដបៀចចកចលងដកើត ដយើងទញបានផលចចកនងចននចននគតចដលមានរាង ដសមើគនន ។ ឧទហរណ ចចកដបៀ៩សនលកដលលនសស៤ ក។ ចចកលយសាអសដបៀចនន សនលក បានមាន កលយសនលក ដៅសលដបៀ សនលក។ ចចកលយសារដទៀតអសដបៀសរបគតទងដលើកលនចនន បានមាន កលយសនលកដទៀត ដហើយសលដបៀលយសនលក។ ដដចនោះជាសរបចចកបានពរសា (ដសមើចននចននចដលមានរាង ) បានមាន កពរសនលកចដរ (ផលចចក)។ ដដចនោះ ពត។ ដយើងទញបាន

Page 64: Problem Book in Mathematical- Vol I Arithmetic

52 ៦ - អនគមននពវនត | លម សវណណ វចររ

ឧទហរណ : ចរបងហា ញថា ចក េះរគបចននគរធមមជារ កគមានចននគរធមមជារ នង ដែល

នង ។

ចកមលើយ

តាង ។ ដយើងយក ។ ដយើងមាន

(សលអានដសចកដពនយលកនងរទសាបទ 6.2) ។ ដដចនោះ មាននយថា មាន ចដលដផាៀងផទា តលកខខណឌ ៕៚

6.3 រេសតបេអដល

កបើ ក េះ ។

សរមាយបញញជ ក

ដយើងមាន ។ តាង ជាបណតា ចននគតវជ ជមាន តចជាង ដហើយបឋលនង ចដល ។ រគបចននគតទងអសចដលបឋលនង ដពលចចកនង សលសណលជា លយកនងចដណតល [*]។ ដោយ ដ ោះ កបឋលនង ចដរ។ ដដចនោះ

ដហើយ ដបើ ដ ោះ នង ចដល ដរ ោះដបើមាន នង ដ ោះ

ដរ ោះ ។ ដដចនោះ

Page 65: Problem Book in Mathematical- Vol I Arithmetic

លម សវណណ វចររ | 6.4 រេសតបេ 53

ដោយ ដ ោះ ។

សេចកដពនយលបននែម

តរងចណច [*] ដែលនយាយថា «តរបចននររទងអសដែលបឋមនង ពេលដចកនង សល

សណលជា មយកនងចពោម »

ឧទហរណ ចព ោះ ពយើងមាន ជា

ចននបឋមនង ។ ែពចនោះ ។ ពយើងមាន បឋមនង ពហើយ

។ ពយើងមាន បឋមនង ពហើយ

។ ពយើងអាចេនយលលកខណៈពនោះជាទពៅពោយ តាងចននររមយពោយ

។ ពបើ នង បឋមគនន ព ោះ តរវដរបឋមនង ដែរ។ ពត ោះថា ពបើមាន ជារដចករមរវាង

នង ព ោះ កជារដចករបស ដែរ។ ែពចនោះ ឱយ នង មនបឋមគនន ។ ែពចនោះ តរវដរបឋម

នង ។ ពោយ បឋមនង ពហើយ ព ោះ តរវដរជាចននមយកនងចពោម ។

6.4 រេសតបេ

តតង ជាចននគរវជជមានរចបផរដែល ។ ចននគរវជជមាន

ដែល បាន កបើ ជាពហគណនន នងរាសមកវញ។

សរមាយបញញជ ក

ដបើ ជាពហគណមន ដ ោះ ។ ដោយ ដ ោះ ពត។

រាសលកវញ សនមតថា ។ យក ។ ដដចនោះ ។ ដយើងមាន

Page 66: Problem Book in Mathematical- Vol I Arithmetic

54 ៦ - អនគមននពវនត | លម សវណណ វចររ

ដបើ ដ ោះ មាន ចដល ។ ដដចនោះផាយពសលមតកលមចដលថា តចជាងដគ។ ដដចនោះ ។ ដដចនោះ ជាពហគណមន ៕៚

6.5 រេសតបេ

តតង នង ជាចននគរវជជមានពរដែលបឋមគនន ក េះ ។

សរមាយបញញជ ក

តាង ជាចននគតធលមជាតលយចដល ។ ដយើងតដរៀបចននគតចនន គ ដចខាងដរោល

ចននគត លយ បឋលនង ដបើ វាបឋលនង នងនង នងរាសលកវញ។ ជាដបងដយើងនងកណតចននមនចននគតកនងតារាងខាងដលើ ចដលបឋលនង នងប ា បលកគណ ឱយដឃើញថា ដតើមានចននប ម នកនងចដណតលដ ោះចដលបឋលនង ។ មានចននគតចនន ដៅកនងប ា តដដកទ១ចដលបឋលនង ។ ឥលវដយើងដលើលប ា តឈរទ ចដល ។ ចននគតនលយៗឋតដៅកនងជរឈរលយដនោះ មានរាង

។ ដោយ ដ ោះ មានតចចកលយជាលយ ដបើ មានតចចករលលយជាលយ ចដរនងរាសលកវញ។ ដដចនោះ ដបើ បឋលនង ដ ោះរគបចននគតដផសងដទៀតឋតដៅកនងជរឈរជាលយ កបឋលនង ចដរ។ ដដចនោះមានចននគតចនន ជរឈរចដលបឋលនង ។ ប ា បលកដយើងកនតថាដតើមានចននគតចននប ម នកនងចដណតលដនោះ ចដលបឋលនង ។ ដយើងនយាយថា លនអាចមានចននគតពរកនងចដណតល ដៅដលើប ា តឈរទ សលលលគនន តាល ដទ។ ដបើលនអញចងដទ ដបើ ដ ោះ

Page 67: Problem Book in Mathematical- Vol I Arithmetic

លម សវណណ វចររ | 6.6 រេសតបេ 55

។ ដោយ ដ ោះ ។ ដោយ ដ ោះ ។ ដដចនោះដយើងទញបានថា បណតា ចននគតឋតដៅកនងជរឈរចតលយ កនងចដ លជរឈរនលយៗមនជរឈរទងអសចនន សលលលដៅនង ។ ចតមានចតចននគតចនន កនងចដ លដនោះប ដណតណ ោះចដលបឋលនង ។ មាននយថា មានចននគតចត ប ដណតណ ោះ ដៅកនងតារាងដនោះ ចដលបឋលនង ៕៚

េមគា ល

- យបើ ជាែននបឋម នង ជាែននគតធមមជាត យ ោះែននគត ជាែននគតវជជមានចតម ចបបគតចដ នងមានកតាា រមជាម ។ ដយែនោះ ។ យបើ

ជាផ គណកតាា បឋមរបស យ ោះ

6.6 រេសតបេ

ចក េះចននគរវជជមាន កគមាន ។

សរមាយបញញជ ក

ដយើងពនតយសណមនចននសនទន

ដយើងដងថា សណដនោះមានធាតទងអសចនន ។ ដរោយសរលលរបភាគដនោះដហើយ ដយើងនងទទលបានសណថម ចដលភាគចបងនងភាគយក បឋលនងគនន ។ បណតា ភាគចបងកនងសណថមសទធចតជាតចចករបស ។ តាង ជាភាគចបងលយ។ ដ ោះ កនងសណថម មានធាតចនន ចដលមាន ជាភាគចបង។ ដដចនោះ សណថមមានធាតទងអសចនន ។ ដោយសណទងពរមានចននតដសមើគនន ដ ោះ ដយើងទញបានសដណើ ពត៕៚

Page 68: Problem Book in Mathematical- Vol I Arithmetic

56 ៦ - អនគមននពវនត | លម សវណណ វចររ

ឧទហរណ : តតង ជាចននគរវជជមាន។

(1) គណ ផលបកននរគបចននគរវជជមាន រចជាង កហើយបឋមនង ។

(2) គណ ផលបកននរគបចននគរវជជមាន រចជាង កហើយបឋមនង ។

ចកមលើយ

តាង

តាង ជាបណតា ចននគតវជ ជមានតចជាង ដហើយបឋលនង ។ ដយើងមាន ទលចត ។ ដយើងមាន ធជាងដគ ដហើយបឋលនង ដដចនោះ បឋលនង ចដរ ចតតចជាងដគ ដដចនោះ រតវចតជា ។ ដយើងទញបាន

ដដចនោះ

លយ ងវញដទៀត ដយើងមាន

ដដចនោះ

ឧទហរណ : ចរឱយឧទហរណចននគរ ដែល ។

ចកមលើយ

Page 69: Problem Book in Mathematical- Vol I Arithmetic

លម សវណណ វចររ | 6.7 រេសតបេឡសង 57

យក ដ ោះ ៕៚

ឧទហរណ : កបើ នង ជាចននបឋម នង ចរបងហា ញថា ។

ចកមលើយ

សដងេតដឃើញថា ដបើ ដ ោះ អាចមានរាង ។ មានចតករណ លយចតប ដណតណ ោះ ចដល នង អាចជាចននបឋលទងពរបាន។ ដដចនោះ

6.7 រេសតបេឡសង

តតង ជាចននបឋម នងតតង

ជាព ល ររបស កនងកគនល ។ ចននគរ ធបផរដែល ដចកោច កណរ

កោយ

សរមាយបញញជ ក

តាលរទសាបទឌបា លញា ក

ដយើងមាន

Page 70: Problem Book in Mathematical- Vol I Arithmetic

58 ៦ - អនគមននពវនត | លម សវណណ វចររ

ដដចនោះ

6.8 រេសតបេខាមរ

(Kummer)

តតង ជាចននបឋម។ ចននគរ ធបផរដែល ដចកោចកមគណកេវធា

កសមើនងផលបកចននរតតេកកនងរបមាណវធបករបស នង សរកសរកនងកគនល ។

សរមាយបញញជ ក

តាង

នង ។ តាង

យក ចដល នង កណតដោយ

Page 71: Problem Book in Mathematical- Vol I Arithmetic

លម សវណណ វចររ | 6.9 រេសតបេ 59

ផលបកមនចននរតាទកកណតដោយ ។ គណសលភាពទងអសដនោះបនាប ា បគនន នង រចបកអងគនងអងគ ដយើងទញបាន

ដោយបកសលភាព(*)បញចលគនន ដយើងទញបាន

តាលរទសាបទឡសង ចននគត ធបផតចដល ចចកោច

កណតដោយ

6.9 រេសតបេ

រគបចននគរធមមជារ សេធដរអាចសរកសរជារាង

បានទងអស ដែល ។

Page 72: Problem Book in Mathematical- Vol I Arithmetic

60 ៦ - អនគមននពវនត | លម សវណណ វចររ

ឧទហរណ

Page 73: Problem Book in Mathematical- Vol I Arithmetic

លម សវណណ វចតរ| ឡសច 61

ឡស ច

តាង ជាចននគតវជជមាន។ ចរបងហា ញថា រគបអងកត មានចននសវយគណគោល២ ជានចច។ (ចននសវយគណគោល២ ជាចននដដលមានរាង )

តាង ជាសណមនទគទ ននចននគតវជជមាន ដដល " គបើ ជាធាតរបស គ ោះ នង កជាធាតរបស ដដរ "។ ចរបងហា ញថា ជាសណននរគបចននគតធមមជាត។ កនងគនោះ តាងឱយដផនកគតនន ។

ចរបងហា ញថា កនងចគោមបោា ចននគតវជជមានដដលធមនគលើសព គគអាចរកបានចននពរកនងចគោមចននទងគនោះ កណតគោយ នង ដដល ។ គគឱយសណ មយមានធាតចនន គផេងោន ។ ធាតទង គនោះជាចននគតធមមជាត ដដលគគគរជើសគរ ើសគោយគសរពកនងចគោមចននគតចាបព ដល ។ ចរបងហា ញថា គគអាចរកបាន សណរងមនទគទខសោន ពរនន ដដលមានផលបកធាតរបសវាគសមើោន ។ គគគរជើសគរ ើស ចននគត គោយគសរគចញព ។ ចរបងហា ញថា មានចននគតពរកនងចគោមចននទង ដដលមានផលសងគសមើ ។ (អាមេរច ១៩៩៤) គគបទសលល កគលខថាសចននមយ គោយគលខ១, ថាសចនន គោយគលខ២, ថាសចនន គោយគលខ៣,…, ថាសចនន គោយគលខ៥០។ គគោកថាសដដលបទសលល កទង

គនោះ គៅកនងរបអបមយ។ គគទញយកថាសមាងមយៗ គចញពរបអបគនោះ គោយនចដន គោយមនោកចលវញ។ គតើថាសតចបផតចននប ម ន ដដលរតវទញគចញ មកគដើមបឱយរបាកដថា មានយ ងតចថាសចនន មានសលល កដចោន ?។

Page 74: Problem Book in Mathematical- Vol I Arithmetic

62 ឡសច |លម សវណណ វចតរ

(អនតរជាត ១៩៦៤) មនសេ ក ទកទងោន តាមសបរត គោយមនសេមាន កៗ សរគសរគៅកានមនសេ កគទៀតដដលគៅសល។ពកគគ ទកទងោន គលើរបធានបទចនន ដតប គោណ ោះ។ រាលបោា មនសេពរ ក សរគសរគៅោន គៅវញគៅមក គលើរបធានបទដតមយ ប គោណ ោះ។ ចរបងហា ញថា មានយ ងតចមនសេចនន៣ ក ដដលសរគសរសបរតគៅកានអនកដនទពរបធានបទដតមយដចោន ។

សខ នង គៅគលងដលបងដចតគៅ។ ពកវាចាបគផា ើមគោយគរជើសគរ ើសចននគត មយ ។ ប ា បមកគទៀត សខគរជើសគរ ើសយកចននគត មយជាសមាា ត ដដល ។ គៅជាអនកទយគលខសមាា តគនោះ។ គដើមបឱយគៅអាចទយបាន គៅរបាបគលខ មយគៅសខ គ ើយសខគឆល ើយមកវញថា ជាចននបឋមឬអត។ ចរបងហា ញថា គៅអាចកណតគលខសមាា តរបសសខបានគោយរបាបសខយ ងគរចើន ដង។ ចរបងហា ញថា រគបឆន ទងអស មាននថាសរកទ១៣ យ ងគោចោសមាង។ គគគរជើសគរ ើសយក ចននគតគចញព ។ ចរបងហា ញថា គបើគទោះជាគគគរជើសគរ ើសយ ងគម ចកគោយ កគគគងទទលបានចននពរដដលបឋមរវាងោន ដដរ។ (អាមេរច AHSME 1999)

ទោកអវា នគៅផារមយរតវបានគគបទសលល កគលខចាបពគលខ១គៅ។ តគលខបាល សាចដដលគរបើសរមាបគធវ ើជាសលល កគលខបទគលើទទងគ ោះ មានតនមលពរគសនកនងមយគលខ។ ដគចនោះគដើមបបទគលខ៩ គគរតវចោយអស ពរគសន គ ើយគដើមបបទគលខ១០ គគរតវចោយអសបនគសន។ គបើសនជាគគរតវចោយអស គដើមបបទសលល កគលខ គតើមានទទងអសចននប ម ន?។

(អាមេរច AMC12 2001)

គតើមានចននគតវជជមានចននប ម នដដលមនគលើសព គ ើយជាព គណនន ឬ ដតមនដមនព គណនន ។

Page 75: Problem Book in Mathematical- Vol I Arithmetic

លម សវណណ វចតរ| ឡសច 63

(អាមេរច AHSME 1983)

តាង

ដដលគលខកនងខាងនមយៗបានមកពចោះគលខគតព ដល តាមលោប។ ចរកណតខាងទ គរកាយគកបៀស។ គកមងរបស២៥ កនងគកមងសស២៥ កអងគយជវញតមលមយ។ ចរបងហា ញថា គទោះជាអងគយ

ដបបោ កគងមានគកមងមាន កដដលអនកគៅសងហា ងវាជាគកមងសសទងពរ កដដរ។

(អាមេរច AMC12 2001)

ពងពាងមានគរសលមគជើងមយនងដសបកគជើងមយសរមាបគជើងនមយៗ ននគជើងទងរបាបរបសវា។ គតើពងពាងអាចពាកគរសលមគជើងនងដសបកគជើងទលដតរគបគជើង តាមលោបលគោយបានប ម នរគបៀបខសៗោន គបើដងថា រាលគជើងនមយៗរបសវា វារតវពាកគរសលមគជើងមនដសបកគជើង?

(អាមេរច AHSME 1986) ថតតឋតកនងបនាបងងតមយមានគរសលមគជើងរក ម គ គរសលមគជើងនបតង គ គរសលមគជើងគខៀវ គ នងគរសលមគជើងគមម គ។ យវជនមាន កយកគរសលមគជើងមយមាងគចញពថតតគនោះ ដតវាមនអាចសលគ លពណរបសគរសលមគជើងទងគ ោះបានគទ។ គតើយវជនគ ោះរតវយកគរសលមគជើងគចញយ ងតចចននប ម នគដើមបឱយវាទទលបានគរសលមគជើងយ ងគោចោស គ? គោយដងថា គរសលមគជើងមយគ ជាគរសលមគជើងពរដដលមានពណដចោន ។ គរសលមគជើងមយមនអាចឱយរាបគៅកនងគគរសលមគជើងគលើសពមយបានគទ។)

(អាមេរច ១៩៩១) គគឱយចននសនទនមយ គគសរគសរវាជាទរមងបរងមរច ប ា បមកគគគណ ផលគណននភាគយកនងភាគដបង។ គតើមានចននសនទនចននប ម នដដលឋតគៅចគ ល ោះ នង ដដលមានផលគណគសមើ ។

Page 76: Problem Book in Mathematical- Vol I Arithmetic

64 ឡសច |លម សវណណ វចតរ

(អាមេរច ១៩៩៨)

ចរកណតចននចតធាត ននចននគតវជជមានគសស ដដល

(អាមេរច AIME 1992)

គតើមានគននចននគតបនាប ា បោន ចននប ម ន ដដលឋតគៅកនងសណ

គ ើយដដលមនរតវការរតាទក គពលគធវ ើរបមាណវធបកននចននគតទងពរ?

(អាមេរច AHSME 1994)

គៅអរបាបនកនងជរមយ សរាបទកគអាយសសេ៦ ក នងសលសតសលា ចារយអាលផា ដបតា នងកាមា អងគយ។ សលសតសលា ចារយទងបមកដលមនសសេទង៦ ក គ ើយសគរចចតាគរជើសគរ ើសគៅអ យ ងោគអាយសលសតសលា ចារយមាន កៗសថតគៅចគ ល ោះសសេ២ ក។ គតើសលសតសលា ចារយអាលផា ដបតា នងកាមា អាចគរជើសគរ ើសគៅអខលនបានប ម នរគបៀប? ចរបងហា ញថាកនងចគោមចននគតវជជមាន១៦គផេងោន មានតនមលមនគលើសព១០០ មានចនន

គត៤គផេងោន ដដល ។

(អាមេរច ១៩៨៩)

កមារមាន កមានឥដឋមយសណមាន៩៦ដគផេងៗោន ។ ដនមយៗអាចគធវ ើពបាល សាចឬគឈើ អាចមានទ តច មធយមឬធ អាចមានពណគខៀវ នបតង រក ម ឬគលឿង នងអាចមានរជងជារងវង ឆគកាណ កាគរ ឬរតគកាណ។ គតើមានដឥដឋប ម នដ ដដលមានលកាណៈខសពឥដឋបាល សាច ទ មធយមពណរក មរាងរងវង រតងពរចណចគត? (ឥដឋគឈើ ទ មធយមពណរក មរាងកាគរ ជាឧទ រណមយ)។

(អាមេរច ១៩៩៨)

គគគៅគលខទរសពា៧ខាង

ថាជាគលខពគសស គបើលោបលគោយ

ដចោន នង ឬ

(ដចទងពរកបាន)។

Page 77: Problem Book in Mathematical- Vol I Arithmetic

លម សវណណ វចតរ| ឡសច 65

ឧទ រណ សទធដតជាគលខពគសស។ សនមតថា នមយៗអាចមាន តនមលជាគលខ ។ ចរកណតចននគលខពគសសទងអសដដលអាចមាន។

(អាមេរច ១៩៩៦)

រកឡាពរននរកឡាទង របសកាា រអកមយ រតវបានគគលាបពណគលឿង គ ើយរកឡាគៅសលរតវលាបពណនបតង។ កាា រអកពណពរ ដដលលាបពណតាមរគបៀបគនោះ សមមលោន គបើកាា រពណមយអាចទទលបានគោយបងវលកាា រពណមយគទៀតកនងបលងកាា រអក។ គតើអាចមានកាា រអកពណមនសមមលោន ចននប ម ន?។ គតើគគមានវធគរៀបគលខ នង ចននប ម នរគបៀប គដើមបឱយផលបក

គលខ៤គៅជាបោន ដចកោចនង៣?។

(អាមេរច ១៩៩៣)

តាង ជាសណមយមានធាត៦។ គតើគគមានប ម នរគបៀបគផេងោន កនងការគរជើសគរ ើសសណរងពរនន ដដលមនចាបាចខសោន កបាន គ ើយដដលរបជននសណរងទងពរជា ? លោបននការគរជើសគរ ើសមនសខានគទ ឧទ រណ គសណរង ដចោន នងការគរជើសគរ ើសគ ដដរ។

(ឥណឌា ១៩៩៨) គគឱយចននគតវជជមាន ។ តាង

ចរបងហា ញថា ចគពាោះរគប បោា ផលគណ សទធដតមានតនមលខសោន ទងអស។ ដលបងគបើកគបៀកនងកពយទរមយគគគលងដតមាន កឯង ដចតគៅ។ គរកាយគលងជាបមយគលើកៗ

គៅតាមលទធផលដដលទទលបាន អនកគលងទទលបាន ឬ ពនា ( ) គ ើយពនារបសោតគកើនគ ើងគោយបកបដនថមនងពនាគលើកមន។ គគដងថា មានពនាចនន របគភទដដលគគមនអាចទទលបាន គ ើយកនងចគោមគ ោះ មានមយគសមើ ។ ចរគណ នង ។

Page 78: Problem Book in Mathematical- Vol I Arithmetic

66 ឡសច |លម សវណណ វចតរ

(អាមេរច ១៩៩៤)

គគមានឥដឋចនន ដ។ មយដៗមានរងហវ ស គគយកមកតគរៀបពគលើោន ជាបគងហគ លមយមានកមពស ដឥដឋ។ ឥដឋនមយៗអាចតគរៀបគោយយករជង ឬ ឬ សសបនងកមពសរបសបគងហគ ល ។ គតើមានកមពសបគងហគ លសរបប ម នរបគភទដដលគគអាចគធវ ើបានគោយគរបើឥដឋទង ដគនោះ?។

(អនតរជាត ១៩៧០) ចរកណតរគបចននគតវជជមាន ដដលគធវ ើឱយសណ

អាចជាបដបកជាសណរងពរ ដដលផលគណននបោា ចននទងអសគៅកនងសណមយ គសមើនង ផលគណននបោា ចននទងអសគៅកនងសណមយគទៀត។ គតើគគរតវដចកប លចនន គៅកនងរបអបចនន យ ងគម ច គដើមបឱយ ផលបកននចននបនេប

លពរៗកនងរបអបនមយៗ បកចលោន នងរបអបទងអសគផេងគទៀត មានចននតចបផត?

Page 79: Problem Book in Mathematical- Vol I Arithmetic

លម សវណណ វចតរ| តរពនធរបារ 67

បរពនធរបារ.ចននគត.ភាព

ចចកដាច

តរពនធរបារ

32. (អនតរជាត១៩៧៥) ពពលពេសរពសរ ពៅកនងរបពនធពោល , ពេទទលបានផលបកតពលខននខទងនមយៗរបសវា ពសមើ ។ តាង ជាផលបកតពលខននខទងនមយៗរបស ។ ចរេណនា ផលបកតពលខននខទងនមយៗរបស ។ ( សរពសរពៅកនងរបពនធពោល )។

33. ចរេណនា ពលខ២ខទងចងពេរបស ។ 34. ចរេណនា ពលខ២ខទងចងពេរបស ។ 35. ចននេតមយថយចះជាេតដង ពពលពេលបតពលខខាងចងពេពោល។

ចរេណនាចននបបបពនះ។ 36. ចរកណតរេបចននេតធមមជាតបដលពផត ើមពោយពលខ នង ថយចះ ដង ពបើពលខ ពនះ

រតវបានលបពោល។ 37. (អនតរជាត ១៩៦៨)

ចរកណតរេបចននេតធមមជាត បដលពផទៀងផទទ តផលេណននតពលខខទងនមយៗរបសវា (ពៅកនងពោលដប) ពសមើនង ។

38. (អាមេរច ១៩៩២) តាង S ជាសណននរេបចននសនទាន បដល ព ើយបដលមានតពលខជាខបរាង

Page 80: Problem Book in Mathematical- Vol I Arithmetic

68 តរពនធរបារ.ចននគរ.ភាពចចកដាច |លម សវណណ វចតរ

បដលតពលខ មនោខសោន កបាន។ ពបើរបភាេនមយៗបដលជាធាតរបស សរមលរចព ើយ ពតើមានភាេយកខសៗោន ចននប នាម ន?។

39. ចរបងហា ញថា រេបចននេតវជជមាន បតងមានព េណរបសវាមយ បដលពលខរបសវាកនងរបពនធទសកៈ (របពនធពោល ) មានរេបតពលខព ដល ។

40. ពលខ ទទលបានពោយពរៀបចននេតតពរៀងោន បនតបនាទ បោន ។ ពយើងតាង ជាអនេមនបដលខទងទ របសពលខបដលឱយ ឋតពៅចចណចបដលពេបបនថមពលខមាន ខទង ។ ឧទា រណ ពររះ ខទងទ សថតពៅចពលខ បដលមាន ខទង។ ចរេណនា ។

41. ចរសរពសរ ពៅកនងពោល ។ 42. ចរសរពសរចននទសភាេ ពៅកនងពោល ។ 43. (អាមេរច ១៩៩៤)

ពេឱយចននេតវជជមាន , តាង ជាផលេណននតពលខខសពសនយរបស ។ (ពបើ មានបតមយខទង ពនាះ )។ តាង

ចរកណតកតាត បឋមធបផតរបស ។

44. (អនតរជាត ១៩៧៨) តាង ជាចននេតធមមជាត បដល ។ ពៅកនងរបពនធរបាបពោល ពលខប ខទងចងពេរបស ពសមើនងពលខបខទងចងពេរបស ។ ចរេណនា ពពល មានតនមលតចបផត។

45. ចរបងហា ញថា ពលខមយរនខទងដបងពរោយពកបៀសរបស សទធបតជាពលខ

ទាងអស។

ចននគរ

46. ចរបងហា ញថា

Page 81: Problem Book in Mathematical- Vol I Arithmetic

លម សវណណ វចតរ| ភាពចចកដាច 69

ជាចននេតចពរះរេបចននេតធមមជាត ។ 47. តាង បដល ជាចននេតមនអវជជមាន។ ចរបងហា ញថា

ជាចននេត។ 48. ចរបងហា ញថា ផលបក

មនអាចជាចននេតពទ។

49. (អាមេរច ១៩៩៣)

ពតើមាន ចននប នាម ន បដលព ពោណនយតមយ មាន រជង មានមកនងជាចននេតេតជាដ

ពរក?។

ភាពចចកដាច

50. ចរបងហា ញថា ពបើ ពសស ពនាះ

ដពចនះ ពបើ ពសស ពនាះ បចកោច ។ 51. ចរបងហា ញថា បចកោច 52. ចរបងហា ញថា ចពរះចននេតធមមជាត មយ ពេមាន ចននេតធមមជាត មយពទៀត បដលត

នមយៗរបសសវត បចកោចនង ។

53. េណនារេបចននេតវជជមាន បដល បចកោច 54. ពបើ បចកោច ចរបងហា ញថា បចកោច ។ 55. ចរបងហា ញថា ពបើ ជាព េណនន ពនាះ ជាព េណនន បដរ នង

រោសមកវញ។ 56. េណនារេបចននេតវជជមាន បដល បចកោច ។ 57. ចរកណតចននេតវជជមាន បដល បចកោច ។ 58. ចរបងហា ញថា បចកោចនង ចពរះចននេត ពរចើនរាបមនអស។

Page 82: Problem Book in Mathematical- Vol I Arithmetic

70 តរពនធរបារ.ចននគរ.ភាពចចកដាច |លម សវណណ វចតរ

59. ចរបងហា ញថា ពបើ ជាចននបឋម ពនាះ បចកោច ។ 60. ចរបងហា ញថា ចពរះរេប បចកោច ។ 61. ចរបងហា ញថា ផលេណនន ចននេតបនតបនាទ បោន បចកោចនង ។ 62. ចរបងហា ញថា បចកោច ចពរះរេបចននេត ។ 63. តាង ជាចននបឋម។ ចរបងហា ញថា ជាព េណនន ។ 64. ចរបងហា ញថា ពកនងចពោមចននេតបោកពោយ ពេអាចពរជើសពរ ើសយកពរ តាងពោយ

បដលពផទៀងផទទ ត បចកោចនង ។ 65. ចរបងហា ញថា ពបើ បចកោច ពនាះ បចកោច ផងនង ផង ។ 66. តាង ជាចននេត។ បងហា ញថា បចកមនោចនង ពបើ បចកោចនង នងរោស

មកវញ។

67. (ហងគរ ១៨៩៩) ចរបងហា ញថា បចកោចនង ចពរះ រេបចននេតធមមជាត ។

68. ចរបងហា ញថា ជាព េណ នន ចពរះ រេបចននេតធមមជាត ។ 69. ចរបងហា ញថា ពបើ ជាចននេតធមមជាតពសស ពនាះ បចកោច ចពរះរេប

ចននេតធមមជាត ។ 70. (រស ១៩៩៥)

ចរបងហា ញថា មានចននេតព េណ ពរចើនរាបមនអស បដល បចកោច ។

71. តាង ជាចននេតវជជមាន នង ជាចននមយបដលសរពសរពោយពរបើតពលខនងចននខទងដច បតតាមលោបលពោយមយពផេងពទៀត។ ចរបងហា ញថា ពបើ ពនាះ បចកោចនង ។

72. (អាមេរច ១៩៩២) បោត ពលខមានពរខទង ោបព ពៅ រតវបានពេយកពៅសរពសរបនតបនាទ បោន ពដើមបបពងក ើតបានជាចននេត

Page 83: Problem Book in Mathematical- Vol I Arithmetic

លម សវណណ វចតរ| ភាពចចកដាច 71

កណត ធបផត បដល បចកោចចននពនះ។ 73. បងហា ញថា ពបើ ជាចននបឋម ពនាះ បចកោច ។ 74. ចរបងហា ញថា ពបើ ជាចននពសសបឋម ព ើយពបើ

ពនាះ បចក ោច។ 75. ចរកណតបោត ចននេត បដល បចកោច បចកោច នង

បចកោច ។

76. (អនតរជាត ១៩៩៤) ចរកណតរេបចននេតវជជមានោចខាត បដល

ជាចននេត។

77. ចរកណតរេបចននេត បដល បចកោច ។

78. (បាលកង ១៩៩៦) ពេឱយចននបឋម ។ ពេកណត

ចរបងហា ញថា មាន នង ជាធាតរបស បដល ព ើយ បចកោច ។ 79. បងហា ញថា ជាចននេតធបផតបដលបចកោចនងរេបចននេត បដល 80. ចននេតវជជមាន មានលកខណៈ

<<ចពរះរេប , ពេមាន បចកមនោចនង >> ចរបងហា ញថា លកខណៈពនះ អាចពកើតមានបតកនងករណ ជាសវយេណនន មយបតប ពោណ ះ។

81. ចរកណតរេបចននេតវជជមាន បដល បចកោច ។ 82. (អនតរជាត ១៩៩២)

ចរកណតរេបចននេត បដល នង បចកោច ។

Page 84: Problem Book in Mathematical- Vol I Arithmetic

72 តរពនធរបារ.ចននគរ.ភាពចចកដាច |លម សវណណ វចតរ

83. ចរបងហា ញថា ចពរះរេបចននេតេវជជមាន ពយើងមាន បចកោច ។ 84. (អនតរជាត ១៩៨៤)

ចរឱយឧទា រណេចននេតវជជមាន មយ បដលពផទៀងផទទ តលកខខណឌ ទាងពរខាងពរោម (i) បចកមនោចនង (ii) បចកោចនង

ចរពផទៀងផទទ តចពមល ើយទទលបាន។

85. តាង ជាចននបឋម។ ចរបងហា ញថា បចកោច ចពរះរេបចននេត នង ។ 86. ពេឱយចននបឋម ។ ចរបងហា ញថា , បដលមានពលខ ចនន ដង, ជា

ចននបចកោចនង ។ 87. បងហា ញថា ពេមាន បចកោចនង ជានចច។ 88. ពេឱយចននបឋមពសស ។ ចរបងហា ញថា មានចននេត ពរចើនរាបមនអស បដល បចកោច

។ 89. ចរបងហា ញថា ោម នចននេត បដល បចកោច ពទ។ 90. ពតើ បចកោច

ឬពទ?។

91. ពេពរជើសពរ ើសយកចននេតចនន ពកនងចពោម ។ ចរបងហា ញថា ពទាះពេពរជើសពរ ើសចននេតទាង ពនាះយ ងពម ចកពោយ កពេេងទទលបាន ចននមយកនងចពោមពនាះ បចកោចចននមយពទៀតបដរ ។

សណល. ភាពសមមល

92. តាង ជាចននបឋមមយ។ ចរបងហា ញថា

i)

ii)

93. តាង ជាចននបឋម បដល ។ ចរបងហា ញថា ។ 94. (អាមេរច ១៩៩៤)

Page 85: Problem Book in Mathematical- Vol I Arithmetic

លម សវណណ វចតរ| ចននការរ 73

តាង ជាសណលននវធបចក នង នង ។ េណនា ។

ចននការរ

95. ចរបងហា ញថា ចននោពរមានរាង ឬ ។ 96. ចរបងហា ញថា ោម នចននេតពៅកនងសវត ជាចននោពរពទ។ 97. ចរបងហា ញថា ពេមានចននេតធមមជាត មយេត បដល ជាចននោពរ។ 98. ចរបងហា ញថា កនងរេបរបពនធរបាបទាងអស សទធបតជាោពរននចននសនទាន ។ 99. ចរកណតចននេត បដល ជាចននោពរ។

100. ចរកណតរេបចននេត បដល ជាចននោពរ។

101. (អនតរជាត ១៩៨៨)

ពបើ ជាចននេតវជជមាន បដល

ជាចននេត ពនាះ

ជាចននោពរ។

102. ពេឱយ ជាចននេត បដល នង ។ ចរបងហា ញថា ជាចននោពរ។

103. ក) ចរបងហា ញថា ផលេណននចននេតពរតោន មនអាចជាចននេតោពរពទ។ ខ) ចរបងហា ញថា ផលេណននចននេតបតោន មនអាចជាចននោពរពទ។ េ) ចរបងហា ញថា ផលេណននចននេតបនតោន មនអាចជាចននោពរពទ។

104. ចរបងហា ញថា ផលេណននចននេតបតពរៀងោន មនអាចជាចននសវយេណពទ (មាននយថា មនអាចជាចននោពរ, ចននេប។ល។)

105. ចរកណតចននមនោពរទ ។ឧទា រណ ជាចននមនោពរទ , ជាចននមនោពរទ , ជាចននមនោពរទ ។ល។

106. តាង ។ ចរបងហា ញថា ចពរះរេបចននេតវជជមាន , សវត

មានចននោពរយ ងពោចោសមយ។ 107. ចរកណតរេបចននេត បដល ជាចននោពរ។

Page 86: Problem Book in Mathematical- Vol I Arithmetic

74 តរពនធរបារ.ចននគរ.ភាពចចកដាច |លម សវណណ វចតរ

108. ចរបងហា ញថា ពៅកនងរេបសណទាងអសបដលមាន ធាត, បដលធាតនមយៗ ជាចននេតខសៗោន ព ើយចននេតនមយៗមានកតាត បឋមជាធាតរបសសណ ; ពេមានធាតពរបដលផលេណរបសវាជាចននោពរ។

109. (អនតរជាត ១៩៨៥)

ពេឱយសណ មាន ធាតជាចននេតវជជមានខសៗោន ។ ោម នធាតោមយមានកតាត បឋមធជាង ពទ។ ចរបងហា ញថា M មានសណរងមយ បដលមាន ធាតខសៗោន បដលផលេណជាចននសវយេណទ ។

ចននចែលមានរាងណាមយ

110. ចរបងហា ញថា រេបចននេតធមមជាត សទធបតអាចសរពសរជាផលបកននចននេតពរធជាង នង បឋមរវាងោន ។

111. (អាមេរច ២០០១)

ពតើមានចននេតវជជមានបដលជាព េណនន ចននប នាម ន បដលអាចសរពសរជារាង បាន បដល នង ជាចននេត ព ើយ ?។

112. ( អាមេរច ១៩៨៦) សវតពកើន

មានរេបចននេតវជជមាន បដលជាសវយេណនន ឬជាផលបកននសវយេណនន ពផេងោន ។ ចរេណនា តទ ននសវតពនះ។

113. (អាមេរច ១៩៧៨) ចននេត ពេពៅវាថាជាពលខពពសស ពបើពេអាចសរពសរជា បាន ពោយ ជាចននេតវជជមាន(មនោបាចខសោន កបាន) បដល

Page 87: Problem Book in Mathematical- Vol I Arithmetic

លម សវណណ វចតរ| ចននចែលមានរាងណាមយ 75

ពោយដងថា ចននេតោបព ដល សទធបតជាពលខពពសសទាងអស ចរបងហា ញថា រេបចននេតទាងអស បដល សទធបតជា ពលខពពសស។

114. (អនតរជាត ១៩៨៣) ពេឱយចននេតវជជមាន នង បឋមនងោន ពរៗ។ ចរបងហា ញថា ជាចននេតធបផត បដលមនអាចសរពសរជារាង បាន បដលកនងពនាះ ជាចននេតវជជមានឬសនយ ។

115. ចរបងហា ញថា រេបចននសនទានវជជមាន សទធបតអាចសរពសរជារាង

បានទាងអស ចពរះចននេត វជជមាន។

Page 88: Problem Book in Mathematical- Vol I Arithmetic

76 តរពនធរបារ.ចននគរ.ភាពចចកដាច |លម សវណណ វចតរ

Page 89: Problem Book in Mathematical- Vol I Arithmetic

លម សវណណ វចតរ| ចននបឋម 77

ពហគណរមតចបផត. ត

ចចករមធបផត

ចននបឋម

116. (ចននមែរមែន) ចរបងហា ញថា បបើ ជាចននបឋម ប ោះ កជាចននបឋមដែរ។

117. ចរបងហា ញថា មានគ បរចើនរាបមនអស ដែល នង មានកតតា បឋមរម ប ើយ នង មានកតតា បឋមរមគនន ដែរ។

118. ចរកណតរគបចននបឋមដែលមានរាង ចប ោះចននគត ។ 119. ចរបងហា ញថា ជាចននបឋម ដតកនងករណ ប ប ណ ោះ ចប ោះ ។ 120. ចរកណតរគបចននគត ដែល ជាចននបឋម។ 121. បគែងថា មានកតតា បឋម ។ គណ កតតា បឋមបនោះ។ 122. ចរគណ ចននននចននបឋមដែល ។ 123. (រ ាមា ន ២០០៣)

បគឱយចននបឋម ។ ចរបងហា ញថា បបើ ដចកដាច

ប ោះកនងចបមចននទងប ោះបគអាចរកបានចននបឋមបតបរៀងគនន ។

ចននពហគណ

124. ចប ោះរគបចននគត បគអាចរកបានចននគត មយ ដែលប ា ចនន សទធដតជាចននព គណ។

Page 90: Problem Book in Mathematical- Vol I Arithmetic

78 ពហគណរមរចបផរ រចចករមធបផរ |លម សវណណ វចតរ

ពហគណរមរចបផរ រចចករមធបផរ

125. តតង ។ ចរបងហា ញថា ឬ ។ 126. តតង ជាចននគតវជជមាន។ ចរបងហា ញថា

127. (អាមែរច ១៩៨៥) ប ា ចននបៅកនងសវត មានរាង ចប ោះ នមយៗ តតង ។ គណ ។

128. ចរបងហា ញថា បបើ នង ជាចននគតធមមជាត ប ើយ ជាចននបសស ប ោះ

129. ចរបងហា ញថា មានសវតនពវនាដវងមយ ដែលតរបសវានមយៗបឋមរវាងគនន ពរៗ។

130. បតើមានចននគតវជជមាន ចននប ម ន ដែលបឋមនង ?។ 131. តតង នង ជាចននបឋមនងគនន ។ ចរបងហា ញថា នង កបឋមនងគនន ដែរ។ 132. បគកណត ចននដភមា ទ បដាយរបមនា

។ ចរបងហា ញថា ចនន ទងអសបឋមនងគនន ពរៗ។

133. (អនតរជាត ២០០៥) បគឱយសវត កណតបដាយ

ដែល ជាចននគតវជជមាន។ ចរកណតរគបចននគតវជជមាន ដែលបឋមនងរគបតទងអសននសវត។

134. គណ គ ននចននគតវជជមានចននប ម ន ដែល ។ 135. (កមរ ា ១៩៩៨)

ចរកណតរគបចននគតវជជមាន បឋមនងគនន ពរៗ ដែល

ជាចននគត។ 136. បគឱយចននគតវជជមាន ។ ចរបងហា ញថា របសប ា ផលគណ

មានតនមលតចជាង ។ កនងបនោះ នង ជាអបថរ នង ជាចននគតវជជមានបផទៀងផទទ ត

Page 91: Problem Book in Mathematical- Vol I Arithmetic

លម សវណណ វចតរ| បចបកជាកតតា បឋម 79

137. (អនតរជាត ១៩៥៩) ចរបងហា ញថា របភាគ

ជារបភាគមនអាចសរមលបានចប ោះរគបចននគតធមមជាត ។

138. (អាមែរច ១៩៧២) ចរបងហា ញថា

139. ចរគណ ចននននចតធាត ដែល

បចបកជាកតតា បឋម

140. គណ ផលគណននរគបប ា តដចកវជជមានរបស ។ 141. គណ ផលបកននតដចកវជជមានគរបស ។ 142. (អាមែរច ១៩៨៨)

ចរគណ របបាប លបត កនងការបរជើសបរ ើសបដាយនចែននវតដចកវជជមានរបស បានជាចននព គណនន ។

143. បយើងបៅថាចននគតធមមជាតមយជាែែបណណមេខ បបើចននប ោះបសមើនងផលបកតដចកវជជមានរបសវា បរៅពខលនវា។ ឧទ រណ ជាសមបណណ បលខ បរ ោះ

។ ចរបងហា ញថា ចននគមយជាសមបណណ បលខ លោះរតតដត វាមានរាង ដែល នង ជាចននបឋម, នងរាសមកវញ។

Page 92: Problem Book in Mathematical- Vol I Arithmetic

80 បចបកជាកតតា បឋម |លម សវណណ វចតរ

Page 93: Problem Book in Mathematical- Vol I Arithmetic

លម សវណណ វចតរ| ផនែកគរ 81

ផនែកគត

144. ចរបងហា ញថា ចព ោះ ពគមាន 145. គណនាផផែកគតរបស

146. ចរបងហា ញថា តរបសសវវតននចននគត ផែល ជាចននគតមន

អវជជមាន ជាចននគនងពសសឆលល សគនែ ។ 147. (អាមេរច ១៩៨៧)

ចរកណតចននគតវជជមាន ធបផត ពែើមបឱយពគមានចននគត មយគតផែល

148. (អាមេរច ១៩៩៧) សនមតថា ជាចននវជជមាន ផែល នង ។ ចរគណនា

149. (អាមេរច ២០០៣)

គណនាចននគតវជជមាន ផែល ពៅជត បផត។ 150. កណតពហធាមនសនយ មយ ផែល ចព ោះរគបចននពត ។ 151. (អនតរជាត ១៩៦៨)

ចរគណនា

152. (សេភាពហ ែរេត)

ចរបងហា ញថា ពបើ ជាចននពត នង ជាចននគតធមមជាត ពនាោះ

Page 94: Problem Book in Mathematical- Vol I Arithmetic

82 ផនែកគរ |លម សវណណ វចតរ

153. (អាមេរច ១៩៨៥)

ពតើកែវងចពោម ចននគតវជជមានែបង ពតើមានប នាម នផែលអាចសរពសរជារាង បាន?

154. (អាមេរច ១៩៩១)

សនមតថា ជាចននពត ផែល

ចរគណនា ។

155. ចរកណតចននតផែលមានតនមលខសគនែ កែវងសវវតខាងពរោម

156. តាង ជាចននគតផែលពហគណរមតចបផតននរាលបោា ពរៗមានតនមលធជាង ។ ចរបងហា ញថា

Page 95: Problem Book in Mathematical- Vol I Arithmetic

លម សវណណ វចតរ| សមការដយផង 83

សមការដយផង

157. ចរគណនាចននគត ដែលផផទៀងផទទ តសមការ 158. គណនាចផមលើយសមការែយផង : 159. ផតើមានចននគត ដែល ឬផទ?។ 160. ចររកចផមល ើយជាចននគត ននសមការ 161. ចរបងហា ញថា សមការ គមា នឫសជាចននគតវជជមានផទ។ 162. សនាតថា ជាចននគត ដែល ។ ចរបងហា ញថា ។ 163. គណនាចននគត ដែល ។ 164. ចរបងហា ញថា សមភាព អាចពតចផ ោះបណតា ចននគត ដត

ករណ ដែល ប ផណតណ ោះ។

165. ចរបងហា ញថា គមា នចននគតដែល ផទ។ 166. ផតើមានចននគតវជជមាន ដែល ឬផទ? 167. (អាមេរច ១៩៧៩)

ចរកណតចននគតមនអវជជមាន ដែលផផទៀងផទទ តសមការខាងផរកាម

168. ផ ោះរាយសមការកនងសណចននគត

169. ផ ោះរាយសមការកនងសណចននគត

170. ចរកណតចននគតវជជមាន ដែល

171. (ហងគរ ១៩៩៧) ចរកណតចផមល ើយជាចននគតននសមការ

Page 96: Problem Book in Mathematical- Vol I Arithmetic

84 សមការដយផង | លម សវណណ វចតរ

172. ចរកណតចននគត ដែល

173. ក) ចរកណតរគបចននគតវជជមាន នង ដែល ។ ខ) ចរកណតរគបចននគតវជជមាន នង ដែល ។ គ) ចរកណតរគបចននគតវជជមាន នង ដែល ។ 174. ចរកណតចននគត ដែល 175. ចរបងហា ញថា ផគមានរតធាតសនទានមនគតវជជមាន ផរចើនរាបមនអស ដែលផផទៀង

ផទទ ត ដែល នង ជាចននគតធបផតដែលតចជាងឬផសាើ ។

176. (ហងគរ ១៩៩៨) ចរកណតរគបចននគតវជជមាន ដែល នង

177. ចរកណតរគបចននគតវជជមាន ដែល ។

178. (លថន ១៩៩៤) ចរកណតរគបចននគតវជជមាន ដែល នង

179. (អតាល ១៩៩៤) ចរកណតរគបចននគត ដែល

180. តាង ជាចននគតវជជមាន ដែល ។ ចរបងហា ញថា ។

181. (មអៀរឡង ១៩៩៥) ចរកណតចននគត ផែើមបឱយសមការ

មានចផមល ើយផរចើនរាបមនអសផៅកនង ?។ 182. ចរកណតរគបចននគតធមាជាត ដែល ។ 183. ក) ចរកណតចននគតធមាជាត ដែល

Page 97: Problem Book in Mathematical- Vol I Arithmetic

លម សវណណ វចតរ| សមការដយផង 85

ខ) ចរកណតរគបចននគតធមាជាត ផែើមបឱយផគអាចរកបាន ចននគតធមាជាត ដែល

184. ចរកណតរគបចននគតវជជមានឬសនយ ដែល 185. (អនតរជាត សតលស ២០០៣)

ចរកណតចននគត តចបផត ដែល ផគមានចននគត ផផទៀងផទទ ត

186. (តតវា ន ១៩៩៨) ផតើសមការខាងផរកាម

ដែល ជាចននគតធជាង មានឫសឬផទ?។

187. (ឥណឌា ១៩៩៨)

ចរកណតរគបចននគតវជជមាន ដែល នង

188. (អនតរជាត ១៩៩៧) ចរកណតរគបគ ននចននគត ដែល ផផទៀងផទទ តសមការ

189. ផយើងពនតយសមការ

ក) ចផ ោះចននគតណតខលោះនន ដែលសមការ (*) មានចផមល ើយ គតផផទៀងផទទ ត ។ ខ) ផ ោះរាយសមការ ចផ ោះ ។

190. ចរកណតរគបចននគតវជជមាន ដែល ដែល ។

Page 98: Problem Book in Mathematical- Vol I Arithmetic

86 សមការដយផង | លម សវណណ វចតរ

191. គទសតបទគតធាតពតាករ

តាង ជាចននគតធមាជាត ដែលមានតដចករមធបផតផសាើ ផផទៀងផទទ ត

ចរបងហា ញថា ឬកមនអចងផទ ជាចននគ។ កនងករណដែល ជាចននគ ចរបងហា ញថា ផគមានចននគត នង ដែលបឋមនងគមន ផ ើយមានភាពគផសសផទយគមន ដែល

192. ក) ចរកណតរតផកាណដកងដែលមានរងហវ សរជងជាចននគតផ ើយមានរកឡានផទជាចននកាផរ។ ខ) ចរកណតចននគតវជជមាន ដែល ។

193. ចរបងហា ញថា គមា នរតធាតននចននគតវជជមាន ដែល ផទ។ 194. គណនាឫសសនទាន ដែល ។ 195. ចរកណតចននសនទាន នង ដែលផផទៀងផទទ ត ។ 196. តាង ជាដផនកគតនន ។ ផតើសមការ

មានឫសឬផទ?។

197. ផ ោះរាយសមការ

198. ផ ោះរាយសមការ

199. (អសតរត ល ១៩៩៩) ចរផ ោះរាយរបពនធសមការ

200. ចរបងហា ញថា មនអាចផសាើ ផទ។ 201. តាង ជាចននគត ដែល

ចរបងហា ញថា មានចផមល ើយ។

Page 99: Problem Book in Mathematical- Vol I Arithmetic

លម សវណណ វចតរ| សមការដយផង 87

202. (អនតរជាត ១៩៨៤) តាង នង ជាចននគតវជជមានផសស ដែល នង ដែល ជាចននគតពរ។ ចរបងហា ញថា ។

203. (មអៀរឡង ១៩៩៦) តាង ជាចននបឋម នង ជាចននគតវជជមាន។ ចរបងហា ញថា ផបើ ផនាោះ ។

204. (អនតរជាត ១៩៨១) គណនាតនមលធបផតរបស ដែលកនងផនោះ ជាចននគតវជជមាន ដែល

នង ។

205. ចរកណតតនមលវជជមានតចបផត នន ចផ ោះរគបចននគតវជជមាន នង ។ 206. ចរកណតរគបចននបឋម នង ចននគត ដែល 207. តាង នង ជាសវតននចននគតពរ ដែលផផទៀងផទទ ត សវត នង ជា

សវតនពវនា។ ចរបងហា ញថា មានចននផថរ មយដែលចផ ោះរគប ផយើងមាន ឬ ។

208. តាង

ជាព ធាែផរកទ ផ ើយមានផមគណជាចននគត។ ផបើ សទធដតជាចននផសស ចរបងហា ញថា គមា នឫសជាចននសនទានផទ។

209. ចរបងហា ញថា ផបើព ធា

ដែលមានផមគណជាចននគត នង មានតនមលផសាើ រតង ចននគតផផេងគមន នន ផនាោះ គមា នតនមល ណត ដែលព ធាផនោះ មានតនលផសាើ ផទ។

Page 100: Problem Book in Mathematical- Vol I Arithmetic

88 សមការដយផង | លម សវណណ វចតរ

Page 101: Problem Book in Mathematical- Vol I Arithmetic

លម សវណណ វចតរ| ឡសច 89

ឡស ច

បបើ ជាសវយគណបោល២ ប ោះសបណើ ពត បោយមនចបាចរាយបញជជ ក។ បបើ មនមមនជាសវ

យគណបោល២ ប ោះវារតវបៅចប ល ោះ សវយគណបោល២ ពរតបរៀងោន មាននយថា ។ ឱយ ឱយ មាននយថាមានចននសវយគណបោល២ បៅចប ល ោះ (បសមើនង )។

បយើងនងរាយបញជជ កតាមវចរបោយកបណើ ន។

បោយារ ជាសណមនទបទននចននគតវជ ជមាន ប ោះ មានធាតមដលតចបផត តាងបោយ ។ តាមសមមតកមម ជាធាតរបស មត មត ជាធាតតចជាងបគ ដបចនោះ ទាលមត ។ មាននយថា ជាធាតមយរបស ។ បោយ ជាធាតមយរបស ប ោះ កជាធាតរបស មដរ ប ើយដចោន មដរ ។ល។ ដបចនោះ រគបចននទាងអសមដលមានរាង ជាធាតរបស ។ ប ទ បមកបទៀត កជាធាតរបស មដរ មាននយថា រគបចននសវយគណបោល២ទាងអសសទធមតជាធាតរបស ។ ឥលវបយើងសនមតថា មាន មយមដលមនមមនជាធាតរបស ។ ដបចនោះ មនមមនជាសវយគណបោល២ បទ។ បោយ ប ោះ ោម នចននគតណាមយ បៅចប ល ោះ មដលជាធាតរបស បទ បររោះ បគមាន មដល ដបចនោះ (បររោះ បបើ ប ោះ កជាធាតរបស មដរ) ។ ដចោន មដរ ចបរោះ ោម នចននគត ណាមយជាធាតរបស បទ បររោះ បបើ ។ បោយ ប ោះ មនមមនជាធាតរបស មនមមនជាធាតរបស មដរ។

Page 102: Problem Book in Mathematical- Vol I Arithmetic

90 ឡសច |លម សវណណ វចតរ

តាមវចរបោយកបណើ ន បយើងទាញបានថា ោម នធាតណាមយ មដលឋតបៅចប ល ោះ

ជាធាតរបស បទ។

ប ទ បមកបទៀត បយើងនងបងហា ញថា ចប ល ោះបនោះធណាសមដលរតវមតមានចននសវយគណបោល២ បៅកនងប ោះ។ បបើដបចនោះមមន ប ោះវាផទយពការសនមតមដលថា ោម នធាតណាមយរបស មដលជាធាតរបស ប ោះបទ។

អនគមន ជាអនគមនបកើន ចបរោះ ។ ដបចនោះ ។ បោយអនគមន ជាអនគមនចោះបលើ ប ោះ បគមានចននគតវជ ជមានធរគបរោន មយ មដល

។ មតរគបចននគតវជ ជមាន ចប ល ោះ សទធមតមានចនន

សវយគណបោល២ ជានចច។ ដបចនោះ រតវមតមានចននសវយគណបោលពរមដលជាធាតរបស

។ បោយចននសវយគណបោលពរជាធាតរបស ប ោះ បយើងទាញបានភាព

ផទយនងការឧបមាមដលបយើងចងបាន។ ដបចនោះសបណើ ពត៕

. មចក ជាសណរបាមយ នង

សណ ទងបរមយន េះមា លកខណៈពនសសបរង ន ើច គរពរឋរកនងសណ តរមយន េះវា

នទៀងផទទ រ ។ ឧទហរណ នយើងមា ។ ឧទហរណ

នយើងមា ។ ឧទហរណ នយើងមា

។ សណ ខលេះដចជា គមា លកខណៈពនសសត ន េះនេ នបរេះន ើនយើងយកច គរពរ

មា នយើងទញរ ។

បយើងរតវបរជើសបរ ើសយកចននគតរបាពរបរសចមតចតត បចញពសណទាងរបាមយខាងបលើ។ តាមបោលការណរទងររប រតវមតមានចននគតពរកនងចបណាមចននគតទាងរបាពរប ោះ មដលឋតបៅកនងសណមតមយកនងចបណាមសណទាងរបាមយខាងបលើ។ ដបចនោះ ចននគតពរបនោះបផទៀងផទទ ត ។

Page 103: Problem Book in Mathematical- Vol I Arithmetic

លម សវណណ វចតរ| ឡសច 91

ឧទហរណ នយើងនបរើសនរ ើសនបសចតរចរត ឧទហរណសណ មយនយើងយកច គរមយ ច

បរមយដ ងមា នហើយច មយនេៀរនដើមប ងគងរ បរពរឧទហរណយកនលខ

។ នយើងទញរ ង ឋរកនងសណ តរមយ នហើយវានទៀងផទទ រ ។

. ពសណមយមដលមាន១០ធាត បគអាចបបងកើតបាននវសណរងមនទបទខសៗោន ចនន

(តាមរបមនត

) សណរងមដលមានផលបកធាតមានតនមលធបផតគ ប ើយមានផលបកបសមើ

បយើងមានសណរងទាងអសចនន ។ ផលបកធាតរបសសណរងនមយៗទាងប ោះរតវមានតនមល ចបព ដល ។ ដបចនោះវារតវមតមានសណរងពរមដលមានផលបកបសមើោន ។

. បយើងសបងកតបឃើញថា បបើបយើងបរជើសបរ ើស ចននគត បចញព ចននគតបនតប ទ បោន

ប ោះបគរតវមតទទលបានចននពរ មដលមានផលសង បសមើ ។ បររោះ បយើងអាចចបផគ ចននគតបនតប ទ បោន បៅជា សណ

ដបចនោះ បបើបយើងរតវបរជើសបរ ើស យក ចននគតបចញព សណ ប ោះ វារតវមានចននគតពរ មដលឋតបៅកនងសណមតមយ ប ើយមានផលសងបសមើ ។ ឥលវ បយើងបបងកើតសណ មដលមាន ធាត ដចខាងបរកាម

បយើងរតវបរជើសបរ ើសយក ចននគត បចញព សណ ខាងបលើ ដបចនោះរតវមានចននគតយាងតច ឋតកនងសណមតមយ។ សណនមយៗមានធាតចនន ជាចននគតបនតប ទ បោន ។ ដបចនោះរតវមតមានចននគតពរមដលមានផលសងបសមើ ។

Page 104: Problem Book in Mathematical- Vol I Arithmetic

92 ឡសច |លម សវណណ វចតរ

. ឧបមាថា បយើងទាញថាសបចញមយចននដបងមានបលខខសោន ទាងអស បោយមនមានថាសណា

មានបលខដចោន ដលបៅ ដងបទ។ ដបចនោះ សនមតថាបយើងទាញបចញថាសមានបលខ១ដលបលខ៩ ទាងអស ដបចនោះ មានទាងអស ថាស។ ដបចនោះ ថាសបៅសលមានមត បលខ១០ដលបលខ៥០។ ប ទ បមកបទៀត សនមតថា បយើងទាញយកថាសពកនងចបណាម ថាសបលខ ១០ដលបលខ៥០ មយមខចនន (បដើមបកឱយទានមានថាសចនន មានាល កដចោន )។ ដបចនោះ បយើងយកថាសបចញបាន ចនន ប ើយ។ ថាសមដលទាញបចញបលើកទ នងបធវ ើឱយមានថាសចនន មានបលខដចោន ។

. សនមតថាសខជាមនសសមាន កបៅកនងមនសសទាង១៧ កប ោះ។ ោតសរបសរសបរតបៅមនសស១៦

កបទៀត។ តាមបោលការណរទងររប មានរបធានបទមយ កនងចបណាមរបធានបទទាង៣ មដលសខសរបសរបៅកានមនសសយាងតច៦ កបផសងោន មដរ ។ បៅរបធានបទប ោះថា របធានបទបលខ១។ បបើសនជាមាន២ កកនងចបណាម៦ កបនោះ សរបសរបៅកានោន បៅវញបៅមកនងបៅសខ ពរបធានបទបលខ១ មដរ ប ោះ មាននយថា មានយាងតចមនសសចនន៣ ក មដលសរបសរសបរតបៅកានអនកដនទពរបធានបទមតមយដចោន ។ ផទយបៅវញ សនមតថា អនកទាង៦ កបនោះ សរបសរបៅកានោន បៅវញបៅមក មតបលើរបធានបទបលខ២ ឬបលខ៣ បាបណាណ ោះ។ សនមតថា បៅបៅកនងចបណាមអនកទាង៦ កបនោះ។ តាមបោលការណរទងររប មានរបធានបទមយ កនងចបណាមរបធានបទ២ ឬ៣ មដលបៅសរបសរបៅកានមនសសយាងតច៣ កបផសងោន មដរ កនងចបណាមមនសសទាង៥ កបទៀត។ បៅរបធានបទបនោះ ថា របធានបទបលខ២។ បបើសនជាមាន២ ក កនងចបណាម៣ កបនោះ សរបសរបៅកានោន បៅវញបៅមក នងបៅបៅ ពរបធានបទ បលខ២បនោះ ប ោះ មាននយថា មានយាងតចមនសសចនន៣ ក មដលសរបសរសបរតបៅកានអនកដនទពរបធានបទមតមយដចោន ។ ផទយបៅវញ បបើអនកទាង៣ ក បនោះ សរបសរបៅកានោន បៅវញបៅមក មតពរបធានបលខ៣ ប ោះក មាននយថា មានយាងតចមនសសចនន៣ ក មដលសរបសរសបរតបៅកានអនកដនទពរបធានបទមតមយដចោន ។

Page 105: Problem Book in Mathematical- Vol I Arithmetic

លម សវណណ វចតរ| ឡសច 93

. បយើងដងថា ចបរោះ បយើងមាន សទធមតជាចននព គ

ណ។ ដបចនោះ បយើងអាចរកបានចននបឋម មយមដល ប ើយបផទៀងផទទ ត សទធមតជាចននព គណ។ បៅរតវគណ ចននបឋម បនោះសន។ បៅរបាបបលខ បៅសខ។ បបើ ប ោះ ជាចននបឋម។ មតបបើ ប ោះ មាននយថា មនមមនជាចននបឋម។ ដបចនោះបបើសខបឆលើយថា ជាចននបឋម ប ោះមាននយថា ។ មតបបើសខបឆលើយថា មនមមនជាចននបឋមបទ ប ោះបៅដងថា ប ើយរតវឱយបលខបផសងបទៀតបៅសខ។ បពលបនោះបៅឱយបលខ ។ បបើ ប ោះ ជាចននបឋម។ មតបបើ ប ោះ មាននយថា មនមមនជាចននបឋម។ ដបចនោះបបើសខបឆលើយថា ជាចននបឋម ប ោះមាននយថា ។ មតបបើសខបឆលើយថា មនមមនជាចននបឋមបទ ប ោះបៅដងថា ប ើយរតវឱយបលខបផសងបទៀតបៅសខ។ បបើសនជាខសរ ត បៅរតវាកតាមវធបនោះរ តដលករណ ថាបតើ បសមើ ឬអត? បពលបនោះបៅឱយបលខ ។ បបើ ប ោះ ជាចននបឋម។ មតបបើ ប ោះ មាននយថា មនមមនជាចននបឋម។ ដបចនោះបបើសខបឆលើយថា ជាចននបឋម ប ោះមាននយថា ។ មតបបើសខបឆលើយថា មនមមនជាចននបឋមបទ ប ោះបៅដងថា មាននយថា ។ ដបចនោះបយើងសននោា នបានថា បៅរតវសរយាងបរចើន សនរ។

. សនរខាងបលើសមមលនងសបណើ “បបើឆន ណាមយមាននថៃសរកទ១៣ ប ោះរតវមាននថៃអាទតយទ១”

។ តារាងខាងបរកាមគណ នថៃទ១ននមខនមយៗ បតើរតវនងនថៃអវ។ កនងតារាងបនោះ - ជរបដកទ 1ជរឈរទ 2 : បលខ 1 ជានថៃទមយ ននឆន ថ ម។ ជរឈរទ 3 សណលនន មចកនង 7 - ជរបដកទ 2 ជរឈរទ 2 : មខមករាមាន 31 នថៃ ដបចនោះនថៃទ 1 ននកមភៈរតវនងនថៃទ 32 គតពបដើមឆន មក។ ជរឈរទ 3 សណលនន មចកនង 7

Page 106: Problem Book in Mathematical- Vol I Arithmetic

94 ឡសច |លម សវណណ វចតរ

- ជរបដកទ 3 ជរឈរទ 2 : មខមករាមាន 31 នថៃ មខកមភៈអាច 28 ឬ 29 នថៃ ដបចនោះនថៃទ 1 ននម រតវនងនថៃទ 60 ឬ 61 គតពបដើមឆន មក។ ជរឈរទ 3 សណលនន ឬ មចកនង 7 …… តាមរយៈតនមលកនងជរឈរទ បយើងបឃើញថា បោយមចកលោបនថៃននបដើមមខនមយៗ គតពបដើមឆន មក បយើងទទលបានសណលមានតាងពបលខ ដលបលខ ។ មាននយថា នថៃទមយននមខនមយៗ ធាល កចនថៃណាមយ រគបនថៃទាង ។ ដបចនោះវារតវមតមាននថៃអាទតយទ កនងមខណាមយ។ មខប ោះជាមខមដលមាននថៃសរកទ ។

1 2 3

មខ លោបនថៃបដើមមខនមយៗគតពបដើមឆន មក

មករា កមភៈ (= ១ កមភៈ) ម ឬ (=១ម ) ឬ បមា ឬ ឬ ឧសភា ឬ ឬ មថ ឬ ឬ កកកោ ឬ ឬ សហា ឬ ឬ កញជា ឬ ឬ តលា ឬ ឬ វចឆកា ឬ ឬ ធន ឬ ឬ

Page 107: Problem Book in Mathematical- Vol I Arithmetic

លម សវណណ វចតរ| ឡសច 95

. បយើងតបរៀប ចននគតជា សណ ដចខាងបរកាម

សណនមយៗមានធាតរបសវាបឋមនងោន ។ បោយបគរតវបរជើសបរ ើស ចននគត ប ោះបគរតវមតបានចននគតពរមដលបៅកនងសណមតមយ។ មាននយថា រតវមតទទលបានចននពរមដលបឋមរវាងោន ។

. បគរតវការបរបើបលខចនន ។ ទពបលខ១ ដលបលខ៩ រតវបរបើបលខ

ចនន៩បលខ។ ពបលខ១០ដលបលខ៩៩ រតវបរបើបលខ បលខ ប ើយទរពបលខ១០០ដលបលខ៩៩៩ រតវបរបើបលខចនន ។ ទបៅសលរតវការតបលខចនន

។ ដបចនោះរតវមានទចនន បទៀត មដលទនមយៗបរបើបលខ៤ខទង។ ដបចនោះជាសរបមានទចនន ។

. កនងចបណាមចននគតមដលធមនបលើសព , ចននមដលជាព គណនន មានចនន

; ចននមដលជាព គណនន មានចនន ; ចននមដលជាព គណនន មានចនន ។ ដបចនោះចននមដលជាព គណនន ឬ មានចនន (ដក បចញ បររោះរាបរចមដលបាណន ងដង)។

ប ទ បមកបយើងរតវដកចននមដលជាព គណនន៥ មដលព គណនង៣ ឬនង៤ បចញ។ ចននព គណនង៥ផងនង៣ផងមានចនន ។ ចននព គណនង៥ផងនង៤ផងមានចនន ។ ចននមដលជាព គណនន៣ ៤នង៥ មានចនន ។ ដបចនោះចននមដលជាព គណនន៥ មដលព គណនង៣ ឬនង៤ មានចនន ។

ដបចនោះចននមដលជាព គណនន៣ឬនន៤ មតមនមមនជាព គណនន៥មានចនន ។

. តាង ជាបលខខទងទ១៩៨៣។ បយើងកាតតបលខបរកាយបកបៀសរបស រតមខទងទ១៩៨៣ជា

Page 108: Problem Book in Mathematical- Vol I Arithmetic

96 ឡសច |លម សវណណ វចតរ

មានបលខ៩ខទង។ មានបលខ ខទង។ ដបចនោះ រតវមានបលខ

ខទង។ បយើងមាន ។ ដបចនោះ មានបលខ៣ខទងដបងចនន ។ បលខ៣ខទងទមយបសមើ១០០ ទពរបសមើ១០១....ទ៥៩៨បសមើ ។ ដបចនោះខទងទ១៩៨៣បសមើ ។

. សនមតផទយបៅវញថា មានរបបៀបអងគយមដលោម នបកមងរសបៅអមសងហា ងបកមងរបស។

បយើងកណតបលកមយបោយ បកមងរសមាន កឬពរ កអងគយជាបោន ប ើយមានបកមងរបសអងគយសងហា ង ...របស.រស.(រស).របស....

បដើមបកឱយមានបកមងមាន កមដលអនកបៅសងហា ងវា ជាបកមងរសប ោះទាលមតបលកនមយៗមានបកមងរសបៅកណាត លយាងបរចើនពរ ក ប ើយរតវមានបកមងរបសអងគយអមយាងតចពរ ក។ ដបចនោះបលកមបបបនោះ មានយាងបហាចណាសចនន ប ើយរតវការបកមងរបសយាងបហាចណាសចនន កមដរបដើមបអងគយបៅរវាងបលកនមយៗ។ បាមនតបយើងមានបកមងរបសមត២៥ កមតបាបណាណ ោះ។ ដបចនោះការសនមតរបសបយើងមនអាចបៅរចបទ។

. តាងបលខបជើងនមយៗរបសពងរងបោយបលខ១ដលបលខ៨។ តាង នង ជាបរាមបជើង

នងមសបកបជើង មដលសថតបៅបជើងទ ។ តបរៀបមដលអាចននបរាមបជើងនងមសបកបជើង ជាចលាសនន មដល រតវបៅមខ ចបរោះ ។មានចលាសននចននទាង១៦បនោះ ចនន ប ើយ បៅមខ ចលាសមានចននរកកណាត លគ ។ ដចោន មដរ ចននចលាសមដល បៅមខ មានចននរកកណាត លននចននបរកាយបនោះគ ។ ដបចនោះបយើងទាញបានចននចលាសមដលមាន បៅមខ មដល មានចនន ។

. សនមតថាយវជនប ោះយកបរាមបជើង៤ដបងមានពណខសោន ទាងអស។ ដបចនោះវាចបមនទាន

បានមយគបទ។ វាយកបរាមបជើងមយបទៀត។ បទាោះជាបរាមបជើងបនោះពណអកបោយ កគងទទលបានបរាមបជើងមានពណដចោន មយគមដរ។ សនមតថាពណបខៀវ។ ដបចនោះយវជនបនោះចបបានបរាមបជើងពណបខៀវ២ រក ម១ នបតង១ នងបមម ១។

Page 109: Problem Book in Mathematical- Vol I Arithmetic

លម សវណណ វចតរ| ឡសច 97

វាចបយកមថមមយបទៀត។ ករណអនបផតគវាយកបានបរាមបជើងមយបទៀត មដលមានពណបខៀវ បររោះមនអាចផសបានបរាមបជើងមយគមថមបទៀតបទ។ ដបចនោះបរាមបជើងមដលយកបចញប ើយមាន បខៀវ៣ រក ម១ នបតង១ នងបមម ១។ វាយកបរាមបជើងមយបមនថមបទៀត។ បពលបនោះបទាោះជាយកពណអកគងផសបានបរាមបជើងមយគបទៀតមដរ។ សនមតថាវាចបចពណបខៀវដមដល។ ដបចនោះវាទាញបរាមបជើងបចញមកបរៅបាន ពណបខៀវ៤ រក ម១ នបតង១ នងបមម ១។ តាមរបបៀបដចោន វាយកបរាមបជើងបចញបាន ពណបខៀវចនន២០ រក ម១ នបតង១ នងបមម ១។ មាននយថាវារតវយកបរាមបជើងបចញចនន២៣។

. របភាគមយមដលបគបរងមរច មានភាគមបងនងភាគយកបឋមនងោន ។ ដបចនោះកតាត បឋមមយ

មនអាចសថតបៅកនងភាគយកផងភាគមបងផងបានបទ។ កតាត បឋមរបស មានចនន៨ គ នង ។ កតាត បឋមនមយៗកនងចបណាមបនោះ រតវឋតបៅបលើភាគយកឬបៅបលើភាគមបងបានមតមតង មតមនអាចឋតបៅបលើភាគយកផង ភាគមបងផងបានបទ។ ដបចនោះបគមាន របបៀប។ មតចននទាង មដលទទលបានប ោះរតវបធវ ើឱយរបភាគមានតនមលតចជាង១។ កនងចប មរបភាគទាង២៥៦ប ោះ បយើងអាចមចកជារបភាគចនន១២៨គ មដលគនមយៗគណោន បសមើ១ ឬកនងគនមយៗមានរបភាគមយមានតនមលតចជាង១។ ដបចនោះចននសនទានមដលមានលកាណៈចងបានមានចនន ១២៨។

. ចននគតបសស នមយៗអាចជនសបោយ មដល ជាចននគតវជ ជមាន។ បោយ

ដបចនោះ ។ ចតធាត មានដចជា ដចោន នងបលខ១ចនន

១៧ ខណឌ បោយបលខ០ ប ទ បមកបលខ១ចនន៥ ខណឌ បោយបលខ០ ប ទ បមកបលខ១ចនន១១ ខណឌបោយបលខ០ ប ទ បមកបលខ១ចនន១៨

Page 110: Problem Book in Mathematical- Vol I Arithmetic

98 ឡសច |លម សវណណ វចតរ

ដបចនោះចតធាតបនោះដចោន នងបលខ១ចនន៥១ រតវមចកជាបនរកមខណឌ បោយបលខ០។ បគមាន

របបៀបកនងការសកបលខ០ចលកនងចប ល ោះទាង៥០រវាងបលខ១ទាងអស៥១។

. សនមតថា មានទរមងទសភាគជា ។ បបើមយកនងចបណាម ឬ ជាបលខ ឬ

ប ោះនងមានរតាទក បពល បកនង ។ បបើ នង ឬ បបើ នង ឬ ប ោះកនងមានរតាទកមដរ បពល បកនង ។ បបើ ជាចននគតមដលមានលកាណៈខាងបលើ ប ោះវារតវមានរាងមយខាងបរកាម

មដល ។ ទាងបនោះមនរតវការរតាទកបទ បពលបយើងបក នង ។ តនមល មបបបនោះ មានទាងអសចនន ។ ដបចនោះមានចននគតបនតប ទ បោនទាងអស គ មដលផលបកគនមយៗមនរតវការរតាទក។

. បៅអខាងចងទាងពររតវមតសរមាបសសសអនកអងគយ ដបចនោះាសតាត ចរយមានបៅអកនងចនន៧

សរមាបបរជើសបរ ើស បោយមនឱយអងគយបៅជាបោន ។ បបើបៅអទាងបនោះរតវតាងបោយបលខ២ដលបលខ៨ ប ោះបៅអទាងបសរមាបាសតាត ចរយមាន កនងរតធាតនមយៗ ាសតាត ចរយអាចផទល សបតរកមនលងោន បាន របបៀប។ ដបចនោះជាសរបមាន ។

. តាង ជាចននគតទាង១៦ប ោះ។ បយើងពនតយគបផសងៗោន ននចននគតទាង

បនោះ។ បគអាចផសបានគមបបបនោះចនន គ។

តាង ជាគមយ មដល ។ បបើបយើងមានគពរបផសងោន នង មដល ប ោះបយើងទាញបានចតធាតមដលចងបាន បលើកមលងមតករណ ។

បយើងថា មនបានការ ចបរោះគពរ គ នង បបើ (ឬ )។ បយើងសមាគ លបឃើញថា សបន ើពត បបើ មនបានការចបរោះគពរននគចននគត បររោះ

Page 111: Problem Book in Mathematical- Vol I Arithmetic

លម សវណណ វចតរ| ឡសច 99

ថា បបើ មនបានការចបរោះ នង ប ោះ

ជាចងបរកាយបយើងសនមតថា នមយៗ មនបានការ បរចើនបផតបៅកនងគមយននគចននគត។ ចបរោះគននគចននគតនមយៗទាងបនោះ បយើងដកគននចននគតមយបចញ។ ដបចនោះោម នចននមនបានការបទៀតបទ។ ដបចនោះបយើងបៅសលយាងបហាចណាស គននចននគតមដលបៅសល។ ផលសងននចននបៅកនងគបៅសលនមយៗ មានតនមលព១ដល៩៩។ តាមបោលការណរទងររប មានផលសងខលោះមានតនមលដចោន ។ សនមតថា ដបចនោះ បផទៀងផទទ តលកាខណឌ សនរ។

. បដើមបឱយឥដាមយដមានលកាណៈខសពឥដាមយដបទៀត បគមានជបរមើសមយបៅបលើសមាភ រៈ

ជបរមើសពរបៅបលើទ ជបរមើស៣បៅបលើពណ នងជបរមើស៣បៅបលើរទងរទាយ។ មាន

របបៀបមដលឥដាមយដខសពឥដាមយដបទៀត រតងពរចណចគត គ ១) សមាភ រៈនងទ ៖ មានឥដា ចនន ដបផសងោន ២) សមាភ រៈនងពណ៖ មានឥដា ចនន ដបផសងោន ៣) សមាភ រៈនងរទងរទាយ៖ មានឥដា ចនន ដបផសងោន ៤) ទ នងពណ៖ មានឥដា ចនន ដបផសងោន ៥) ទ នងរទងរទាយ៖ មានឥដា ចនន ដបផសងោន ៦) ពណនងរទងរទាយ៖ មានឥដា ចនន ដបផសងោន ដបចនោះជាសរបមាន ឥដាចនន ដ មដលមានលកាណៈខសពដមយដមដលឱយរតងពរចណចគត។

. ចននបលខមដល អាចបបងកើតបានបោយបរបើបលខ០ដល៩ ជារាង មានចនន

។ ដបចនោះបយើងមានករណ

(សណ )ចនន

ករណ នង ករណ

(សណ )ចនន ករណ។ ករណ

អាចមតបពល មតបាបណាណ ោះ(សណ )។ ករណមបបបនោះមាន ករណ។ ដបចនោះ

Page 112: Problem Book in Mathematical- Vol I Arithmetic

100 ឡសច |លម សវណណ វចតរ

. បគមាន របបៀបកនងការបរជើសបរ ើសទតាងននរកឡាពណបលឿង។ បោយារបគ

អាចបងវលកាត រអក៩០ដបរក ប ោះកាត រពណមនសមមលោន មានតចជាង ន ង។ កាត រពណមដលមានពណបលឿងមនឋតបៅឈមោន ជាអងកតផចត មានទរងសមមល៤មបប។ កាត រពណមដលរកឡាពណបលឿងពរឈមោន ជាអងកតផចត មានទរមងសមមលពរ ប ើយបយើងមានគរកឡាពណបលឿងមបបបនោះចនន

។ ដបចនោះចននកាត រអកពណមនសមមលោន មានចនន

Y1 Y2 x Y2 Y1

រកឡាY1 ឈមោន ជាអងកតផចត។ រកឡាY2 ឈមោន ជាអងកតផចត។

. បោយបយើងចងសកាសនរបនោះ តាមរយៈសមមលតាម៣ ដបចនោះបយើងសរបសរបលខ

នង ជា ។ សនមតថា ជាតបរៀបមដលបយើងចងបាន។ បយើងបឃើញថា

។ ដបចនោះ រតវមតជាតបរៀបនន បររោះ ។ បោយ ប ោះ ។ តាមរបបៀបដចោន បយើងទាញបានថាលោបរបស កណតបោយ មតមយគត។

Page 113: Problem Book in Mathematical- Vol I Arithmetic

លម សវណណ វចតរ| ឡសច 101

ដបចនោះបយើងមាន របបៀប (បដើមបបរជ ើសបរ ើស បយើងមាន របបៀប។ សនមតថាដបងបរជើសបរ ើសយក ដបចនោះ បដើមបបរជ ើសបរ ើស បយើងមាន របបៀប បដើមបបរជ ើសបរ ើស បយើងមាន របបៀប បដើមបបរជ ើសបរ ើស បយើងមាន របបៀប ដបចនោះបយើងមាន របបៀបកនងការបរជើសបរ ើស ។ ប ទ បមកចលាសរបសវាមាន )។

. បដើមបឱយ ប ោះចបរោះធាតនមយៗ របស បគមានសបណើ មយគតកនងចបណាម

សបណើ ទាងបខាងបរកាមមដលពត

ដបចនោះបបើ មាន ធាត ប ោះមាន កនងការបរជើសបរ ើសសណ នង ។ បរៅពករណ បយើងរាបជាបោន ពរដង បររោះឧទា រណ គសណរង នងគ រតវរាបមតមតង។ បោយ ប ោះ ទាលមត ។ ដបចនោះចននគននសណរងមដលមានរបជរបសវាជា បសមើនង

មដលបសមើ បពល ។

. សនមតថាផទយមកវញថា មានចននគតបនបផសងោន មដល ។ តាង

មដល ជាចននគត។

ដចោន បយើងទាញបាន ។ បយើងមាន

បបើ ប ោះ ឬ ។ មតបយើងរតវសកាករណ ដបចនោះបយើងមនផទតបចលករណបនោះ។ ដបចនោះ ។ បយើងទាញបាន

Page 114: Problem Book in Mathematical- Vol I Arithmetic

102 ឡសច |លម សវណណ វចតរ

បោយ បររោះ សទធមតជាចននគត។ ករណ ។ ដបចនោះ ជាឬសននសមការ

សមការបនោះមានឬស

មដល

បយើងមាន ទាលមត នង បររោះ ។ ដបចនោះ

មតមនអាចបររោះ ។ ដបចនោះ ។

បបើ ប ោះ

បបើ ប ោះ

កនងករណទាងពរ បយើងមាន ឬ មានតនមលោចខាតធជាង មដលករណបនោះផទយពសមមតកមម។ ដចោន ចបរោះករណ ។

. ពន ទមដលបគអាចបធវ ើបាន ជាចននគតមនអវជ ជមានមដលមានរាង ។ តាមរទសតបទ

Bachet-Bézout បបើ ប ោះ រតវមតមចក ោច។ បបើ ប ោះមានចននគត បរចើនរាបមនអស មដលមចកមនោចនង ។ ដបចនោះមានពនទបរចើនរាបមនអសមដលបគមនអាចទទលបាន ករណបនោះផទយពសមមតកមមមដលមានមតពនទចនន បាបណាណ ោះមដលបគមនអាចទទលបាន។ ដបចនោះ ។ តាមរទសតបទរបបន ចននពន ទមដលមនអាចទទលបានមាន ចនន ។ ដបចនោះ

Page 115: Problem Book in Mathematical- Vol I Arithmetic

លម សវណណ វចតរ| ឡសច 103

លកាខណឌ នង ឱយបយើងទាញបានពរករណ នង ។ បោយ

ប ើយ ជាពនទមដលបគមនអាចទទលបាន ដបចនោះករណទ១មនរតមរតវ។ ប ទ ត កាតតាមចណច នង ។ ប ទ តបនោះមនកាតតាមចណចគត

ណាមយបៅកនងការដងទ១បទ។ ដបចនោះមនមានចននគតវជ ជមាន មដលបផទៀងផទទ តសមការបនោះបទ។

ដបចនោះចបមលើយមានមតមយគតគ ។

. តាង ជាចននដឥដាមដលតបរៀបជាកមពស នង បរៀងោន ។ បយើងចងដងថាបតើផល

បក មានបា ម នរបបភទខសោន បោយឱយលកាខណឌ

។ បយើងមាន ។ បោយយក បយើងរាបចននចបមលើយមនអវជ ជមានរបសវសមភាព

តាមរទសតបទរបបន រគបចននគត សទធមតអាចសរបសរជារាង បាន ប ើយតាមរទសតបទរបបន ចនននន មដលមនអាចសរបសរជារាង បាន មាន

ចនន

។ បយើងមានករណ នង មដលមនអាចសរបសរជារាង

Page 116: Problem Book in Mathematical- Vol I Arithmetic

104 ឡសច |លម សវណណ វចតរ

បាន។ ដបចនោះ កនងចបណាម ចននគតមនអវជ ជមាន មដល បយើងបឃើញថា មាន ចននគតមដលអាចសរបសរជារាង បាន។ យក បយើងទាញបាន

។ តាមរទសតបទរបបន បណាត មដលមនអាចសរបសរជា

រាង បាន មានចនន

ករណ ។ ប ើយតាមរទសតបទរបបន បណាត

ទាងប ោះរតវមត មដលកនងប ោះមាន ដបចនោះរតវនង

។ ដបចនោះរគបចននគត បលើកមលងមតករណ

បចញ អាចសរបសរជារាងខាងបលើបាន ដបចនោះចននផលបកខសៗោន មានចនន ។

. បយើងនងបងហា ញថា បគមនអាចបមបកសណខាងបលើជាពរដចបរៀបរាបបទ។ សនមតផទយបៅវញ

ថាបគអាចបមបកវាជាពរបាន បោយសនមតថា ផលគណននបណាត ធាតរបសសណមយបសមើ ប ើយនងផលគណននបណាត ធាតរបសសណមយបទៀតបសមើ ។ កនងចបណាមចននគតចនន បនតប ទ បោន បយើងបយើងអាចមាន២ករណគ ករណមានចននមយគតកនងចបណាមប ោះមដលមចកោចនង នងករណមយបទៀត ោម នចននណាមយមចកោចនង បទ។ ករណទ១៖ មានមតមយគតកនងចបណាម មដលមចកោចនង ។ កនងករណបនោះមានមត Aឬ B មយបាបណាណ ោះមដលមចកោចនង ។ ឧទា រណ បបើ មចកោចនង ប ោះ មចកមនោចនង បទ ដបចនោះ A មនអាចបសមើ B បទ។ ករណទ២៖ រគបធាតទាងអសសទធមតបឋមនង ។ កនងករណបនោះ បយើងមាន

មតបបើ ប ោះ សមមលខាងបលើបៅជា ។ ចននកាបរសមមលនង ដបចនោះ មនអាច។

. ឧបមាថា ជាចននប លកនងរបអបបលខ ។ បយើងមាន

។ ចននបនសគប លកនងរបអប មានចនន ។ ដបចនោះ ជាសរបបសមើនង

Page 117: Problem Book in Mathematical- Vol I Arithmetic

លម សវណណ វចតរ| ឡសច 105

មដលបយើងចងឱយវាតចបផត។ បយើងរមលកប លតាមវធដចតបៅបនោះបដើមបចននប

នសគប លថយចោះរ តដលតចបផត។

បបើមាន មដល ប ោះ បោយដកប លមយបចញពរបអបបលខ ប ើយយកបៅោកកនងរបអបបលខ បយើងទាញបាន ចននគប លថយចោះ

បនតប ទ បមកបទៀត បយើងបនថយចននគប លតាមរបបៀបមបបបនោះ បោយបធវ ើយាងណាឱយចននប លកនងរបអបពរបផសងោន មានចននប លបរចើនជាងោន មនបលើសពមយបទ។ ដបចនោះ ជាបញចប របអបនមយមានប លចនន

ប ល។ បៅខណៈប ោះ ចននបនសគប លបៅតចបផត។

Page 118: Problem Book in Mathematical- Vol I Arithmetic

106 ឡសច |លម សវណណ វចតរ

Page 119: Problem Book in Mathematical- Vol I Arithmetic

លម សវណណ វចតរ| តរពនធរបារ 107

បរពនធរបារ.ចននគត.ភាព

ចចកដាច

តរពនធរបារ

32. យ ើងមាន

តាង ជាផលបកយលខខទងនម ៗរបស ។ តាមរទសតបទននភាពចចកដាចនង យ ើងទាញបាន

យ ើងមាន

មានន ថា មានយាងយរចើន ខទង។ ដយចនេះ ផលបក តយលខខទងនម ៗរបស យាងយរចើនយសមើ ដយចនេះ ។ កនងចយោមចននគតធមមជាតទាងអស ចដល , ចននចដលមានផលបកតយលខននខទងនម ៗរបសវាធបផត គ ដយចនេះ ។ កនងចយោមចននគតធមមជាតទាងអស ចដល , ចននចដលមានផលបកតយលខននខទងនម ៗរបសវាធបផត គ ចដលផលបកតយលខយសមើ ។ ដយចនេះ ផលបកតយលខរបស យាងយរចើនយសមើ ។ ចតយដា យ េះមានន ថា ។ 33. យ ើងមាន

Page 120: Problem Book in Mathematical- Vol I Arithmetic

108 តរពនធរបារ |លម សវណណ វចតរ

ឬតាមរយបៀបម ចបបយទៀត យដា យ េះ តាម

រទសតបទអលែ យ ើងទាញបាន ។

ដយចនេះ

យ ើងទាញបានយលខពរខទងចងយគគ ។ តរសតររអលល

បរើ ប ោះ ។

លែល ជាចននននចននគរវជជមានលែលធមនបលើសព នងរឋមនង ។

តរសតររ 6.5 បរើ

ជាផលគណកតតត រឋមររស ប ោះ

34. យ ើងមាន ។ ដយចនេះ តាមរទសតបទអលែ

។ យ ើងមាន ដយចនេះ ។ យ ើងមាន ។ ដយចនេះ

មានន ថា ចយ េះចននគត ម ។ យ ើងទាញបាន

មានន ថា យលខ២ខទងចងយគគ ។ 35. តាងចននយ េះមនលបយលខខាងចងយោលយដា ចដល ជាចននគតធមមជាត យ ើ អាចមានយលខោបពម ខទងយ ើងយៅ នង ។ តាមសមមតកមម យ ើងមាន

ចដល m ជាចននគតធមមជាត។ ដយចនេះ

ជាចននគត។ ដយចនេះ ។

យបើ យ េះ រគបចននគត ចចកដាច ។ យបើ យ េះ យ ើងទាញបាន ចននយ េះគ ។ យបើ យ េះ ឬ យ ើងទាញបានចននយ េះគ នង ។

Page 121: Problem Book in Mathematical- Vol I Arithmetic

លម សវណណ វចតរ| តរពនធរបារ 109

តាមរយបៀបដចយនេះ យ ើងទាញបាន ចននចដលរតវរកមាន៖ នងយលខជាព គណនន ។ 36. សនមតថា ចននទាងយ េះមាន ខទង។ ដយចនេះចននយនេះអាចសរយសរជា ចដល ជាយលខមាន ខទង (វាអាចយផតើមយដា យលខសនយម ឬយរចើនជាងយនេះ)។ តាមសមមតកមម យ ើងមាន

ចយ េះ យ ើងទាញបាន មនចមនជាចននគត។ ចយ េះ យ ើងទាញបាន មានន ថា មានរាង (មានយលខសនយចនន )។ យ ើងទាញបាន ចននចដលរតវរកមានរាង

37. សនមតថា មានរាង

តាង ជាផលគណតយលខខទងនម ៗរបស ដយចនេះ

យ ើងមាន

ដយចនេះ យដា អងគទាងពរយសមើគនន យពល មានយលខម ខទង ។ ដយចនេះ ។ យ ើងមាន ចយ េះ ។ ដយចនេះ ។ 38. យ ើងយឃើញថា

នង ។ យបើ ចចកមនដាចនង យ ើ នងនង

យ េះរបភាគយនេះជារបភាគសរមលរចយ ើ ។ ចននចដលជាព គណនន យ ើ តចជាងឬយសមើ មានចនន ។ ចននចដលជាព គណនន មានចនន ។ ចននចដលជាព គណនន ផងនង ផង មានចនន

ដយចនេះ ចននចដលជាព គណនន ឬ មានចនន ។

Page 122: Problem Book in Mathematical- Vol I Arithmetic

110 តរពនធរបារ |លម សវណណ វចតរ

ចននចដលមនចមនជាព គណនន នងនន មានចនន ។ របភាគមានរាង

ចដល នង ឋតយៅកនងសណ ។ យគមានរបភាគចបបយនេះចនន ចប

ប។ គរសមាគ លថា យ ើងមន ករបភាគមានរាង

ចដល នង យទ យរ េះរបភាគអសយនេះ ធជាង

ចដលមនចមនជាធាតរបស ។ ដយចនេះចននភាគ កខសៗគនន សរប យៅកនងសណយនេះយរោ សរមលរច មានចនន ។ 39. តាង ជាចននគតវជ ជមាន ចដលមាន ខទង។ តាង ។ ដយចនេះរគប ចននគតតគនន សទធចតោបយផតើមយដា យលខ នងមានម កនងចយោមយ េះចចកដាចនង ។ ដយចនេះ ចននយ េះជាព គណនន យ ើ មានរគបតយលខព ដល ។ 40. យៅកនងយលខចដលឱយមកយនេះ យគមានចននគតវជ ជមានមាន ខទងចនន ។ ចននខទងសរបរបសយលខយនេះយពលយគ ករតមយលខមាន ខទងយាងយរចើន មកយរៀប កណតយដា

យដា

យ ើងទាញបាន ។ ដយចនេះ

នង

។ ដយចនេះ មានន ថា ចននខទងរបសយលខយនេះបយងកើតយដា យរបើរតមយលខមាន ខទង, មានតចជាង ខទង យ ើ យដា យរបើរតមយលខមាន ខទង, មានយរចើនជាង ខទង។ ដយចនេះ ទតាងខទងទ ឋតយៅរតម កយលខមាន ខទងមកយរបើ ដយចនេះ ។ 41. យ ើងយឃើញថា ។ ដយចនេះ យ ើងគណ ចដល

។ យ ើងមាន

Page 123: Problem Book in Mathematical- Vol I Arithmetic

លម សវណណ វចតរ| តរពនធរបារ 111

ដយចនេះ ។ 42. យ ើងរតវសរយសរ

យ ើងមាន

យ ើងទាញបាន

43. យ ើងដងថា ចដល ។ យ ើងមាន

ដយចនេះ ចននចដលរតវរកគ ។ 44. យ ើងមាន ។ ដយចនេះ តចបផត យពល នង តចបផត។ យ ើងមាន ចចកដាចនង យរ េះយលខបខទងខាងចងរបស នង មានតនមលដចគនន ។ យដា កតាត ទពរជាចននយសស យ េះ រតវចតចចកដាច ដយចនេះ ។ ដចគនន រតវចតចចកដាច ។ ដយចនេះ មានចននគតវជ ជមានតចបផត ម ចដល

យដា យ ើ តាមរទសតបទអលែ ។ ដយចនេះ ។

Page 124: Problem Book in Mathematical- Vol I Arithmetic

112 ចននគរ |លម សវណណ វចតរ

យដា ជាព គណនន ។ យដា យ េះ ឱយ ។ យ ើងពនតយករណនម ៗ។ ករណ យ ើងមាន

មនយផទៀងផទទ ត មានន ថា ។ ដចគនន ចដរ

មានន ថា ។ ដយចនេះមានចត ។ យដា ជាចននគតវជ ជមានតចបផតចដល យ េះ យ ើង ក នង មានន ថា ។ យ ើងទាញបាន ។

45. យ ើងដងថា

ជាចននគតគ។ យ ើងមាន

(យរ េះយបើ

)

ដង

ខទងដបងយរោ យកបៀសរបស សទធចតជាយលខ ។

ចននគរ

46. យ ើងមាន

យដា នង ជាចននបឋមរវាងគនន យ ើ យដា ចផនកខាងសត របសសមភាពខាងយលើជាចននគត យ េះ វារតវចត ចចកដាច

47. យ ើងដងថា យបើ យ េះ ។ ដយចនេះតាមកយណើ នយ ើងទាញបាន

Page 125: Problem Book in Mathematical- Vol I Arithmetic

លម សវណណ វចតរ| ចននគរ 113

តាមរទសតបទឌបា លញាក ចយ េះចននបឋម ម សវ គណ ធបផតនន ចដល ចចកដាច កណតយដា

សវ គណ ធបផតនន ចដល ចចកដាច កណតយដា

យដា

យ េះ សវ គណធបផត ននចននបឋម ោម ចដល ចចកដាចភាគចបងរបសរបភាគ

មានតនមលតចជាងឬយសមើ សវ គណធបផត ននចននបឋម ោម ចដល ចចកដាចភាគ ករបសរបភាគ។ មានន ថារបភាគយនេះជាចននគត។ 48. យ ើងកណត ក ម ។ តាង ជាចននគតធបផតចដល នង តាង ជាផលគណននរគបចននគតយសសធមនយលើសព ។ យ ើងមាន

ជាផលបក ចដលតនម ៗជាចននគតទាងអស យលើកចលងចត

។ ដយចនេះ មនអាច

ជាចននគតយទ មនអាចជាចននគតយទ។ តរសតររ

បរើផលគណកតតត រឋមររស មានរាង

ប ោះ បយើងមាន

លែល ជាចននរលចកវជជមានររស ។

49. រងវវ សមកនងរបសព យោណន តម ចដលមាន រជង យសមើនង

។ ដយចនេះមានន ថា រតវចតចចកដាច ។ យដា យ េះ យបើ បឋមនង យ េះ បឋម

Page 126: Problem Book in Mathematical- Vol I Arithmetic

114 ភាពលចកដាច |លម សវណណ វចតរ

នង ចដរ។ យ ើងមាន មានតចចកចនន យ េះ ចយមលើ គ យរ េះ ដយចនេះយ ើងមនគតតចចកយលខ នងយលខ ។

ភាពលចកដាច

50. យ ើងមាន

ជនស យៅកនង យៅកនងរបមនត យ ើ យដា ដងថា យរ េះ យសស យ ើងទាញបានរបមនតចដលរតវរស បញជជ ក។ 51. យ ើងមាន ចចកដាចនង ចចកដាចនង …………………………………………………. ចចកដាចនង ដយចនេះ ផលបកចចកដាចនង ។ 52. ក ដយចនេះ យសស ។ យ ើងទាញបាន ចចកដាចនង ចចកដាចនង

… ដយចនេះសយណើ ពត។ 53. យ ើងមាន

យដា ឬ ។ ដយចនេះ ។ ចតយដា ជាចននគតវជ ជមាន ដយចនេះ ។ 54. យ ើងមាន

Page 127: Problem Book in Mathematical- Vol I Arithmetic

លម សវណណ វចតរ| ភាពលចកដាច 115

ដយចនេះ មានន ថា យបើ យ េះ ។ 55. យ ើងមាន

ដយចនេះ យបើ ចចកដាចនង យ េះ ចចកដាចនង ចដរ។ រោសមកវញ យដា

យ េះ យបើ ចចកដាចនង យ េះ ចចកដាចនង ចដរ។ 56. យ ើងមាន

យដា យ េះ

ជាចននគត។ ចននគតចដលធជាង យ ើ តច

ជាង គ ។ ដយចនេះ

57. យ ើងដងថា ផលយធៀប

ឋតយៅចយ ល េះ នង ដាចខាត យដា វាជាចននគត យ េះវារតវ

ចតយសមើ ។ ដយចនេះ យ ើងរតវយដាេះរស សមោរ យ ើងទាញបាន ។ ចននយនេះមនចមនជាចននគតវជ ជមានយទ ដយចនេះ គនម នចននគត យទ ចដលយផទៀងផទទ តយទ។ 58. យ ើងមាន

ក យ ើងទាញបាន ចចកដាចនង ។ 59. ចននគតទាងអសអាចមានរាងម កនងចយោមរាងទាង ៖ ។ យបើ ជាចននបឋម យ េះវារតវចតមានរាង (យរ េះយផេងយទៀត ចចកដាចនង ឬនង )។

យបើ ជាចននគ យ េះ ។

Page 128: Problem Book in Mathematical- Vol I Arithmetic

116 ភាពលចកដាច |លម សវណណ វចតរ

យបើ ជាចននយសស យ េះ ជាចននគ ។ 60. យបើ យ េះសយណើ ពត។ សនមតថា ។ យ ើងមាន

រគបតទាងអសរបសផលបកយនេះ ចចកដាចនង ។ ដយចនេះផលបកចចកដាចនង ។ 61. ជាដបង យ ើងសនមតថា បោត ចននគតបនតប ទ បគនន ជាចននគតវជ ជមាន។ យដា ដងថា យមគណយទវធា ជាចននគត

ដយចនេះ មានន ថា ផលគណ ចចកដាចនង ។ យបើមានម កនងចយោមបោត ចននគតបនតប ទ បគនន ទាងអសយនេះ យសមើ យ េះផលគណយសមើ យ ើ ចចកដាចនង ។ យបើបោត ចននគតបនតប ទ បគនន ទាងអសយនេះ សទធចតជាចននអវជ ជមាន យ េះ ផលគណយ េះយសមើនងផលគណករណវជ ជមាន គណនង ។ ដយចនេះ វាកចចកដាចនង ចដរ។ 62. យ ើងមាន

មានន ថា ជាផលគណននចននគតបតយរៀងគនន ។ កនងចយោមចននទាងយ េះ ដាចខាតរតវចតមាន

ម ជាព គណនន យរ េះចននគតបតយរៀងគនន រតវឋតយៅកនងចយោមចននចដលមានរាង ។ យបើយផដើមយដា យ េះយ ើងមានចននគ ។ យបើយផដើមយដា យលខយសស យ ើងមានចននគ ។ ដយចនេះយាងោករតវចតចននគម ចដរ។

នងម យទៀតជាព គណនន យរ េះចននគតបតយរៀងគនន រតវឋតយៅកនងចយោមចននចដលមានរាង ។ យបើយផដើមយដា យ េះយ ើងមានចននព គណនន មាន ។ យបើយផដើមយដា យ េះយ ើងមានចននព គណនន មាន ។

Page 129: Problem Book in Mathematical- Vol I Arithmetic

លម សវណណ វចតរ| ភាពលចកដាច 117

ដយចនេះ ជាព គណនន ផង នងនន ផង។ យដា បឋមនង ដយចនេះ ជាព គណនន ។ 63. យ ើងមាន

យដា ជាចននបឋម យ េះ ជាចននយសស។ ដយចនេះ នង គទាងពរ ដយចនេះផលគណរបសវាជាព គណនន ។ យដា ដយចនេះ មនអាចជាព គណនន យទ។ ដយចនេះ រតវមានរាង ឬ ។ យបើ យ េះ ។ យបើ យ េះ ។ ដយចនេះរតវចតមានម កនងចយោម ឬ ជាព គណនន ។ ដយចនេះ ជាព គណនន ។ ដយចនេះ ជាព គណនន ផងនង នន ផង យ ើ យដា បឋមនង យ េះ ជាព គណនន ។ 64. យ ើងមាន ជាចននគ ជានចចយរ េះ កនងចយោម នង រតវចតមានម ជាចននគ យរ េះយបើមានម កនងចយោម គ យ េះ គ, យបើ យសសទាងពរយ េះ គ។ យបើមានម កនងចយោមចននគតទាងប មានរាង យ េះ យបើយ ើង ក នង យដា ជាចននគ យ េះវារតវមានរាង ដយចនេះចចកដាចនង ។ យបើចននគតទាងបសទធចតចចកមនដាចនង យ េះ ពកវាឋតយៅកនងចយោមរកមចននមានរាង ឬ ។ យដា យគមានចននគតប យ េះយគរតវចតមានចននគតពរចដលសថតយៅកនងរកមចតម (រកម ចដលមានរាង ឬ )។ ផលបករយបើមនអចងយទ ផលសងននចននពរយ េះ រតវចតចចកដាចនង ។ យដា ជាចននគ យ េះវារតវមានរាង ដយចនេះចចកដាចនង ។ 65. យបើ យ េះ មនអាចចចកដាចនង យទ។ យបើ ឬ យ េះ មនអាចចចកដាចនង យទ។ ដយចនេះ មានចត នង ចចកដាចនង ទាងពរ យទើប ។

Page 130: Problem Book in Mathematical- Vol I Arithmetic

118 ភាពលចកដាច |លម សវណណ វចតរ

66. ជាដបងយ ើងបងវា ញថា យបើ ចចកដាចនង យ េះ ចចកមនដាចនង ។ ឧបមាថា ចចកដាច ។ យដា ជាចននបឋម យ េះ ឬ រតវចតចចកដាចនង ។ កនងករណទាងពរ ចចកមនដាចនង ។ ដយចនេះ យបើ ចចកដាចនង យ េះ ចចកមនដាចនង ។ រោសមកវញ យ ើងបងវា ញថា យបើ ចចកមនដាចនង យ េះ ចចកដាចនង ។ យដា ចចកមនដាចនង យ េះ ឬ ។ យបើ យ េះ

ចចកដាចនង ។ យបើ យ េះ ចចកដាចនង ។ ដយចនេះ យបើ ចចកមនដាចនង យ េះ ចចកដាចនង ។ 67. យ ើងមាន ចចកដាចនង ចចកដាចនង មានន ថា ចចកដាចនង ។ ដចគនន ចដរ ចចកដាចនង ចចកដាចនង មានន ថា កយនោមខាងយលើចចកដាចនង ។ យដា នង មនមានកតាត បឋមរមគនន យ េះ កយនោមខាងយលើចចកដាចនង ។ 68. ករណ យ ើងមាន ជាព គណនន ពត។ សនមតថាវាពត រ តដល មានន ថា ចយ េះចននគត ោម ។ យ ើងមាន

ព គណនន ពត។ ដយចនេះ ជាព គណនន ចយ េះរគបចននគតធមមជាត ។

Page 131: Problem Book in Mathematical- Vol I Arithmetic

លម សវណណ វចតរ| ភាពលចកដាច 119

69. ករណ

យដា ជាចននយសស យ េះ ចដល ជាចននគតធមមជាត។ ដយចនេះ

យបើ គ យ េះ ចចកដាចនង យ ើ ចចកដាចនង ។ ដយចនេះ ចចកដាចនង ។ យបើ យសស តាង យ េះ ចចកដាចនង យ ើ ចចកដាចនង ដយចនេះ ចចកដាចនង ។ ដយចនេះ ករណ សយណើ ពត។ ឧបមាថា សយណើ ពតដល មានន ថា ចចកដាច ។ យ ើងរតវបងវា ញថា

ចចកដាចនង ។ យ ើងមាន

យដា ចចកដាចនង យ េះ យ ើងនងបងវា ញថា ចចកដាចនង ។ យដា យសស យ េះ គ ដយចនេះចចកដាចនង ។ សយណើ ពត។ 70. យ ើងមាន

តាមសមភាពយនេះ យ ើងទាញបានថា i) យបើ ចចកដាច យ េះ ចចកដាច ii) យបើ ជាចននព គណ យ េះ កជាចននព គណចដរ។ យ ើងនងបងវា ញថា មាន យរចើនរាបមនអសចដលចចកដាច ។ យ ើង ក ដយចនេះយ ើងទាញបាន ចចកដាច ។ យ ើងរតវបងវា ញថា ចចកដាច យទៀត។ យ ើងនងបងវា ញតាមវោរយដា កយណើ ន។ យបើ យ េះ ចចកដាច ពត។ សនមតថា ពតចយ េះចននគត ម ។ យ ើងមាន

យ ើងមាន ចចកដាចនង តាមកយន ើន យ ើ ជាចននគ ដយចនេះ ចចកដាចនង ដយចនេះសយណើ ពត។ ដយចនេះ យ ើងបានបងវា ញថា មាន យរចើនរាបមនអសចដល ចចកដាច ។

Page 132: Problem Book in Mathematical- Vol I Arithmetic

120 ភាពលចកដាច |លម សវណណ វចតរ

។ យដា ជាចននព គណ យ េះ កជាចននព គណចដរ(តាម ii))។ យដា ចចកដាច យ េះ ចចកដាច (តាម i))។ ដយចនេះជាសរប យ ើងបានបងវា ញថា មាន

ចដល យរចើនរាបមនអស ចដល ចចកដាច ។ 71. យ ើងយឃើញថា នង រតវចតមានយលខ ខទង។ តាង នង

។ ដយចនេះ លេះរតាចត មាន ម ចដល យ ើ ចដល

កនងផលបកយលើ យបើ មានន ថា គនម នត យទ។

យបើ យ េះ មានចត ។

យដា បក ផលបកទាងយនេះបញចលគនន យ ើងទាញបាន

យដា ជាចមាល សរបស យ េះ យ ើងទាញបាន

ដយចនេះ រតវចតជាចននយសស ។ ដយចនេះ

ចចកដាចនង ។ 72. តាមរទសតបទននភាពចចកដាចនង ចននយនេះ ចចកដាចនង យបើ

ចចកដាចនង នង រោសមកវញ។ ចតចននយនេះ ចចកដាចនង ចតចចកមនដាចនង យទ។ ដយចនេះ ធបផត ចដល ចចកដាចចននយនេះគ ។ 73. យ ើងមាន

ចយ េះ យ ើងមាន

Page 133: Problem Book in Mathematical- Vol I Arithmetic

លម សវណណ វចតរ| ភាពលចកដាច 121

ដយចនេះ ឱយ ។ យដា យ េះ ចចកមនដាច ។ ដយចនេះរតវចត

យ ើងទាញបាន ចចកដាចរគបតបកទាងអសកនង (*) ដយចនេះ រតវចចកដាច ។ 74. យ ើងយរៀបផលបកជា

យ ើងមាន

ដយចនេះ ផលបករបសរបភាគយនេះ មានកតាត រមរបសភាគ កយសមើ ។ តនម ៗននផលបកខាងយលើមាន ភាគចបង ចដលមានកតាត ផលគណដចជា សទធចតតចជាង ។ យដា ជាចននបឋម យ េះ គនម នកតាតោម របសភាគចបងអាចសរមល បានយទ។ ដយចនេះ យៅកនងភាគ កយៅរកោដចដល យរោ បករបភាគទាងអសបញចលគនន មក។ ដយចនេះ ចចក ដាច។ 75. ចចកដាច យ េះ មាន ចដល ។ ដយចនេះ ។ យបើ ជាតចចករមធបផតរវាង នង យ េះ ចដល បឋមនង ។ ដយចនេះ

។ អងគខាងយឆវងននសមភាពយនេះ ជាចននគត ដយចនេះមានចត

។ ដយចនេះ នង ជាចននបឋមនងគនន ។ តាមរយបៀបដចគនន យ ើងទាញបាន បឋមនងគននពរៗ។

Page 134: Problem Book in Mathematical- Vol I Arithmetic

122 ភាពលចកដាច |លម សវណណ វចតរ

យបើ ចចកដាច យ េះ វាចចកដាច ។ ដចគនន នង រតវចតចចកដាច ។ យដា បឋមនងគនន ពរៗ យ េះយ ើងទាញបាន ចចកដាច ។ យ ើងមាន

យដា

ជាចននគត យ េះ

។ ដយចនេះ

ជាដបងយ ើងសនមតថា ។ យ ើងទាញបាន

យដា ។ សមោរ (*)យៅជា

យដា ។ យបើ យ េះ

សមោរគនម នចយលើ ។ យបើ យ េះ

យ ើងយផទៀងផទទ តយឃើញថា ជាចយមលើ របសសមោរ។ ដយចនេះជាសរបសមោរមានចយមលើ នង ចមាល សរបសវា ចដលមាន ។

Page 135: Problem Book in Mathematical- Vol I Arithmetic

លម សវណណ វចតរ| ភាពលចកដាច 123

76. សនមត ជាគចយមលើ ចដលរតវរក ចដល

ជាចននគត។ ដយចនេះ

ជាចននគត

កជាចននគតចដរ។ មានន ថា យបើ ជាចយមលើ យ េះ កជា

ចយមលើ ចដរ។ ដយចនេះជាដបងយ ើងយរៀប សន។

តាង

។ យ ើងទាញបាន

ចចក ដាច។ ចយ េះចននគត ោម ។ ចយ េះ

យបើ យ េះ

យរ េះ

ចចក ដាច។ យដា ដយចនេះ មានចត ។ យបើ យ េះ

យ ើង ក (យរ េះយ ើងបានសនមតថា )។ យបើ យ េះ

Page 136: Problem Book in Mathematical- Vol I Arithmetic

124 ភាពលចកដាច |លម សវណណ វចតរ

យ ើងទាញបាន ។ ដយចនេះជាសរបចយមលើ គ នង ចមាល សរបសវា ។ 77. តាង ។

យបើ យ េះ

មនពត។

យបើ យ េះ មនពត។

ដយចនេះ មានចត ។ ដយចនេះ

ចដល ។

រោសមកវញ យបើ

យ េះ

ចចកដាចនង ។

ដយចនេះ ចយ េះ

យ េះ ចចកដាច ។

78. ដបងយ ើងពនតយករណ ។ ដយចនេះ មានចននគត ចដល ។ យដា យ េះ ។ យដា យសស យ េះ គ។ តាង ។ យដា នង យ េះ យ ើ យផទៀងផទទ ត យ ើ ចចកដាច យរ េះ គ។

មតងយនេះយ ើងសនមតថា ។ ក យ េះ

ចតយដា ជាចននបឋម យ េះ ។ យដា យ េះ ។ តាង ។ យ ើងមាន

Page 137: Problem Book in Mathematical- Vol I Arithmetic

លម សវណណ វចតរ| ភាពលចកដាច 125

យរ េះ ជាចននបឋម នង

តាង ។ យ ើងមាន ។ យ ើងមាន នង យរ េះ ។ ដយចនេះ ចចក ដាច។ 79. សនមតថា ចចកដាចនងរគបចននគត ។ តាង ជាបោត ចនន

បឋម យ ើ តាង ជាចននគតម ចដល

។ ដយចនេះ

តាង

តាមសមមតកមម រតវចចកដាចនងរគបចននគត ចដល ដយចនេះ រតវចចកដាចនង

។ ដយចនេះ មានន ថា

។ ដយចនេះ ជាបោដចននបឋមចដល ។ យដា ។ យ ើងពនតយចននគតមដងម ៗ ព ដល យ ើងយឃើញថា យផទៀងផទទ តចយ េះ ។ ដយចនេះ ជាចននធបផតចដលចងបាន។ 80. យ ើងមាន

កនងចយោម នង រតវមានម គ នងម យទៀតយសស។ កតាត ចដលគ មានតនមល ។ តាង ចដល ជាចននយសស។ យបើ យ េះ ចដលមានកតាត យលខគមានតនមលតចជាង មនអាចចចកដាចនង យទ។ ដយចនេះ ចចកមនដាចនង ។ ចតយបើ យ េះ ចចកដាចនង ចយ េះ ចដល

Page 138: Problem Book in Mathematical- Vol I Arithmetic

126 ភាពលចកដាច |លម សវណណ វចតរ

ដយចនេះកនងករណយនេះ មានតនមល ចដល យ ើ ចចកដាចនង ចដលផទ ពលកខខណឌ ។ 81. យ ើងមាន

យបើ ជាចននយសស យ េះ ជាផលបកននចននយសសចនន ដយចនេះផលបកយនេះជាចននយសសចដរ។ ដយចនេះ មនអាចជាព គណនន បានយទ។ ដយចនេះមានចត ។ យបើ ជាចននគ តាង ។ លកខខណឌ យៅជា ចចកដាច ។ ចននទាងពរគ នង មានផលសងយសមើ ដយចនេះ របសចននពរយនេះ ជាតចចករបស ។ មាាងវញយទៀតចននទាងយនេះជាចននគ ដយចនេះ ។ យ ើងទាញបានថា យបើមន ក ចដរ ចដលចចកដាចនង ។ ចតរតវចត ។ វសមភាពយនេះមនពតយទ យបើ ។ យ ើងយផទៀងផទទ តយដា នដយឃើញថា ជាចយមលើ ។ ដយចនេះចយមលើ គ ។ 82. តាង

យដា ចចកដាច យ េះ ជាចននគត។ ដយចនេះ ។ យបើ យ េះ

មនអាច។ ដយចនេះ ។ យដា ដងថា ចយ េះរគបចននពត យ ើងមាន

។ ដយចនេះ ចយ េះ យ ើងមាន

យរ េះ យបើ ចចកដាចនង យ េះរតវចត

ចដល ។ ចតយនេះ ដយចនេះ មនអាចចចកដាចនង បានយទ។ ដយចនេះ ។ សនមតថា ។ ដយចនេះ ជាចននគ មានន ថា ជាចននយសស ដយចនេះ រតវចតយសស ដយចនេះ ។ សមោរយៅជា

Page 139: Problem Book in Mathematical- Vol I Arithmetic

លម សវណណ វចតរ| ភាពលចកដាច 127

យពលយនេះ សនមតថា ។ យបើ

យដា ដយចនេះ

យ ើងមាន យពល ។ ដចយនេះមានចត ។ ករណយនេះ

យបើ

យដា ដយចនេះគនម នចយមលើ ។ យបើ

ចត ដយចនេះគនម នចយមលើ យទៀត។

ជាសរប ចយមលើ មានចត នង ។ 83. តាង យ េះ យសស។ យ ើងរតវបងវា ញថា ចចកដាច ។ យ ើងមាន ដយចនេះ ។

តាមរទសតបទអលែ យ ើងទាញបាន ។

ដចគនន យ ើងទាញបាន ។ យដា ជាចននយសស យ េះ

ដយចនេះ ។ 84. យ ើងមាន

Page 140: Problem Book in Mathematical- Vol I Arithmetic

128 ភាពលចកដាច |លម សវណណ វចតរ

តាមសមមតកមម (i) នង (ii) យ ើងទាញបានសមមតកមមថម (i) ចចកមនដាចនង (ii) ចចកដាចនង យដា ។ យដា សកយលខមតងម ៗ យ ើងទាញបាន ជាចយមលើ ម យរ េះ ។

85. យ ើងមាន

យបើ យ េះ ។ យបើ យ ើ យដា ជាចននបឋម យ េះ ។ ដយចនេះ តាមរទសត

បទលែមា យ ើងទាញបាន ។ ដយចនេះ

។ ដយចនេះ ចយ េះរគប ។

86. យ ើងមាន

យដា ជាចននបឋម យ ើ យ េះ តាមរទសតបទលែមា , ចចកដាច

។ យដា យ ើ

ជាចននគត យ េះ ចចកដាច

87. តាង ។ តាង

យ ើងសយងកតយឃើញថា

Page 141: Problem Book in Mathematical- Vol I Arithmetic

លម សវណណ វចតរ| ភាពលចកដាច 129

តាមរទសតបទលែមា ចយ េះចននបឋម ម យ ើងទាញបាន

ដយចនេះ (*) យ ើងទាញបាន

យដា ទាងអសយនេះសទធចតជាចននបឋម យ េះយ ើងទាញបាន ។

88. ចយ េះចននបឋមយសស ោម តាមរទសដបទលែមា យ ើងមាន ។

យ ើង ក យ េះ

89. យបើ ចយ េះចននគត ោម យ េះ រតវចតជាចននយសស នង មានតចចកបឋមយសស តចបផត។ យ ើងមាន ចចកដាច យ ើ ចចកដាច

។ តាង ជាចននគតតចបផត ចដល ។ តាង ។ ដយចនេះ

យបើ យ េះ ដយចនេះយ ើងមាន ចដល ផទ ពសនមត ចដល ថា តចជាងយគ។ ដយចនេះ មានន ថា ចចកដាច ។ យដា ជាចននយសស ដយចនេះ កយសសចដរ។

Page 142: Problem Book in Mathematical- Vol I Arithmetic

130 សណល. ភាពសមមល |លម សវណណ វចតរ

តាមរទសតបទលែមា យ ើងមាន ។ យដា ជាចននតចជាងយគចដល

យ េះ ។ ដយចនេះ ។ ផទ ពសនមតចដល ជាតចចកតចជាងយគរបស ។

90. តាមរទសតបទឌបា ែញា ក តនមល ធបផត ចដល ចចកដាច យសមើនង

ដចគនន ចដរ តនមល ធបផត ចដល ចចកដាច យសមើនង ។ យដា

ដយចនេះ តនមល ធបផត ចដល ចចកដាច

យសមើនង ។ ដយចនេះ ចចកមនដាច

យទ។

91. រគបចននគតកនងចយោមចននគតទាង យ េះ សទធចតអាចសរយសរជារាង បានទាងអស ចដល ជាចននយសស។ ឧទា រណ

។ ចយ ល េះ នង , ចននយសសមានចននចត ចតបាយោណ េះ មានន ថាយ ើងមានចននយសស យផេងគនន ចននចត ចតបាយោណ េះ គ ។ ដយចនេះ កនងចយោមចននគតទាង ចដលយគយរជើសយរ ើសយ េះ រតវចតមានចននគតពរ ចដលមាន មានតនមលយសមើគនន ។ តាងចននពរយ េះយដា នង ចដល ។ ដយចនេះ ករណយនេះ ចចកដាច ។

សណល. ភាពសមមល

92. i) យ ើងមាន

Page 143: Problem Book in Mathematical- Vol I Arithmetic

លម សវណណ វចតរ| ចននកាបរ 131

ii) យ ើងមាន ។ សយណើ ពត។

93. យ ើងមាន ។ តាមរទសតបទលែមា យ ើងមាន នង

។ រគបចននគតវជ ជមានយសសសទធចតបឋមនង ។ យ ើងមាន ។ តាមរទសត

បទអលែ យ ើងទាញបាន ។ ដយចនេះ

ចយ េះ នង ។ យដា នង បឋមនងគនន ពរៗ យ េះ យអា ។

តរសតររលភមាា

តតង ជាចននរឋមនងសនមរថា លចក មនដាច ប ោះ ។

94. តាមវធចចកចបបអគលដ

ចដល ជាចននគត។ យ ើងទាញបាន

ដយចនេះ មានន ថា នង ។ យដា ដយចនេះ ។ យដា យ េះ ។ ដយចនេះ ។

ចននកាបរ

95. ចននគតទាងអសអាចមានរាងម កនងចយោម នង ។ យដា យលើកវាជាោយរ យ ើងទាញបាន

Page 144: Problem Book in Mathematical- Vol I Arithmetic

132 ចននកាបរ |លម សវណណ វចតរ

96. ោយរននចននគតម មានរាង ឬ ។ រគបចននទាងអសយៅកនងសវតយនេះ មានរាង

ដយចនេះ វាមនអាចជា ជាោយរននចននគតម យទ។ 97. យបើ យ េះ

ចត យដា ជាចននយសស យ េះ ។ ដយចនេះ ។ 98. យៅកនងយគនល កណតយដា

99. យបើ យ េះ

តាង ។ ដយចនេះ ជាោយរននចននគតឋតយៅចយ ល េះោយរននចននគតពរតគនន ដយចនេះ មានចត ។ ដយចនេះ ។ យបើ យ េះ មនចមនជាចននោយរ។ យបើ យ េះ

Page 145: Problem Book in Mathematical- Vol I Arithmetic

លម សវណណ វចតរ| ចននកាបរ 133

ដយចនេះ យផទៀងផទទ ត ជាចននោយរ។

100. ចយ េះ យ ើងមាន

យបើ ជាចននគ យ េះ ឋតយៅចយ ល េះចននោយរពរតគនន ដយចនេះវាមនអាចជាចននោយរបានយទ។ យបើ ជាចននយសស យ េះចយ ល េះ

នង

មានចតចននគត

ម គត ដយចនេះរតវចត

ដយចនេះ ចយលើ គ ។

101. សនមតថា

ចដល ជាចននគត។ យ ើងរតវបងវា ញថា ជាចននោយរ។ យ ើងសនមត

ថា មានគ ចដលយផទៀងផទទ ត ជាចននគត យរជើសយរ ើស ក ជាគចដលតចជាងយគ។ យ ើងអាចសនមតថា យរ េះ កយនោមខាងយលើមានលកខណៈ សយមរទយធៀបនង ។

យបើ យ េះ

យបើ ជាចននគត យ េះ ជាចននោយរ។ យបើ យ េះ

ជាសមោរដយរកទពរ យធៀបនង ចដលមានផលបកឫសយសមើ នង ផលគណឫសយសមើ (តាមរទសតបទចវយត)។

តាង ជាឫសរបសវា ដយចនេះ នង ។ យដា ជាចននគត យ េះ កជាចននគតចដរ។ យ ើងមាន ជាចននគតវជ ជមាន យ ើ នង ។ មនអាចធជាងសនយបានយទ យរ េះ យបើមនអចងយទ

Page 146: Problem Book in Mathematical- Vol I Arithmetic

134 ចននកាបរ |លម សវណណ វចតរ

មានន ថា មាន ជាចននគតវជ ជមាន(តចជាងឬធជាង កបាន) ចដល មនចមនជាចននគតចដលតចជាងយគ ចដលផទ ពសនមត។ ដយចនេះ ។ ចត

ដយចនេះ នង

ចដល យ ើ ជាចននគត។ ដយចនេះ យអា

ជាចននោយរ។ 102. យ ើងមាន យដា យលើកវសមភាពយនេះជាោយរ យ ើងទាញបាន

បក ចលអងគទាង២ យ ើងទាញបាន ដយចនេះ

យបើ យ េះ យ ើងទាញបាន

មនពត។ ដយចនេះ ។ ដយចនេះ

ដយចនេះសមភាពអាចយកើតមាន យពល ដយចនេះ ។ តាង ជាតចចកបឋមរមរបស នង ។ ដយចនេះ រតវចតចចកដាច យ ើ ចចកដាច មនអាចយទ យបើ ។ ដយចនេះ បឋមនង ។ យដា ផលគណរបសវាជាចននោយរ យ េះ ករតវចតជាចននោយរចដរ។ 103. ក) សនមតថា ជាចននោយរ។ យដា នង បឋមនងគនន យ េះវារតវចតជាចននោយរទាងពរ។ ចតមនចដលមានចននោយរពរខសគនន ម ឯកតាយទ យលើកចលងចត នង យរ េះ ។ ខ) សនមតថា ជាចននោយរ។ យដា នង បឋមនងគនន យ េះវារតវចតជាចននោយរទាងពរ។ ចតយដើមបឱយ ជាោយរននចននគត ទាលចត ចតយពលយ េះ ផលគណ យសមើសនយ។

Page 147: Problem Book in Mathematical- Vol I Arithmetic

លម សវណណ វចតរ| ចននកាបរ 135

គ) អាចជាចននោយរ មានចត

។ 104. ពនតយចននគត ។ យដា នង បឋមរវាងគនន យ េះ យដើមបឱយផលគណចននទាងពរជាចននសវ គណទ យ េះវារតវចត ជាចននសវ គណទ នង ជាចននសវ គណទ ចដរ។ ដយចនេះ

នង ចដល

បឋមនង ។ យ ើងទាញបាន

ចយ េះ នង ចយ េះ នង យ ើងមានកតាត ទពរននអងគខាងយឆវងធជាង ដយចនេះ អងគខាងយឆវងនងសត មនអាចយសមើគនន បានយទ។ 105. តាង ជាចននមនោយរទ ។ យគមានចននគតធមមជាត ចដល ។ មានចននោយរចនន ចដលតចជាង នង ចននមនោយរចនន មករតម ដយចនេះយ ើងយឃើញថា ។ ដយចនេះ

យដា សទធចតជាចននគត យ េះវសមភាពខាងយលើយៅជា

106. តាង មានន ថា ជាចននមនោយរ ឃលល តពចននោយរ ចនន ។ យបើ មានន ថា ជាចននោយរ។

Page 148: Problem Book in Mathematical- Vol I Arithmetic

136 ចននកាបរ |លម សវណណ វចតរ

យ ើងចចក សណនន ជាសណរងពរ ចដលកនងយ េះ

ជាសណនន ចដលធជាងចននោយរចនន ចដល ។

ជាសណនន ចដលធជាងចននោយរចនន ចដល ។ យ ើងសយងកតយឃើញថា ទាងករណ នង យ ើងមាន

សនមតថា ។ យដា យ េះ

ចដល ។ ដយចនេះ ។ ដយចនេះ យ ើងោបយផដើមពករណ ចតមដងកបាន។ ករណយនេះ យ ើងមាន

ដយចនេះ

មានន ថា យរ េះ ។ យ ើងសយងកតយឃើញថា ឃលល តព ចននោយរ ចនន (យរ េះ ) ប ទ បមក ឃលល តពចននោយរ ចនន ។ ដយចនេះ ជហានបនតប ទ បមកយទៀត តនមលចដលយលើសយ េះនងថ ចេះោនចតតចយៅៗ រ តដលយពលម តនមលយនេះយសមើសនយ។ យៅយពលយ េះ យ ើងនងទាញបានចននោយរ។

Page 149: Problem Book in Mathematical- Vol I Arithmetic

លម សវណណ វចតរ| ចននកាបរ 137

107. តាង ជាចយមលើ តចជាងយគបងអសរបសសមោរ

យបើ នង សទធចតគ យ េះ ចចកដាចអងគខាងយឆវង ដយចនេះ វាចចកដាច ដយចនេះ ជាព គណនន ។ ដយចនេះ

ជាចននគត យ ើ កជាចយមលើ របសសមោរចដរ។ ករណយនេះផទ ព ោរសនមត។

ដយចនេះ ឬ ជាចននយសស។ ឧបមាថា គ យ ើ យសស យ េះ

មនអាចជាចននោយរបានយទ យរ េះចននោយរសមមលនង ឬ តាម ។ ដយចនេះ នង រតវចតយសសទាងពរ។ ករណយនេះ

មនអាចជាចននោយរបានយទ។ ដយចនេះ គ ចដល ជាចននោយរ មានចត ម បាយោណ េះ។ 108. ចននចដលជាធាតរបសសណយ េះ មានរាង

ដយចនេះ ធាតនម ៗ មាន ម ចបបមាន ក។ យបើគតពលកខណៈគយសស នន យ ើងមានបនេ ចនន ចបប។ ឧទា រណ (គ,យសស,យសស,គ,យសស) ជាករណម កនងចយោមករណទាង យ េះ។ យដា យ ើងមានចននគតចនន យ េះរតវចតមានចននគតពរ ចដលមាន តយរៀបតាមលកខណៈគយសសដចគនន ។ ដយចនេះផលគណននចននពរយនេះជាចននោយរ។ 109. ចនននម ៗឋតយៅកនងសណ មានរាង

យបើគតតាមលកខណៈគយសស យ ើងមាន ចនន ចបប។ ដយចនេះយបើយ ើងោប ក ធាត យ េះ យ ើងនងទទលបានធាតពរខសគនន ចដលមាន មានលកខណៈគយសសដចគនន ដយចនេះធាតពរយនេះមានផលគណជាចននោយរ។

Page 150: Problem Book in Mathematical- Vol I Arithmetic

138 ចននលែលមានរាងណាមយ |លម សវណណ វចតរ

យដា យ ើងមាន យ េះយ ើងអាចរកបានចននខសគនន ម គ ចដល ។

យ ើងដកម គយនេះយចញ។ ពកនងចយោម យៅសល យ ើងអាចរកបានចននខសគនន ម គ ចដល

។ យ ើងដកម គយនេះយចញ។ ពកនងចយោម យៅសល យ ើងអាចរកបានចននខសគនន ម គ ចដល

។ យ ើងបនតដក ក រ តបាន គ ចដល មានន ថា ។ ដយចនេះ យ ើងអាចរបមលបាន គចននគតខសគនន ចដល

។ ប ទ បមកយទៀតបោត ចននគត ទាង មានតចចកបឋមរបសវាតចជាងឬយសមើ យ ើ យដា យ េះ យ ើងអាចរកបានចននគត ពរខសគនន តាងយដា នង ចដល ។ ដយចនេះ ជាចននសវ គណទ៤។ ដយចនេះមានន ថា យ ើងអាចរកបានចននគតបនចដលមានផលគណរបសជាចននសវ គណទ៤។

ចននលែលមានរាងណាមយ

110. យបើ ជាចននយសស យ េះយ ើង ក ។ យបើ គ យ េះ អាចជា ឬ ។ យបើ យ េះ យ ើង ក ។ ចននពរយនេះបឋមរវាងគនន យរ េះយបើ យ េះ យដា យសស យ េះ ។ យបើ យ េះ យ ើង ក ។ 111. យ ើងមាន នង បឋមនង ។ ដយចនេះ យ ើងរតវរក នង ចដល ចចកដាចនង ។ យ ើងមាន ដចយនេះ ។ យ ើងយផទៀងផទទ តយដា ោរជនសយលខមដងម ៗយឃើញថា ជាចននតចជាងយគចដល ។ ដយចនេះ ទាលចត ចយ េះចននគតវជ ជមាន ោម ។ ដយចនេះ យ ើងរតវរាបចននចយមលើ ជាចននគតវជ ជមាន របសសមោរ

ចដល យ ើ ។ ដយចនេះ ។

Page 151: Problem Book in Mathematical- Vol I Arithmetic

លម សវណណ វចតរ| ចននលែលមានរាងណាមយ 139

ចយ េះ សមោរយៅ ជា ដយចនេះ មាន ចយមលើ ។ ចយ េះ សមោរយៅ ជា ដយចនេះ មាន ចយមលើ ។ ...... ចយ េះ សមោរយៅ ជា ដយចនេះ មាន ចយមលើ ។ ដយចនេះជាសរប មាន ចយមលើ ។ 112. រយបៀបទ១៖តននសវតយនេះអាចសរយសរជា យគនល១០ (១) យគនល៣ (២)

សយនរកនងយគនល២ (៣) យគនល១០ (៤) ទ កទនង(១) ជាតរបសសវតសរយសរកនងយគនល១០។ (២) ជាតរបសសវតសរយសរកនងយគនល៣ ដចជា មានន ថា កនងយគនល១០ យសមើនង កនងយគនល៣។ ទ កទនង(៣) ជា(២) បាចនតោតទកថាសយនរយនេះជាយគនល២។ (៤)ជាតនលរបស(៣)កនងយគនល១០។ ជារមមានន ថា (១)=(២) នង (៣)=(៤) ។ ដយចនេះតទ១០០ននសវតយនេះ ជាយលខ១០០កនង(៤)។ យ ើងទាញរក តនលរតវគនន នន១០០យនេះ កនងទ កទនងទ(៣)។ យ ើងសរយសរ១០០ យៅកនងយគនល២គ យ ើ បចលងវាយៅជាយគនល៣ គ ។ ដយចនេះតទ១០០របសសវតយសមើនង ។ រយបៀបទ២៖យបើយ ើងយរបើចត៦តដបងគ នង យ ើងអាចបយងកើតបានសវតយនេះចនន

Page 152: Problem Book in Mathematical- Vol I Arithmetic

140 ចននលែលមានរាងណាមយ |លម សវណណ វចតរ

តចតបាយោណ េះ។ យ ើងរតវោរយាងតច១០០ត។ សវ គណនន៣ធប ទ បគ ។ ប ទ បពតទាង៦៣ខាងយលើ តធប ទ បជាបនេនន នង ចដលមានទាងអសចនន

ដយចនេះយ ើងមានទាងអសចនន ត។ យដើមបរកតទ១០០ យ ើងរតវបចនថម

យទៀត ដយចនេះតទ គ

។ 113. តាង ជាសយណើ ចដលថា «បោត ចននគត សទធចតជាយលខពយសស»។ យ ើងមាន ពត។ យ ើងនងបងវា ញថា កពតចដរ មានន ថា «បោត ចននគត សទធចតជាយលខពយសស»។ ដយចនេះ យ ើងរគននចតបងវា ញថា នង ជាយលខពយសស ជាបចនថមយទៀតបានយ ើ ។ យ ើងនងបងវា ញថា យបើ ជាយលខពយសស យ េះ នង កជាយលខពយសសចដរ។ យ ើងមាន

ដចគនន ចដរ

ដយចនេះ យបើ ជាយលខពយសស យ េះ នង កជាយលខពយសសចដរ។ 114. ជាដបងយ ើងបងវា ញថា មនអាចសរយសរជារាង បានយទ។ យដើមបរស បញជជ កនវអណេះអោងយនេះ ជាដបងយ ើងសនមតថា អាចសរយសរបានសន មានន ថា

Page 153: Problem Book in Mathematical- Vol I Arithmetic

លម សវណណ វចតរ| ចននលែលមានរាងណាមយ 141

យដា នង បឋមនងគនន ពរៗ យ េះ រតវចចកដាច ។ មនអាចយសមើសនយបានយទ យរ េះ ។ ដយចនេះ ។ ដចគនន យ ើងទាញបានថា នង ។ ចតយបើដយចនេះ យ េះ

ផទ ពោរសនមត។ ដយចនេះ មនអាចសរយសរជារាង បានយទ។

ប ទ បមកយទៀត យ ើងបងវា ញថា រគបចននគតទាងអសចដលធជាង សទធចតអាចសរយសរជារាង បានទាងអស។ យ ើងនងបងវា ញថា រគបចននគតទាងអសចដល ធជាង សទធចតអាចសរយសរជារាង បានទាងអស។ យដា បឋមនង យ េះរគបចននគតទាងអសយពលចចកនង មានសណលម កនងចយោម ។ យបើ យ េះ ចចកនង សល ចដល ។ យដា យ េះ ចដល ។ ដយចនេះមានន ថា រគបចននគតធជាង សទធចតអាចសរយសរជារាង បានទាងអស។

យដា ដងថា នង បឋមនងគនន យ េះរគបចននទាងអសចដលធជាង សទធចតអាចសរយសរជារាង បានទាងអស ចដល នង ជាចននគតវជ ជមាន។ ដយចនេះ

តាង ។ យ ើងមាន ដយចនេះយ ើងអាចសរយសរជារាងខាងយរោមបាន

ចយ េះចននគតវជ ជមាន នង ខលេះ។ ដយចនេះ

យ ើងទាញបាន (*) & (**)

យ ើងមាន យ េះ

Page 154: Problem Book in Mathematical- Vol I Arithmetic

142 ចននលែលមានរាងណាមយ |លម សវណណ វចតរ

យ ើងបានបងវា ញថា រគបចននទាងអសចដលមានរាង

សទធចតអាចសរយសរជារាង បាន។ 115. តាង ជាចននសនទាន។ តាង

។ យ ើង ក

ជាចននសនទាន។ យដា

។ យដា

។ ដយចនេះ

។ មាន

ន ថា រគប

យ ើងមាន ចដល

កនងករណទយៅ យ ើងសនមតថា ។ ដយចនេះ យគមាន ចដល

ដយចនេះ

ជាចននសនទានយៅចយ ល េះ នង ។ តាមលទធផលខាងយលើ យ ើងទាញបាន

ចយ េះចននគតវជ ជមាន នង ោម ។ យ ើងទាញបាន

យ ើងទាញបានថា សយណើ ពត។

Page 155: Problem Book in Mathematical- Vol I Arithmetic

លម សវណណ វចតរ| ចននបឋម 143

ពហគណរមតចបផត.តចចក

រមធបផត

ចននបឋម

116. សនមតថា ជាចននពហគណ ហហើយថា ជាតចចករបស ខសពមយ។ ហយើងមាន នង

ហោយ ខសព ហ ោះ ជាចននពហគណ។ ដហចនោះ ហបើ មនចមនជាចននបឋម មនចមនជាចននបឋមហទ។

សមគា ល ហបើ ជាចននបឋម អាចជាចននបឋម ឬ មនបឋម។ ហបើ ជាចននបឋម កជាចននបឋមចដរ។

117. យក មានកតតា បឋមរមគនន ហហើយ

118. ជាចននបឋម។ ហោយ ធជាង ជានចចហ ោះ ហដើមបឱយ ជាចននបឋម មានចត ។ ករណ ហយើងមាន ជាចននបឋម។ 119. ចហ ោះ ជាចននបឋម។ ពនតយ

Page 156: Problem Book in Mathematical- Vol I Arithmetic

144 ពហគណរមរចបផរ.រចចករមធបផរ |លម សវណណ វចតរ

កតតា នមយៗ ធជាង ចហ ោះ ដហចនោះ មនអាចជាចននបឋមបានហទ។ 120. ហបើ ជាចននបឋម ហ ោះ វារតវចតជាចននហសស ហហើយ ឱយ ហសស។ ចហ ោះ ហយើងមាន ជាចននបឋម ករណ

ហយើងហឃើញថា ករណ ជាចននហសស ហ ោះ អាចបចបកជាផលគណននចននគតពរ ចដលធជាង ដហចនោះ វាមនអាចជាចននបឋមហទ។ 121. តតង ។

ចដល ។ ដចហនោះ ។ 122. ហយើងហឃើញថា ។ តតមរទសាបទ 3.10 រគបចននពហគណទងអសចដល មានកតតា បឋមមយ កនងចហោម ឬ ។

តរសតបរ 3.10៖

បបើចននគរវជជមាន ជាចននពហគណ ប ោះ វាមានកតតត បឋម មយ ចែល ។ តតង ជាសណចននពហគណនន ចដល 100 ។ ដហចនោះ

Page 157: Problem Book in Mathematical- Vol I Arithmetic

លម សវណណ វចតរ| ចននពហគណ 145

ជាចននននចននចដលជាពហគណនន ឬ ចដលកនងហនោះ ហយើងរាបទង ចដលជាចននបឋមចលចដរ។ ដហចនោះ ចននននចននចដលជាពហគណនន ឬ ហហើយមនចមនជាចននបឋម មានចនន ។ ដហចនោះ ចននចននបឋមចដល គ

កនងហ ោះ ដក ហរ ោះ មនចមនជាចននបឋម ហហើយកមនចមនជាចននពហគណចដរ។ 123. តតង

។ ហយើងមាន ហរ ោះហបើមនអចងហទ រគប ជាចននហសសទងអស ជាចននហសស ដហចនោះ មនអាចចចកោច ហទ ដហចនោះផទយពសមមតកមម។ ប ទ បមកហទៀតហយើងសននោា នថា ។ ហរ ោះហបើ ហ ោះ ហោយ ជាចននបឋម ហហើយ ហ ោះ រតវចតមានរាង ដហចនោះ

ចហ ោះរគប ។ ផទយពសមមតកមម។ ជាចងហរោយហយើងនងបងហា ញថា ។ ហបើ ហ ោះ

ចហ ោះរគប ។ ផទយពសមមតកមម។ ដហចនោះហយើងសននោា នបានថា ចននបឋម៣តហរៀងគនន គ ឋតហៅកនងចហោមចននចដលឱយ។

ចននពហគណ

124. ពនតយ ។ ករណហនោះ បោា ចនន សទធចតជាចននមនបឋម។

Page 158: Problem Book in Mathematical- Vol I Arithmetic

146 ពហគណរមរចបផរ.រចចករមធបផរ |លម សវណណ វចតរ

ពហគណរមរចបផរ. រចចករមធបផរ

125. តតង ។ ដហចនោះ ចចកោច

។ ដហចនោះ ចចកោច ។ ដចគនន ។ ដហចនោះ ។ ហោយ ហ ោះ

។ ដហចនោះ មាននយថា ឬ ។ 126. តតង ដហចនោះ

: ចចកោចនង 1da ។ ដចគនន

ចចកោចនង ។ ដហចនោះ (*)។ តតមរទសាបទ Bachet-Bézout ហគអាចរកបានចននគត ចដល ។ គរកតសមាគ លថា រតវចតមានសញញា ផទយគនន ។ វាមនអាចមានសញញា ដកទង២ហទ ហរ ោះ ។ វាមនអាចសញញា បកទង២ហទ ហរ ោះ ឱយ ចដលតតមពត ។ ដហចនោះ ហយើងសនមតថា ។ តតង ។ ដហចនោះ នង ។ ដហចនោះ (**)។ (*) នង (**) សហណើ ពត។ 127. ហយើងមាន

ដហចនោះ មាននយថា ចហ ោះរគបចននគត ។ ដហចនោះ មនអាចធហលើសព ហទ។ ហតើ អាចមានតនមលហសមើ ឬហទ?។ ឱយ ហ ោះ

ដហចនោះ អាចមានតនមលរហតដល ។ មាននយថា ។

Page 159: Problem Book in Mathematical- Vol I Arithmetic

លម សវណណ វចតរ| ពហគណរមរចបផរ. រចចករមធបផរ 147

128. តតង ។ ដហចនោះ រតវចតជាចននហសស នង នង ចហ ោះចននគតធមមជាត ។ ដហចនោះ ចដល

។ ដចគនន ហបើសនជា ជាចននហសស។ ហោយ ហ ោះ ហហើយហោយ ជាចននហសស ហ ោះ ។

129. បោា ចននចដលមានរាង បហងកើតបានជាសវតនព វន ា ចដលមានតចនន នង មានផលសងរម ។ សនមតថា ។ ដហចនោះ ។ មាននយថា តពរៗននសវតហនោះ បឋមរវាងគនន ។

130. ហយើងមាន ។ ដហចនោះ ហយើងរតវរកចននននចននវជ ជមានចដល ហហើយចដលចចកមនោចនង នងនង ។ តតង ជាសណននចននគត ចដលជាពហគណនន , តតង ជាសណននចននគត ចដលជាពហគណនន ,...។ល។ ហយើងដងថា

ដហចនោះចននចននគតចដលរកគ ។

131. តតង ជាតចចករបសរមរបស នង ។ ដហចនោះ ចចកោច ។ ដចគនន ចចក ោច។ ហោយ នង បឋមនងគនន ហ ោះ នង កបឋមនងគនន ចដរ។ ដហចនោះ ។

Page 160: Problem Book in Mathematical- Vol I Arithmetic

148 ពហគណរមរចបផរ.រចចករមធបផរ |លម សវណណ វចតរ

132. ហយើងដងថា

តតង ជាតចចករមរវាង នង ។ សនមតថា ។ តតមទ កទនងខាងហលើ ហោយសារ ចចក ោច ហ ោះ កចចក ោចចដរ។ ហោយសារ ចចក ោច ហ ោះ ចចក ោច។ ហោយសារ នង ជាចននហសស ហ ោះ ជាតចចកកហសសចដរ ហហើយចចក ោច មាននយថា ។

133. ហយើងមាន នង ចចកោចនង ។

សនមតថា ។ តតមរទសាបទភែមគា ហយើងមាន

ដហចនោះ

មាននយថា ចចកោចនង ។ ហោយ បឋមនង ហ ោះ ចចកោចនង ។ តរសតបរចភមាា

តតង ជាចននបឋមនងសនមរថា ចចក មនដាច ប ោះ ។ ដហចនោះ រគបចននបឋមទងអសចចកោចយាងតចតមយរបសសវតហនោះ។ មាននយថា មនអាចមានចននោមយហរៅព ចដលបឋមនងរគបតទងអសននសវតហនោះហទ។

134. នង ជាតចចករបស ដហចនោះ នង ចហ ោះ ចននគតមនអវជ ជមាន ខលោះ។ ហោយ ជា នន ដហចនោះ

នង ។ ដហចនោះ អាចជា ។ ដហចនោះហយើងមាន ចនន ចបប។ ដចគនន ហយើងមាន នង ចនន នង ចបប។ ដហចនោះហយើងមាន ចបបននចននគតវជ ជមាន ចដលមាន ។

Page 161: Problem Book in Mathematical- Vol I Arithmetic

លម សវណណ វចតរ| ពហគណរមរចបផរ. រចចករមធបផរ 149

135. យ ើងមាន

ជាចននគត យបើ ចចកដាច

។ យដា បឋមនងគនា ពរៗ យ ោះ ចចកដាច

ដចយនោះ ចចកដាច ចចកដាច ។ យដា បឋមនង ដយចាោះ ចចកដាច ។ ដចគនា យ ើងទាញបាន ចចកដាច នង ចចកដាច ។ យដា អយថរទាងអសយនោះមានលកខណៈស យមទទ យ ោះយ ើងអាចសនមតថា ធជាងយគ។ ដយចាោះ យ ើងទាញបាន ។ យដា ចចកដាច ដយចាោះ ឬក ។ ករណ អាចមានចតយពលណា (យទរោះ )។ យដា បឋមនងគនា ពរៗ យ ោះ ។ ករណ លកខខណឌ យៅជា

ជាចននគត សមមលនង

ជាចននគត។

យដា បឋមនង យ ោះកាងចយណាមចននពរយនោះ ទតវមានម ជាចននយសស។ យ ើងអាចសនមតថា យសស។ ដយចាោះ ចចកដាច សមមលនង ចចកដាច ។ ករណយនោះអាចចតយពលណា ប យណាណ ោះ (យទរោះ បឋមនង )។ រចយ ើ ចចក ដាច។ ដយចាោះ ឬ ។

យបើ យ ើងទាញបាន ។ យបើ យ ើងទាញបាន ។

ដយចាោះជាសរ បចយមលើ មាន នង ចលាសទាងអសចដលអាចមានននចយមលើ យនោះ ។

Page 162: Problem Book in Mathematical- Vol I Arithmetic

150 ពហគណរមរចបផរ.រចចករមធបផរ |លម សវណណ វចតរ

136. ហយើងពនតយផលគណ ោមយ នងចននបឋម ោមយ។ សនមតថា

។ ហោយ នងហោយ ហ ោះ

ដហចនោះមាននយថា សវយគណ ននចននបឋម ោមយចដល , មានតនមលធមនហលើសព

ហទ មាននយថា

។ ចតកនងករណ

ហ ោះ

មាននយថា កនងចហោមបោា ផលគណរបស ទងអស មានយាងហោចោសផលគណមយចដលហផទៀងផទទ ត

។ ដហចនោះ ពហគណរមតចបផតននបោា ផលគណកនងសនរ

កណតហោយ

បឋម

តតមរបមនាឌបា លញាក សវយគណធបផត នន ចដល ចចកោច កណតហោយ

បឋម

ដហចនោះ

បឋម

បឋម

137. ហយើងមាន ដហចនោះ ភាគយកនងភាគចបងរបសរបភាគហនោះ មនអាចមានកតតា រមធជាង ហទ។ មាននយថា វាមនអាចសរមលបានហឡើយ។ 138. តតង

ចដល ជាចននបឋម។ ហយើងមាន

Page 163: Problem Book in Mathematical- Vol I Arithmetic

លម សវណណ វចតរ| ពហគណរមរចបផរ. រចចករមធបផរ 151

ហយើងសនមតថា ។ សមភាពខាងហលើហៅជា

ពត។ 139. តតមរទសាបទរគោះននចនននព វន ា ហយើងទញបាន រតវចតមានរាង

។ ជាងហនោះហៅហទៀត រតវមានយាងហោចោសធាតពរ កនងចហោមធាតទងបន ចដល រតវហសមើនង ហហើយនង រតវមានយាងហោចោសធាតពរកនងចហោមធាតទងបន ចដល រតវហសមើនង ។ ហយើងពនតយករណ សន។ ហយើងមាន

ករណ (1) ទងបនមាន ដចគនន (ហយើងមាន មយករណ)។ ករណ (2) មានបកនងចហោម មាន ហហើយមយហទៀត មាន ។

ករណហនោះ ហយើងមាន រហបៀប ហរ ោះកនងោរហរជើសហរ ើសចននទបននមយៗចដល , ហយើងមាន រហបៀប, ហហើយកនងោរហរជើសហរ ើស យកបកនងចហោមបន ចដល , ហយើងមាន

រហបៀប។ ដហចនោះសរបមាន រហបៀប។

ករណ (3) មានពរកនងចហោម មាន ហហើយពរហទៀត មាន ។ ករណហនោះ ហយើងមាន រហបៀប ហរ ោះ កនងោរហរជើសហរ ើសចននទប នង បននមយៗចដល , ហយើងមាន រហបៀប ហហើយកនងោរហរជើសហរ ើស យកពរកនងចហោមបន ចដល ហយើងមាន

រហបៀប។ ដហចនោះសរបមាន រហបៀប។

សរបករណទងបបញចលគនន ហយើងមានទងអស ចបបកនងោរហរជើសហរ ើសឱយបានយាងហោចោស ពរកនងចហោមធាតបន ហដើមបឱយបានសវយគណ ។ ដចគនន ចដរ មានទងអស

Page 164: Problem Book in Mathematical- Vol I Arithmetic

152 ពហគណរមរចបផរ.រចចករមធបផរ |លម សវណណ វចតរ

ចបបកនងោរហរជើសហរ ើសឱយបានយាងហោចោសពរកនងចហោមបន ហដើមបឱយបានសវយគណ ។ ដហចនោះជាសរបហគមានចតធាតទងអសចនន ។

បចបកជាកតតត បឋម

140. ហយើងមាន ។ ដហចនោះ ជាតចចករបស លោះរតតចត អាចសរហសរជារាង បាន ចដល នង ។ ដហចនោះ ហយើងមាន នង ហរៀងគនន ចនន នង ហផេងគនន ។ ដហចនោះ មានតចចកវជ ជមានចនន

។ ហបើ ហ ោះ

កជាតចចករបស ចដរ ហហើយផលគណននត

ចចកទងពរហសមើ ។ ដហចនោះ ហយើងអាចចចកតចចកទង របស ហលើកចលងចត ហចញ ជា គតចចក ចដលមានរាង

ហហើយផលគណននតចចកពរហនោះ ហសមើ ។ ដហចនោះចហមលើយគ

។ 141. តចចកគរបស មានរាង ចដល នង ជាចននគត ចដល នង ។ ផលបកតចចកគវជ ជមានរបស ហសមើនង

142. ហយើងមាន ។ ដហចនោះតចចករបស មានរាង ចដល នង ជាចននគតចដល ។ ចហ ោះ នមយៗ ហយើងមានជហរមើសចនន យាង ដហចនោះ មានតចចកគតវជ ជមានចនន ។ កនងចហោមចននទងហនោះ បោា ពហគណនន រតវហផទៀងផទទ តលកខខណឌ ។ ដហចនោះហយើងមានជហរមើស ចនន ចបប ដហចនោះ មាន ចបប កនងចហោមតចចកចនន នន ចដលជាពហគណនន ។ ដហចនោះរបបាប លហតចដលចងបានគ

143. តតង ជាផលបកតចចកវជ ជមានទងអសរបស ។ 1) បងហា ញថា ហបើ ហ ោះ ។

Page 165: Problem Book in Mathematical- Vol I Arithmetic

លម សវណណ វចតរ| បចបកជាកតតត បឋម 153

សនមតថា ជាចននបឋម ហ ោះ ។ ហោយ

1=1 ហ ោះ

ដហចនោះ ផលបកតចចករបស ហរៅពខលនវា ហសមើនង

ដហចនោះ ជាចននសមបណណ ហលខ។ 2) បងហា ញថា ហបើ ហ ោះ ។ រាសមកវញ តតង ជាចននគតគ។ តតង ហសស ហ ោះ

ហោយ ជាសមបណណ ហលខ ហ ោះ ។ ដហចនោះ

ចហ ោះចននគតធមមជាត ោមយ។ ដហចនោះ

។ ហយើងចងបងហា ញថា ។ ហយើងសហងកតហឃើញថា

ហបើ ហ ោះ មានយាងតចតចចករបស ចនន៣ តតងហោយ ហហើយ ឱយ ផទយពោរពតចដល ។ ដហចនោះ ។ ហយើងទញបាន នង ដហចនោះ រតវចតជាចននបឋម។ ហបើ មនចមនជាចននបឋម ហ ោះ ។ ដហចនោះ

ដហចនោះ មនអាចជាចននបឋមហទ។ ដហចនោះ រតវចតជាចននបឋម នង ជាចននបឋម ហហើយ ។

Page 166: Problem Book in Mathematical- Vol I Arithmetic

154 ពហគណរមរចបផរ.រចចករមធបផរ |លម សវណណ វចតរ

Page 167: Problem Book in Mathematical- Vol I Arithmetic

លម សវណណ វចតរ| ផនែកគរ 155

ផនែកគត

144. យ ើងដងថា ចយ ោះគគបចននគត ។ យគ ោះ

ពត។ បនទា បមកយទៀតយ ើងនងបងហា ញថា គមា នចននគត ដដល យទ។ យលើកវសមភាពជាកាយេ យ ើងដងថា ដយចនោះ ។ យោ សាេ ជាចននគត ដយចនោះ ។ ដតយ ើងដងថា មនអាចជាកាយេននចននគតបានយទ ។ យគ ោះ យបើ យនទោះ ។ ដតយបើ យនទោះ ។ ដត មានន ថា មនអាចយសាើ យទ។ ដយចនោះ មនអាចជាចននគតបានយទ។ ដយចនោះ ពត។ 145. អនគមន ជាអនគមនចោះ។ ដយចនោះ ចយ ោះចននគតវជ ជមាន នង

Page 168: Problem Book in Mathematical- Vol I Arithmetic

156 ផនែកគរ |លម សវណណ វចតរ

យ ើងមាន

ដយចនោះ

146. យ ើងមាន

ជាចននគតគ។ យោ យនទោះ - កេណ ជាចននយសស យនទោះ

ជាចននគតគ - កេណ ជាចននគ យនទោះ

ជាចននយសស។

Page 169: Problem Book in Mathematical- Vol I Arithmetic

លម សវណណ វចតរ| ផនែកគរ 157

ដយចនោះមានន ថា ចយ ោះ យនទោះ មានតនមៃយសស,គ,យសស,…ឆលៃ សគមន េហ

ត។ 147. វសមភាពសមមលនង

យដើមបឱយមាន ដតម គតទាលដត

។ ចយ ោះ យ ើងទាញ

បាន ដយចនោះ មានដត ម គត។ 148. យោ យនទោះ ។ យោ យនទោះ

។ ដយចនោះ ។ ដយចនោះ

។ ដយចនោះ

ដដល មានឫសវជ ជមានដតម គតគ

។ ដយចនោះ ចយ ោះតនមៃ យនោះ យ ើងមាន

នង ។ យ ើងទាញបាន

149. យ ើងមាន

ដយចនោះ

កណតា លនន

នង

។ ដយចនោះ

យៅជត ជាង។

ដយចនោះ ។ 150. តាង

ដយចនោះយ ើងនងគណនទតនមៃេបស ដតកនងកេណ ។ យ ើងមាន

Page 170: Problem Book in Mathematical- Vol I Arithmetic

158 ផនែកគរ |លម សវណណ វចតរ

យបើ

យនទោះ ដយចនោះ

យបើ

យនទោះ ដយចនោះ

យបើ

យនទោះ ដយចនោះ

យបើ

យនទោះ ដយចនោះ

ដយចនោះ

151. យ ើងយឃើញថា យៅកនងផលបកខាងយលើ ចននតដដលខសព មានចននកណត គថា

យសាើសនយយពលដដល ។ សយណើ ៖ ចយ ោះគគបចននពត យគមាន

សគមា បញជជ ក៖ យបើ

យនទោះ

នង ។ យបើ

យនទោះ

នង ។

ដយចនោះ

152. យបើ ជាចននគតយនទោះ យ ើងយឃើញថា សមភាពពត។ យ ើងសនាតថា មនដមនជាចននគត មានន ថា ។ ក ដយចនោះ

តាម (*) យ ើងទាញបាន

Page 171: Problem Book in Mathematical- Vol I Arithmetic

លម សវណណ វចតរ| ផនែកគរ 159

ដយចនោះ

តាម (**) ។

153. តាង ។ យ ើងយឃើញថា យបើ ជាចននគតវជ ជមាន យនទោះ ។ យ ើងទាញបានថា យបើចននគត ម អាចសេយសេជារាង បានដដល ជាចននគត យនទោះចយ ោះ យគអាចសេយសេ ជារាងដដលចងបាន បានដចគមន មានន ថា ។ យោ យហតយនោះយហើ ជាដបងយ ើងគគមនដតកណតថាយតើកនងចយណតម២០ចននគតដបងម ណតដដលអាចជាតនមៃេបស ចយ ោះ ។ យ ើងសយងេតយឃើញថា យពល យកើន តនមៃេបស បាេជាតនមៃថាដតយពលណតដដល ឬ មានតនមៃគតប យណតណ ោះ យហើ យពលយនទោះតនមៃេបស យកើនធជាងមន។ យពល ដគបគបលចយនទៃ ោះ កយណើ ន ដបបយនោះ យកើតមានដតយពលណតដដល មានរាង ដដល នង ឬ ។ គបភាគដបបយនោះមានចនន១២យ ងគ

គតវនង មានតនមៃ

ដយចនោះកនងចយនទម ២០ចននគតវជ ជមានដបងមានដតចនន១២ប យណតណ ោះដដលអាចសេយសេតាមរាងចងបាន។ យោ ដយចនោះមាន ចននគតវជ ជមានដដលអាចសេយសេជារាងចងបាន។

Page 172: Problem Book in Mathematical- Vol I Arithmetic

160 ផនែកគរ |លម សវណណ វចតរ

154. ផលបកដដលឱយមាន ត ដដលតនម ៗ យសាើនង េក ។ ដត ដយចនោះមានន ថា ។ តាង ជាចននតដដលយសាើ ដយចនោះចននតដដលយសាើ មាន ។ ដយចនោះ

យោ ដយចនោះ ។ ដយចនោះ តដបងយសាើ នង តបនទា បយសាើ ។ តទ គ

នង តទ គ

។ យ ើងទាញបាន

155. ចយ ោះ តាង

យោ យនទោះ ។ ចយ ោះចននគត ដដល យោ

យនទោះ ។ ដយចនោះ មានតនមៃខសៗគមន ។ ចយ ោះចននគត ដដល យោ

យនទោះ ។ យោ ដងថា សតយនោះមនដមនជាសតថ យនទោះមានន ថា

។ដយចនោះគគបចននគតវជ ជមានទាងអសដដលមានតនមៃតចជាង ជាតននសតយនោះ។ យ ើងមាន នង ។ ដយចនោះចននតដដលមានតនមៃខសគមន គ

(តនមៃទាងយនទោះគ ) ។

156. យ ើងយឃើញថា គមា នតណតម កនងចយណតម ដដលដចកោចតម យទៀតយទ យគ ោះយបើមនអចងយទ យ ើងនងមាន ។ ដយចនោះ ដដល ជាចននយសស។ យ ើងយឃើញ

Page 173: Problem Book in Mathematical- Vol I Arithmetic

លម សវណណ វចតរ| ផនែកគរ 161

ថា ខសគមន ទាងអស។ យោ យ ើងមាន ទាងអសចនន ដយចនោះ សណេបស ជាសណចននគតយសសវជ ជមាន ដដលតចជាង ។ ឥលវយ ើង យ ើងពនតយ ។ យបើ

យនទោះ នង

។ ដយចនោះ ជាចននយសសដដល ដយចនោះ ចយ ោះតនមៃ ណតម នង ។ ដយចនោះ ឬក ។ ដយចនោះវាផា ពសមាតកមា ដដល ។

Page 174: Problem Book in Mathematical- Vol I Arithmetic

162 ផនែកគរ |លម សវណណ វចតរ

Page 175: Problem Book in Mathematical- Vol I Arithmetic

លម សវណណ វចតរ| សមការដយផង 163

សមការដយផង

157. សមការដដលចមមលើយរបសវាជាចននគតមៅថាសមការដយផង។ មយើងមឃើញថា សម

ការដយផង

មានចមមលើយជាចននគត ដតកនងករណ ប ម ណ ោះ។ បបមាណវធរបសអគលដជាមមធោបាយដមានបបសទធភាពមយ កនងការដសែងរកចមមលើយរបសសមការមនោះ។ មយើងមាន

ដមចនោះចមមលើយងាយរបសសមការមនោះ គ ។ មយើងមឃើញថា

កជាចមមលើយរបសសមការមនោះដដរ។ សមការមនោះគមា នចមមលើយមផេងពមនោះមទ (សមអានបទសតបទកនងមមមរៀនទ៣)។ 158. មយើងមាន

គណអងគសងាា ងននសមការនង មយើងទាញបាន

ដមចនោះ ។ 159. មោយ មាននយថា អងគខាងមឆែងដចកោចនង ដតអងគខាងសត ដចកមនោចនង២ ដមចនោះសមការគមា នចមមលើយជាចននគត។

Page 176: Problem Book in Mathematical- Vol I Arithmetic

164 សមការដយផង |លម សវណណ វចតរ

160. មយើងមាន ។ មយើងទាញបាន

។ 161. សនាតផទយមៅវញថា សមការមានឫសជាចននគតវជ ជមាន។ ដមចនោះ

ករណទ១ មយើងទាញបាន មនយកមបរោះ បតវដតជាចននគតវជ ជមាន។ ករណទ២ មយើងទាញបាន មនយកមបរោះ បតវដតជាចននគតវជ ជមាន ។

162. សនាតថា សមការមានចមមលើយវជ ជមាន ខសពសនយ មផេងមទៀត ម ើយមយើងតាង ជាចមមលើយដដលតចជាងមគ។ បតវដតជាចននគ

បតវដតជាចននគ

បតវដតជាចននគ

មយើងមឃើញថា កជាចមមលើយរបសសមការមនោះដដរ ដដល ។ ដមចនោះ ផទយពការសនាត ។ ដមចនោះ សមការមនមានចមមលើយ មបៅព មទ។ 163.164.

165. មបើ ។ មយើងពនតយករណ

ដមចនោះគមា នចននគត ដដលកាមររបសវាសមមលនង តាម មទ។ ដមចនោះសមការគមា នចមមលើយ។

166. ចននគបសមមលនង ។ ចននសែយគណនន២ (គចននដដលមានរាង ) សមមលនង ។ ដមចនោះ ។ ដមចនោះអងគទាង២មនអាចមសាើគមនមទ។

Page 177: Problem Book in Mathematical- Vol I Arithmetic

លម សវណណ វចតរ| សមការដយផង 165

167. បគបចននសែយគណបនសទធដតសមមលនង ។ មាននយថា

យ ងមបចើនសមមលនង ។ ដត ។ ដមចនោះ សមការ

មនអាចមានចមមលើយមទ។

168. សមការសមមលនង

ដមចនោះ នង បតវដតជាសែយគណនន ទាងពរ។ មយើងដងថាចននសែយគណនន ដដលខសគមន ចនន ឯកតា ( ) មានដត នង ប ម ណ ោះ។ ដមចនោះ ជាចមមលើយដតមយគត ម ើយ ។

169. សមការមនោះអាចសរមសរមៅជា

ជាមយគមន មនោះដដរ មយើងមាន

ដមចនោះ មបើ ម ោះ មយើងទាញបាន សមការគមា នរស។ ដមចនោះទាលដត មាននយថា ។ ករណ មយើងទាញបាន ។ ករណ មនដមនជាចននគត។

170. មោយ មានលកាណៈសមមបទនងគមន ម ោះមយើងអាចសនាតថា សន ។ តាមលកាខណឌ មនោះ មយើងមាន

មបើ ម ោះ មនអាចមបរោះ ។ មបើ ម ោះ ។ មោយ ម ោះ

Page 178: Problem Book in Mathematical- Vol I Arithmetic

166 សមការដយផង |លម សវណណ វចតរ

មោយ ដមចនោះ ។ ដមចនោះ ។ មនអាចមាន ។ មបើ ម ោះ ។ មបើ ម ោះ ។ មបើ ម ោះ

ដមចនោះ ។ ដមចនោះជាសរប ចមមលើយរបសសមការមាន នងបគបតមរៀបទាងអសដដលអាចមានននបតធាតទាងអសមនោះ ( )។ 171. តាង

។ មបើ ម ោះ

ជនសតនមល ទាងមនោះចល មយើងទាញបាន

គមា នចមមលើយជាចននគតទាងពរ។ ដមចនោះ សមការគមា នចមមលើយ។ មយើងមាន ដមចនោះ ជាចមមលើយ ទាលដត ជាចមមលើយដដរ។ ដតសមការគមា នចមមលើយចមរោះ មទ។ ដមចនោះ ប ត ចមមលើយ បតវដត ។ មយើងមាន មនដមនជាចននគប។ នង ។ ដមចនោះ មយើងទាញបានចមមលើយ នង ។ ម ើយគចមមលើយមផេងមទៀតគ នង ។

172. មយើងមាន ចមរោះបគប ។ ដតមយើងមាន ចមរោះបគប ។ ដមចនោះសមការមនោះគមា នឫស។

Page 179: Problem Book in Mathematical- Vol I Arithmetic

លម សវណណ វចតរ| សមការដយផង 167

173. ក) បតវដតជាចននសែយគណនន ដមចនោះ ។ សមការមៅជា ដមចនោះ ជាចននគ។ ដមចនោះ ជាចននគ ម ើយ ។

ខ) ។ មោយ បឋមនង មបើ ម ោះ នង បតវដតជាសែយគណនន ទាងពរ។ ដតមនដដលមានសែយគណនន ដដលខសគមន ចនន ឯកតាមទ។ ដមចនោះសមការគមា នចមមលើយ។

គ) មោយ ។ ដតមយើងដងថាចននកាមរ សមមលនង ឬ ។ ដមចនោះសមការគមា នចមមលើយ។

174. សនាតថាសមការមានចមមលើយ។ សនាតថា ជាចមមលើយវជ ជមានឬសនយ ដដលតចជាងមគ កនងចមមចមមលើយដដលអាចមានននសមការ។ ជាព គណនន ដមចនោះ ជាព គណនន ។ តាង ។ មយើងទាញបាន

ដមចនោះ ដដល កជាឫសរបសសមការ ដដរ។ ដតមយើងបានសនាតថា ជាចមមលើយដដលតចជាងមគ ដមចនោះ មានដត ។ 175. សមការមានឫសងាយ

នង

ភាគដបងរបស

មសាើនង ។ មបើមយើងគណ នង ដដល

ជាចននគត ម ោះមយើងទទលបានដផនកទសភាគរបស មៅរកាតនមលមដើមដដដល។ មបរោះ

គត

ដមចនោះ ប ត ចនន

Page 180: Problem Book in Mathematical- Vol I Arithmetic

168 សមការដយផង |លម សវណណ វចតរ

សទធដតជាចមមលើយរបសសមការ។ ដមចនោះសមការមានឫសមបចើនរាបមនអស។

176. មយើងមាន

នង

ដមចនោះ នង ។ ដមចនោះ

នង ។ ដមចនោះ ដដល ជាចននគតដដលដចកមនោចនង ។ មោយ ម ោះសមការ មនអាចមានចមមលើយមទ។ ដមចនោះសមការគមា នចមមលើយ។ 177. សមការអាចសរមសរមៅជា

មោយសកាមៅមលើអនគមន

មយើងទាញបានថា អនគមនមនោះជាអនគមនមកើនមលើ

ម ើយចោះមលើ ។

ដមចនោះ មយើងទាញបានថា មបើ នង ជាចននគតពរខសគមន (សនាតថា ) ដដល

ម ោះ

វាបតវដត នង ។ ចម ល ោះ មៅ មយើងមានចននគតដត មយគត។ ដមចនោះ នង ។ សរបសមការមានចមមលើយ ។

178. តាង ។ មយើងមាន

ចមរោះ មយើងមាន

Page 181: Problem Book in Mathematical- Vol I Arithmetic

លម សវណណ វចតរ| សមការដយផង 169

ដមចនោះមាននយថា ចមរោះ មគមាន ដចកោចនង ដតដចកមនោចនង ដដល ជាចននគតដចកមនោចនង ។ មោយ ម ោះ បតវដត មបរោះ ។ មយើងមាន ។ ចមរោះ មយើងមាន

ដមចនោះ មនអាចមសាើ បានមទ។ មយើងទាញបាន ថា សមការគមា នចមមលើយមទ មពល ។ មយើងពនតយករណនមយៗខាងមបកាម ចមមលើយសមការ : គមា នចមមលើយ ចមមលើយសមការ មនអាចជាចននសែយគណមទ មនអាចជាចននសែយគណមទ មនអាចជាចននសែយគណមទ មនអាចជាចននសែយគណមទ 179. តាង ជាចមមលើយរបសសមការ មបើសនជាមាន។ មយើងមាន

មបើ ជាចននមសស ម ោះ នង បឋមនងគមន ម ើយជាចននគបពរដដលខសគមន ឯកតា។ ករណមនោះមនអាចមាន។ ដមចនោះ ជាចននគ ជាចននគដដរ។ សមការមៅជា

ដមចនោះ ជាចននគ។ ដមចនោះមយើងទាញបាន

។ ដមចនោះ

ដមចនោះ នង បតវដតជាចននគ ឱយ ជាចននមសស។ ដមចនោះ

មោយ នង បឋមនងគមន ម ោះ វាបតវដតគបទាងពរ ដមចនោះ ម ើយ ។

Page 182: Problem Book in Mathematical- Vol I Arithmetic

170 សមការដយផង |លម សវណណ វចតរ

ដមចនោះ សមការមានចមមលើយពរគត គ ។

180. មបើ ម ោះ មយើងទាញបាន ។ សនាតថា ។ មបើមគមនមាន មទ ម ោះមានមយកនងចមម នង ដដលបតវដតធជាង ោចខាត(នងមយមទៀតតចជាងោចខាត) ដមចនោះ ធជាងឬមសាើ ។ មបើសនជាមយម ោះជា មយើងទាញបាន

ផទយពសមាតកមា។ មបើសនជាមយម ោះជា ម ោះមយើងទាញបាន

កផទយពសមាតកមាដដរ។ 181. មយើងដងថា ជាចមមលើយរបសសមការជានចច។ ម ោងវញមទៀត មបើ ជាចមមលើយមយននសមការ (*)ម ោះ ជាឫសរបសព ធា

ម ើយ ឫសមយមទៀតរបសព ធាមនោះ គ (មបរោះ )។ ដមចនោះមយើងទាញបាន សមការ(*)មានឫស ។ ដចគមន សមការ(*)មានរស ។ ជាសរបមយើងទាញបានឫសរបសសមការ(*) មាន ។ ម ោងវញមទៀត មបើ ម ើយមបើ នង មយើងមាន - មបើ ម ោះ

មាននយថា មបើ ជាចមមលើយ ម ោះ មយើងអាចរកបាន ចមមលើយមផេងមយមទៀត កណតមោយ ដដល ។ - មបើ ម ោះ មាននយថា មបើ ជាចមមលើយ ម ោះ មយើងអាចរកបាន ចមមលើយមផេងមយមទៀត កណតមោយ ដដល ។

Page 183: Problem Book in Mathematical- Vol I Arithmetic

លម សវណណ វចតរ| សមការដយផង 171

តាង នង ។ តាមលកាណៈ ដចដដលមយើងបានមរៀបរាបពខាងមដើម មយើងទាញរកសែតននចមមលើយ ដដល មោយ មោយ មោយ ..................... លកាខណឌ ចងមបកាយមនោះ ធា ថា ចមមលើយទាងអសរបសសមការខសគមន ពរៗ។ កនងករណមនោះសមការមានឫសមបចើនរាបមនអស។ មបើ សមការសរមសរមៅជា មានចមមលើយមបចើនរាបមនអស។ ដចគមន ករណ ។ មបើ ម ោះសមការ មៅជា ។ ប ត ចនន នង បតវដតជាចននវជ ជមាន ។ ដមចនោះ សមការមានឫសគត រាបមនអសមទ។ ដចគមន មពល ។ ជាចងមបកាយ មបើ ម ោះសមការមៅជា

ដដលឫសរបសសមការមនោះមានដត មានចមមលើយមានចននកណត។ ដមចនោះ មដើមបឱយសមការមានចមមលើយមបចើនរាបមនអស សណចននគត, បតវដត ។

182. មយើងមឃើញថា សមការមានឫស ។សនាតថា សមការមានចមមលើយមផេងមទៀត។ តាង ជាចមមលើយតចបផតរបសសមការដដលខសពសនយ។ - មបើ ដចកោច ឬ ម ោះ បតវដតដចកោច នង ទាងពរ ដមចនោះ ដចកោចនង ។ ដមចនោះ ដចកោច ។ តាង ។ ដមចនោះ

។ ដមចនោះ តចជាង កជាចមមលើយសមការដដរ។ ករណមនោះផទយពការសនាតដដល ជាចមមលើយតចបផត។ - ដមចនោះ នង បតវដតបឋមនង ។ មយើងមាន

Page 184: Problem Book in Mathematical- Vol I Arithmetic

172 សមការដយផង |លម សវណណ វចតរ

តាមបទសតបទ Bachet-Bézout មយើងអាចរកបាន នង ដដល ។ ដមចនោះ ។ ដមចនោះ ។ ដតចននកាមរ មនអាចសមមលនង តាម មទ។ ដមចនោះសមការមានចមមលើយដតមយគតគ ។ 183. ក) ដបងមយើងសនាតថា ។ មយើងទាញបាន

។ មនអាចមសាើ ឬ មទ។ មបើ សមការមៅជា

ដមចនោះ ចមមលើយរបសសមការ គ ។ ខ) តាមរមបៀបដចខាងមលើ មយើងទាញបានករណ នង សមការគមា នឫស។ ចមរោះ មយើងយក ។ មយើងនងបងាា ញថា មនមផទៀងផទទ តមទ។ មបរោះមបើ មផទៀងផទទ ត មោយ ម ោះ មយើងទាញបាន ដមចនោះ ។ ដចគមន មយើងទាញបាន រចម ើយ

មនអាច។ មោយដងថា

ម ោះ

ដមចនោះ ជាចមមលើយមយរបសសមការករណ ។ មយើងមាន

ដមចនោះ ជាចមមលើយមយរបសសមការករណ

។ មយើងដងថា មបើ មផទៀងផទទ តសមការ ម ោះ កមផទៀងផទទ តសមការដដរ មាននយថា មបើ មផទៀងផទទ តលកាខណឌ សមការមានឫស ម ោះ កមផទៀងផទទ តដដរ។ ដមចនោះ បគបចននគតវជ ជមាន ខសព សទធដតមផទៀងផទទ តលកាខណឌ ទាងអស។

184. សនាតថា ជាសណចមមលើយដដលតចជាងមគ។

Page 185: Problem Book in Mathematical- Vol I Arithmetic

លម សវណណ វចតរ| សមការដយផង 173

មោយគណអងគទាងពរនង មយើងទាញបាន

គណ សមមលតាម សមការមនោះមៅជា

មោយចននកាមរ តាម សមមលនង ម ោះ

ដមចនោះមានដត ។ ដមចនោះ នង បតវដតដចកោចនង ។ មយើងមាន

មោយអងគខាងមឆែងដចកោចនង ដមចនោះបតវដត ។ មយើងមាន

ដមចនោះ បតវដតដចកោច នង ។ ដមចនោះ ដចកោច ។ មយើងទាញបាន

ជា

ចមមលើយមយមទៀតរបសសមការដដលតចជាងមន។ ករណមនោះផទយពការសនាត។ ដមចនោះសមការគមា នចមមលើយមផេងពសនយមទ។

185. មយើងមាន

ដមចនោះ អាចសរមសរជាផលបកននចននគបបន។ មយើងមាន

មបរោះ ។ ដតថាបគបចននគបសទធដតសមមលនង តាម ។ ដមចនោះមយើងទាញបានថា ផលបកននចននគបបឬតចជាងមនោះ មនអាចសមមលនង តាម មទ។

ដមចនោះ ។

Page 186: Problem Book in Mathematical- Vol I Arithmetic

174 សមការដយផង |លម សវណណ វចតរ

186. សមការ

មានរសងាយ មបរោះ

សនាតថាសមការមានចមមលើយមផេងមទៀត ។ សមការមានលកាណៈសមមបទ។ សនាតថា ។ មបើ ម ោះ ។ មបើ ម ោះ ។ ដមចនោះ ។ មយើងសរមសរសមការ(*)ជាសមការដមបកទ២មធៀបនង ។ មយើងទាញបានថា មបើ ជាឫសមយ ម ោះ ជាឫសមយមទៀត។ តាម (**) មយើងមាន ដមចនោះ ជាចមមលើយមយមទៀតដដលធជាងមន (គ )។ ឧទា រណ ជាឫសមយរបសសមការ ម ោះឫសមយមទៀតកណតមោយ

ជាឫសរបសសមការ(*)។ បតមជហានមនោះ ឫសរបសសមការ(*) មនដមនមានដត នង ដតតាមពត ចមាល សរបសឫសមនោះកជាឫសរបសសមការដដរ ដចជា ។ ប ទ បមកមទៀត ចមរោះចមមលើយថាមនោះ គ មយើងមរៀបឫសសមការមនោះតាម មាននយថា កជាឫសរបសសមការដដរ។ ចមរោះ មយើងទាញបានឫសថាមយមទៀតដដលធជាងមន គ ។ ដមចនោះមយើងទាញបានឫសបដនថមរបសសមការ (*) មាន នងចមាល សរបសឫសមនោះ ដដលកនងម ោះមាន ដដរ។ ដមចនោះជាបនតប ទ ប មយើងទាញបានធាតតចជាងមគមចោះដតមកើនមៅៗ។ ដមចនោះជាចងមបកាយ មៅប ទ បពប ា នជហានមបកាយមកមយើងនងទាញបានចមមលើយដដលមាន ដដលនមយៗធជាង ទាងអស។

187. មបើ ជាតដចកបឋមរបស ម ោះ ដចកោច ម ើយដចកមនោច មទ មបរោះ

បឋមនង ។ ដតមយើងមាន

Page 187: Problem Book in Mathematical- Vol I Arithmetic

លម សវណណ វចតរ| សមការដយផង 175

ដមចនោះ កដចកមនោច ដដរ។ ដមចនោះ បឋមនង ។ ដមចនោះ តាមទ កទនង (*) មយើងទាញបាន ជាចននមានរាង ។ ដតវាមនអាចមានមទ មបរោះ ។ ដមចនោះសមការគមា នចមមលើយ។

188. មបើ ម ោះ ។ ដមចនោះ

។ តាង ។ មយើងនងបងាា ញថា ជាចននគត។ មយើងមាន

ជាចននគត ជាចននគត (មបរោះ ជាចននសនទាន)។ មយើងមាន

មបើ មយើងទាញបាន ។ មបើ ម ោះសមភាព (*) មនមានមទ មពល ។ ចមរោះ មយើងទាញបាន ម ើយចមរោះ មយើងទាញបាន ។ ចមមលើយរបសសមការគ ។ មបើ ម ោះមយើងមាន

ដមចនោះ វសមភាពមនោះមនអាចពតមទ មបើ

។ ដមចនោះ មយើងទាញបានចមមលើយសមការ ។ ជាសរបសមការមានចមមលើយ ។

189. ក) បគបចននគត សទធដតមផទៀងផទទ តទាងអស មលើកដលងដតករណ មចញ។ មបរោះ មបើ មយើងយក ។ មបើ មយើងយក នង ។ មយើងនងបងាា ញថា មនមផទៀងផទទ តមទ។ មយើងសនាតផទយថា មានចននគត ដដល

ទ កទនងមនោះបងាា ញថា បគបចននបឋមដដលដចក ោច សទធដតដចក ោចទាងអស។ មយើងមឃើញថា មបើ ម ោះ ម ើយ មបើ ម ោះ

។ ដមចនោះវាបតវដត ។

Page 188: Problem Book in Mathematical- Vol I Arithmetic

176 សមការដយផង |លម សវណណ វចតរ

តាង ជាចននបឋម ដដលដចកោច ដមចនោះដចកោច ដដរ។ តាង ។ មយើងមាន

ប ទ បមកមទៀត មោយដងថា បគបចននបឋម ដដលដចកោច សទធដតដចក ោចដដរ ម ើយ ម ោះមយើងទាញបានថា ជាព គណនន ដតវាមនអាចមបរោះ ។ ដមចនោះ

មនមផទៀងផទទ ត។ ខ) សមការមានឫសងាយ ។ សនាតថា ជាចននគត ដដល

តាមរមបៀបបសយបញជជ កដចកនងសនរក) មយើងទាញបានថា ម ើយថា ដចកោច ។ ដមចនោះ ដដល ។ សមការមៅជា

ចមរោះ មយើងមាន មនអាច។ ចមរោះ មយើងទាញបាន មនអាច។ ចមរោះ មយើងទាញបាន ។ មយើងអាចមផទៀងផទទ តថាវាជាចមមលើយរបសសមការ។ ដមចនោះសមការមានចមមលើយ ។

190. តាង នង ដដល នង ជាចននគតបឋមនងគមន ។ សមការមៅជា

មយើងទាញបានថា ដចកោច ដមចនោះ ចមរោះចននគត ។ សមការបដលងមៅជា

បតវដតជាចននគត ម ើយមោយវាជាចននវជ ជមាន

តនលរបស

តចជាង ោចខាត មពល ។ ដមចនោះ មបើ ម ោះ

Page 189: Problem Book in Mathematical- Vol I Arithmetic

លម សវណណ វចតរ| សមការដយផង 177

ម ើយសមការ (*) មៅជា

ជាសមការដមបកទ២មធៀបនង ដដលមាន ។ មនអាចជាចននកាមរបានមទ មបរោះ ។ ដមចនោះ ករណមនោះ គមា នចមមលើយ។ ដមចនោះមៅសលករណ នង មទៀត។ ចមរោះ មយើងមាន

ដមចនោះ មយើងទទលបានចមមលើយពរគ ។ មបើ មយើងទាញបាន

ដមចនោះ ។ មយើងទទលបានចមមលើយពរមទៀតគ ។ សរបសមការមានចមមលើយគ ។

191. មបើ ជាតដចករមរបស នង ម ោះ ដចកោច ដមចនោះ ដចកោច ។ ដមចនោះ ដចកោច ម ើយ ជាតដចករមរបស ។ តាមសមាតកមា ។ ដមចនោះ បឋមនងគមន ។ ដចគមន មយើងទាញបាន បឋមនងគមន ពរៗ។ មបើ នង មសសទាងពរ ម ោះ មយើងមាន

មនអាច មបរោះចននកាមរសមមលនង តាម ។ ដមចនោះបតវដតមានមយជាចននគ។ សនាតថា ជាចននគ។ តាង ។ សមការមៅជា

មោយ ជាចននគ ម ោះមបើ គ មយើងទាញបាន គដដរ, ដតមបើ មសស ម ោះ មសសដដរ។ កតាតទាងពរ នង គទាងពរដចគមន ។ មោ ងវញមទៀត មបើ ជាតដចករមធបផតរបស នង ម ោះ ដចកោចផលបកនងផលសងននចននទាង២ មាននយថា ដចកោច នង ។ ដមចនោះ ដចក ោច មបរោះ បឋមនងគមន ។ មាននយថា ។ ដតមោយ

Page 190: Problem Book in Mathematical- Vol I Arithmetic

178 សមការដយផង |លម សវណណ វចតរ

នង គទាងពរដចគមន ម ោះមាននយថា ។ មយើងទាញបាន ចននគត

នង

ជា

ចននបឋមនងគមន ។ មោយផលគណរបសវាជាចននកាមរ ម ោះវាបតវដតជាចននកាមរទាងពរ

ចមរោះចននគតវជ ជមាន បឋមរវាងគមន ។ មោយជនសចលកនងសមការ មយើងទាញបាន (មបរោះសនាតថា វជ ជមាន) ។ ជាបញចប នង មានលកាណៈគមសសខសគមន មបរោះមបើ មសសទាងពរ ម ោះ នងជាចននគទាងពរ ដត បឋមនង ។

192. ក) តាង ជាចននគតវជ ជមាន ដដល នង

ជាចននកាមរ។ សនាតថា

ជាចមមលើយតចបផត ដមចនោះ ពកបឋមរវាងគមន ពរៗ។ តាមបទសតបទពតាករ មយើងអាចរកបាន ដដល

ដដល នង ជាចននវជ ជមាន នង បឋមនងគមន ម ើយមានលកាណៈគមសសផទយគមន ។ ដមចនោះបកឡានផទបតមកាណមៅជា ។ មោយ នង មានលកាណៈគមសសផទយគមន ម ោះ នង មសសទាង២ ដមចនោះបឋមនងគមន ដដរ។ ដមចនោះ កតាត ទាង៤សទធដតបឋមនងគមន ២ៗ ម ើយនមយៗជាចននកាមរ។ ដមចនោះ មានចននគត នង ចននគតមសស ដដល

មយើងមាន មបរោះ មសសទាងពរ។ ដមចនោះ ជាចននគ។ តាង ម ោះ

ដមចនោះបតវដតមានយ ងមហាចមយកនងចមម

នង

ជាចននគ។ មោយ នង មសស

ទាង២ ម ោះ មានដតមយប ម ណ ោះកនងចមម

នង

ដដលជាចននគ។

មបើ

ជាចននគម ោះ

មោយ

នង

បឋមនងគមន ម ោះ មាន ដដល

Page 191: Problem Book in Mathematical- Vol I Arithmetic

លម សវណណ វចតរ| សមការដយផង 179

2 24 ; 2c d s c d r (*) ដមចនោះ បតធាត ជាចមមលើយមយមទៀតដដរ ដដល ។ ផទយពការសនាត ដដលថា តចបផត។ ដមចនោះសមការគមា នចមលើយ។

ខ) តាង ជាចននគតវជ ជមានោចខាត ដដល

តាង នង ។ មយើងមាន

ដមចនោះ ,X Y នង Z ជារងាែ សបជងរបសបតមកាណដកងមយ ដដលមានបកលានផទជាចននកាមរ។ តាមសនរ ក) ករណមនោះអាចមៅរចលោះបតាដត នង ។ ដត ដមចនោះចមទគមា នចមមលើយ។

193. សនាតថា បតធាតដបបម ោះមាន។ កនងចមមចមមលើយទាងម ោះ មយើងយក ដដល ឬកជាចមមលើយដដលតចជាងមគបងអស។ មៅកនងលកាខណឌ អសមនោះ តាមបទសត

បទតរធារពតាករ ម ោះមគមានចននគត ដដលបឋមនងគមន ដដល

ដមចនោះ នង ចននគត មានតដចករមធបផតមសាើ ។ មោ ងវញមទៀត ជាចននមសស (មបរោះ ជាចននគ) ម ើយដមចនោះ តាមបទសតបទដដដល មគមានចននបឋមនងគមន ដដល

។ មបើ ជាតដចករមធបផតរវាង នង ម ោះ ដចក ោច។ ដត បឋមនង ដមចនោះ ។ ដមចនោះ បឋមរវាងគមន ពរៗ ម ើយ ជាចននកាមរ។ ដមចនោះ ចនននមយៗបតវដតជាចននកាមរ មាននយថា

Page 192: Problem Book in Mathematical- Vol I Arithmetic

180 សមការដយផង |លម សវណណ វចតរ

មយើងទាញបានថា ជាឬសរបសសមការដដរ ដដល

។ ដមចនោះវាផទយពការសនាត ដដលថា ជាចមមលើយវជ ជមានដដលតចជាងមគ។ ដមចនោះសមការគមា នចមមលើយគតវជ ជមានមទ។

194. ចមទមនោះសមមលនង ការកណតបគបចណចដដលមានកអរមោមនជាចននសនទាន ឋតមៅមលើរងែងមយដដលមានការងាែ សមយឯកតា។ មៅមលើរងែងមនោះ មយើងយកចណច ដដលមានកអរមោមនជាចននសនទាន។ មយើងគសប ទ ត មយ(មនឈរ)កាតតាម ។ ប ទ តមនោះកាតរងែងបតងចណច មយមទៀត។

តាមពតមៅចណច មានកអរមោមនជាចននសនទាន លោះបតាដត មមគណបបាបទសរបសប ទ តមនោះជាចននសនទាន។ មបរោះ មបើ មនដមនជាប ទ តឈរមទ សមការប ទ ត មានរាង

ចណចបបសព ែរវាង ជាមយរងែងមផទៀងផទទ ត

សមការមនោះមានឫសមយមសាើ ។ ផលបកឫសទាង២របសសមការ មសាើ

។ ឫសមយមទៀតមានតនមល

អរមោមន កណតមោយ

Page 193: Problem Book in Mathematical- Vol I Arithmetic

លម សវណណ វចតរ| សមការដយផង 181

ចននពរមនោះជាចននសនទានមបើ ជាចននសនទានដដរ។

បាសមកវញ មយើងមឃើញថា មបើចណច នង មានកអរមោមនជាចននសនទាន ម ោះប ទ ត

ជាមមគណបបាបទសជាចននសនទាន។ ដមចនោះ មយើងបានបងាា ញថា បគបចមមលើយសនទាន មបៅព កណតមោយ

195. មយើងមឃើញថា នង ជាចមមលើយមយ។ តាង ជាចននសនទាន ដដល ។ សមការមៅជា

ចមមលើយរបសសមការមនោះគ នង

។ តនមល បតវគមន

។ ជាបញចប

ចមមលើយរបសសមការគ

ចមរោះបគប ជាចននសនទាន។

196. មយើងនងបងាា ញថា សមការមនោះគមា នឫសមទ។ មយើងដងថា ។ ដមចនោះ

មយើងសរមសរ មៅកនងមគមល២

មោយ ឬ ។ ដមចនោះ

បកសមភាពទាងអសមនោះបញចលគមន មយើងទាញបាន

Page 194: Problem Book in Mathematical- Vol I Arithmetic

182 សមការដយផង |លម សវណណ វចតរ

សមការមនោះមនអាចមានចមមលើយមទមបរោះ

197. លោះបតាដត ដដល នងបាសមកវញ ម ើយករណមនោះមកើតមានមបើ នងបាសមកវញ។ ដមចនោះ សមការដដលឱយមានឫស លោះបតាដត ។ វសមភាពមនោះមានចមមលើយ

198. មយើងមាន

សក មតងមយៗ មយើងមឃើញថា មានដត ដតប ម ណ ោះដដលមផទៀង

ផទទ ត។ ដមចនោះ ឫសសមការគ

។ បាសមកវញ មយើងយកចមមលើយទាងមនោះ

មៅមផទៀងផទទ តសរម ើងវញកនងសមការ មយើងនងមឃើញថា ឫសទាងមនោះជាចមមលើយពតបបាកដ។

199. មយើងមាន ចមរោះបគបចននពត ។ បកអងគនងអងគននសមការ មយើងទាញបាន

ជនសសមការមបកាយមនោះចលកនងសមការទាង៣ដដលឱយ

ដចគមន មយើងទាញបាន ។ ដចគមន ។

200. មយើងមាន

Page 195: Problem Book in Mathematical- Vol I Arithmetic

លម សវណណ វចតរ| សមការដយផង 183

មបើ ម ោះ កតាត ផលគណខាងមលើខសគមន ទាងអស។ ដមចនោះមយើងមានទាងអស៥កតាត ។ មយើងមាន មាននយថា អាចបដបកបានជាផលគណននយ ងមបចើនចននគត មផេងគមន ។ ដមចនោះ វាមនអាចមសាើគមន មទ។ មបើ កមនាមខាងមលើមៅជា ម ើយវាកមនអាចមសាើ ដដរ។

201. ចននគតវជ ជមាន មានចនន ។ ចននគតវជ ជមាន សទធដតខសគមន ទាងអស ម ើយមានចននសរប ។ សណ មានធាតសរបចនន ។ មយើងមាន មបរោះ

។ ធាតនមយៗននសណមនោះ មានតនមលធមនមលើសព

មទ ដមចនោះវាបតវមានយ ងមហាចសធាតពរដដលមានតនមលមសាើគមន ។ មោយ សទធដតខសគមន ទាងអស នង សទធដតខសគមន ទាងអសដដរ ម ោះ ធាតពរដដលមសាើគមន ម ោះ បតវដតជា មយ នង មយ។ ដមចនោះ ។ សែត

ដដល

មផទៀងផទទ តសមាតកមា។ មយើងមាន

មបើ ម ោះ ។ មោយ ដមចនោះសមការ គមា នចមមលើយ។ មបើ ម ោះ សមការ គមា នចមមលើយដដរ។ ដមចនោះករណ

សមការគមា នចមមលើយ។ ដមចនោះម ើយវាបតវដត

202. មយើងមាន

ម ោងវញមទៀត មយើងមាន មបរោះ ពមបរោះ

។ ដមចនោះ ជាព គណនន ។ មោយ មសស ម ោះ មនដមនជាព គណនន មទ ដមចនោះ នង មនអាចជាព

Page 196: Problem Book in Mathematical- Vol I Arithmetic

184 សមការដយផង |លម សវណណ វចតរ

គណនន ទាងពរបានមទ។ មយើងទាញបានថា មានមយកនងចមមចននពរមនោះ ដចកោចនង ។ ដត

ដមចនោះ មានដត ជាព គណ នន ។ មយើងមានវសមភាពមយមផេងមទៀត

ដមចនោះ ។ មបើ ម ោះ បតវដតដចកោច ដមចនោះ បតវដតមានរាង ។ មោយ នង សទធដតជាចននមសសទាងពរ ម ោះ មាននយថា នង បឋមនងគមន ។ តាមរមបៀបដចគមន មយើងបងាា ញថា នង បឋមនងគមន តាមរយៈ

។ ជាបញចប ដចកោច ម ើយបឋមនង នង នង ។ ដមចនោះ ។

203. មបើ ម ោះ សមណើ ពត។ មបើ ម ោះ ជាចននបឋមមសស។ ចមរោះចននមសស មយើងមាន

ដចកោច ។ មបើ ម ោះ កដចកោច ដដរ។ ដមចនោះ ដចកោចនង ។ មោយ ដមចនោះ ដចកោច មាននយថា មបរោះ ជាចននបឋម។ ដត មនដមនជាចននសែយគណមទ។ ដមចនោះ សមណើ ពត។

204. មយើងតាង ជាគតនមល ដដលមផទៀងផទទ តលកាខណឌ ។ មបើ ម ោះមានដត នង ប ម ណ ោះ ដដលមផទៀងផទទ តលកាខណឌ ។ ឥលវមយើងសនាតថា ជាគតនមលដដលមផទៀងផទទ តលកាខណឌ ។ មោយ ម ោះបតវដត ។ តាង ។ ដមចនោះ ម ើយ

Page 197: Problem Book in Mathematical- Vol I Arithmetic

លម សវណណ វចតរ| សមការដយផង 185

ដមចនោះ កជាគតនមលដដលមផទៀងផទទ តលកាខណឌ ដដរ។ ដមចនោះ មយើងទាញបានគ ដដលមផទៀងផទទ តមាន

(គប ទ បមកមទៀត គ មនយកមបរោះមានតនលធជាង )។ ម ើយតនមលធបផតរបស គ ។ មតើមយើងមធែ ើដចមមតច មទើបដងថាគមា នគតនមល មផេងមទៀត មបៅពគតនមលខាងមលើមនោះ។ សនាតថាមានគតនមល មផេងមទៀត ដដលមផទៀងផទទ តលកាខណឌ ដដរ។ សនាត ។ មយើងតាង ។ មយើងអាចបងាា ញថា កជាគតនមលដដលមផទៀងផទទ តលកាខណឌ ដដរ។ មាននយថា មបើមាន មផទៀងផទទ ត ម ោះ កមផទៀងផទទ តដដរ។ ដមចនោះមានប ត គតនមល ដដលមផទៀងផទទ តលកាខណឌ នងមានតនមលកានដតតចមៅៗ ម ើយបតវចបមៅបតម គ ។ ដតគតនមល មានដត នង ដតប ម ណ ោះ។ ដមចនោះ មាននយថា មនអាចមានគចមមលើយ មបៅអព គចមមលើយ ដដលបានមរៀបរាបខាងមលើមទ។

205. តាង ជាតនមលតចបផតដដលមយើងបតវកណត។ មយើងមាន

។ ដមចនោះ ដចកមនោចនង ផង នងនង ផង។ ដចគមន ដដរ ដចកមនោចនង ។ ដមចនោះមយើងទាញបាន ឬក ។ ករណ មនអាចមៅមកើតមទ មបរោះ

ដមចនោះ (មពល )។

206. មយើងមាន ជាចននមសស។ ដមចនោះបតវមានមយកនងចមម នង គ នង មយមទៀតមសស។ មោយ នង សទធដតជាចននបឋម ម ោះបតវដតមានមយមសាើ ។ ជាដបងមយើងសនាតថា ។ ប ទ បមកមទៀត ជាចននបឋមមសស ដដល

Page 198: Problem Book in Mathematical- Vol I Arithmetic

186 សមការដយផង |លម សវណណ វចតរ

មបើ ជាចននមសស ម ោះ

ដមចនោះ ជាព គណនន ដដលជាចននមសសធជាង ោចខាត។ ដមចនោះវាមនអាចជា មទ។ មយើងទាញបានថា ជាចននគ។ តាង ។ ករណ : ជាកាមរននចននមសស ដមចនោះ វាសមមលនង ។ ដមចនោះ ។ តនមល មនោះមយើងមនយក មបរោះ ។ ករណ : ផលគណ ជាសែយគណនន ដមចនោះ កតាត នមយៗបតវដតជាសែយគណនន ដដរ។ ចននសែយគណនន ចននពរ ដដលខសគមន ឯកតា មនមានអែមបៅដតព នង មទ។ ដមចនោះ នង ដមចនោះ ។ ដមចនោះសមការមានចមមលើយ ។

207. មោយ នង ជាសែតនព ែន ត តាង នង ដដល នង ជាចននគត។ នង ជាផលសងរមរបសសែត នង មរៀងគមន ។ មបើ ម ោះ ជាសែតមថរ។ ចមរោះបគប មគមាន នង ជាចមមលើយរបសសមការ

ដមចនោះ ឌសបគមណង បតវដតជាចននគតកាមរ ចមរោះបគបចននគត ។ មយើងមានសមភាព

ដដល មនអាសសយនង ។ ដមចនោះមបើ ចមរោះ ធបគបបគមនមយ ម ោះមយើងមានវសមភាព

ដតមោយ បតវដតជាចននគតកាមរ ចមរោះបគប (មទាោះធបគបបគមនរមនបគបបគមន) ម ោះ

ដមចនោះ ។ ដមចនោះសមការខាងមលើ មានឫសមសាើនង

។ ដមចនោះ ចមរោះបគប មយើងមាន

ឬ ។

Page 199: Problem Book in Mathematical- Vol I Arithmetic

លម សវណណ វចតរ| សមការដយផង 187

208. សនាតថាផទយថា មាន នង ជាចននបឋមនងគមន ដដល

។ ដមចនោះ

អងគខាងសត បតវដតដចកោចនង នង នង ។ មោយ នង ជាចននបឋមនងគមន ដមចនោះ បតវដតដចកោច នង បតវដតដចកោច ។ មោយ សទធដតជាចននមសស ម ោះ នង បតវដតជាចននមសសដដរ។ ដមចនោះ

មោយ ជាចននមសស ម ោះ ។ ដមចនោះ

ដតផទយព (*)។ ដមចនោះការសនាត

មនពត សមការគមា នឫសសនទាន។ 209. មយើងមាន ។ សនាតថា ចមរោះតនមលខសៗគមន នន ។ ដមចនោះ

ដដលកនងម ោះ ព ធា មានមមគណជាចននគត។ សនាតថា មានចននគត ដដល ។ ដមចនោះ

មោយកតាត មានតនមលខសគមន ទាងអស ម ោះ បតវដតអាចបដបកជាផលគណននយ ងមហាចស ចននគតខសគមន ដដរ។ ដតវាមនអាច មបរោះ ជាចននបឋម អាចបដបកបាន ដតពរកតាត ប ម ណ ោះ គ ។

Page 200: Problem Book in Mathematical- Vol I Arithmetic

188 សមការដយផង |លម សវណណ វចតរ

Page 201: Problem Book in Mathematical- Vol I Arithmetic

189

ឯកសារដ ើម

ស ៀវសៅស េះដកស ងសេញពស ៀវសៅខាងសរោមស េះ

1. Pierre Bornsztein, Xavier Caruso, Pierre Nolin, Mehdi Tibouchi, Inégalité Cours

d’Arithmétique, 2004

2. David A. Santos, Number Theory for Mathematical Contests, 2005

3. D.O. Shklarsky, N.N. Chentzov, I. M. Yaglom, The USSR Olympiad Problem

Book, Dover Publications, INC. New York, 1993.

4. Dusan Djukic, Vladimir Jankovic, Ivan Matic, Nikola Petrovic, The IMO

Compendium, Springer, 2006

Page 202: Problem Book in Mathematical- Vol I Arithmetic

កមមងលហាតគណតវទយា,

កមមតវទយាលយ,

ភាគ ១- នពវនត

ដោយ លម សវណណ វចមត

ដសៀវដៅកណណលហាតណផែកនពវនត ណែលរមមានកណណលហាតដលើបនស តកកវទយា នងមទយសដនន

ចនន ណែលនយាយអពភាពណចកោច តណចករមធបផត ពហគណរមតចបផត ចននបឋម

សមការមានឫសជាចននគត។ល។ ដសៀវដៅដនេះមបជដោយលហាតងាយ នងពបាកចនន

មបណហលជា២០០លហាត ែកមសងដចញពការមបឡងសសសពណកដៅមបដទយសនានាជវញ

ពភពដោក។ ដបើអែកចងកាា យជាសសសពណកមាែ ក ដសៀវដៅមយកាលដនេះ ជាដសៀវដៅណែល

អែកគរណតមាន៕